Anda di halaman 1dari 148

1

Russian Federation Ministery of Health State Educational Board on High Professional Education

Sechenovs Moscow Medical Academy Department of Pathophysiology

Litvitsky P.F., Pirozhkov S.V.

COLLECTED CLINICO-PATHOPHYSIOLOGICAL SITUATIONS AND CLINICO-LABORATORY PROBLEMS

Moscow

2010

Editors:
LITVITSKY PIOTR FRANTSEVICH Head of the Department of Pathophysiology of the Sechenov's Moscow Medical Academy, correspondent member of the Russian Academy of Medical Sciences, academician of the International High School Academy of Sciences, professor, M.D. PIROZHKOV SERGEI VICTOROVICH - professor of the Department of Pathophysiology, M.D.
This textbook is prepared for students and teachers of the English-speaking Division of the Department of Pathophysiology.

The textbook for students studying the basic course of pathophysiology in English contains clinico-pathophysiological situational and clinico-laboratory problems which are recommended for discussion during practical classes; also included are questions to prepare for classes, control tests and examination. The order in which the problems are placed in the textbook agrees with the plan of lectures and classes on pathophysiology for the third-year students. Versions of solutions of typical problems are presented. : . .., - , , , . - , . , . , "" , - , , , . . . . .., 2010 . .., 2010

PREFACE
Collection of clinico-pathophysiological situational and clinico-laboratory problems is a constituent part of the educational methodical complex in English, which also includes manual "Concise lectures and tests on pathophysiology". - P.F.Litvitsky, S.V.Pirozhkov, E.B.Tezikov. All the materials are prepared by the collaborators of the Pathophysiology Department of the Sechenovs Moscow Medical Academy in consideration with the State educational standard of higher professional education (year 2000) qualificational (professional) characteristics of medical institutes' graduates, modern Programmes on pathophysiology (year 2003) and clinical pathophysiology (year 2002), candidates degree minimum examination on speciality in "Pathophysiology" 14 00 16 (1997), confirmed by the Ministry of Education of the Russian Federation and by the Ministry of Public Health of the Russian Federation. The aim of the collection of problems is to provide students with an educational material to develop skill, as well as to improve the obtained ability of pathophysiological analysis of real clinical situations. Conducting of pathophysiological analysis is a model of the physician's behavior with a patient (construction of the scheme of diagnostical search for causes and risk factors of disease, mechanisms of its development, substantiation of theraputical and prophylactical measures). Pathophysiological analysis and formulation of medical comment on specific data about the patient are a version of highly delicate and peculiar phenomenon - intellectual modelling of physician's actions during solution of professional problems. Collection of situational and clinico-laboratory problems is intended for three categories of English-speaking students: 1) the 3rd year students, assimilating basic knowledge of pathophysiology; 2) students in their last year taking a course of studies in clinical pathophysiology (dedicated to the most common "typical" clinical syndromes, nosological forms, and emergency conditions); 3) physicians specializing in specific branches of medical science during internship, residency, or research post-graduate students within the frameworks of special educational course in clinical pathophysiology

(dedicated to analysys of the most frequently observed diseases and diseaseous conditions of humans). Scheme of work over each problem presupposes pathophysiological analysys of the data on patient with determination of the most probable cause and conditions evoking illnes or pathological process, stages of their pathogenesis, and mechanisms of the symptomes' development. This stage of solving the problem shapes a pattern of one of the most important physician's action proper diagnosis and its substantiation. Next stage involves formulation (if necessary) and substantiation of etiotropic, pathogenetic and symptomatic therapy, and also prophylaxis of illness and diseaseous state. Such (or similar) scheme of work over problems is conditioned by model questions formulated after description of each situation. They could be changed or added (by professor or partner on situational game) with a purpose of problem's simplification or complication. Situational problems could be used either as typical, standard ones or as non-typical ones demanding ability to apply knowledge under new nonstandard conditions, or as unforeseen situations requiring creative approach to their solution. The above approach will allow to achieve the main aim of educational course of pathophysiology at medical high school - to form the grounds of rational physicians thinking, as well as effective performance of the future doctor. Examples of conduction of pathophysiological analysis and formulation of conclusions for some problems (marked by starlet) are given at the end of the collection. But sometimes other variations of problems solution are possible (depending on additional questions being put by professor or by the collegue on the situational "game"). "Concise lectures and tests on pathophysiology" (Moscow 2002) and the manual of "Pathophysiology"(P.F.Litvitsky. - M. - GEOTAR -MED. - 2002) serve as source of information, which is necessary and minimally sufficient for analyzing and formulating conclusions on problems and also for the answers with regard to tests. Besides that, other modern manuals, quides and monographes can be used. Authors of the edition would be grateful for useful advice, suggestion, or recommendations.

Head of the Pathophysiology department of Sechenov's Moscow Medical Academy, correspondent member of the Russian Academy of Medical Sciences, professor P.F.Litvitsky

CONTENTS Page:
I. CLINICO-PATHOPHYSIOLOGICAL SITUATIONS AND QUESTIONS TO PREPARE FOR CLASSES: 1. Nosology. 2. Cell injury and cell death. 3. Hereditary disorders. 4. Disorders of regional circulation and microcirculation. 5. Inflammation. 6. Disorders of the host defence system 7. Disorders of thermoregulation. Fever. Infectious process. 8. Hypoxia. 9. Derangements of carbohydrate metabolism. 10. Derangements of lipid metabolism. Atherosclerosis. 11. Disorders of water exchange. Edema. 12. Disturbances of acid-base balance. 13. Abnormal tissue growth. Tumors. 14. Emergency states. 15. Pathophysiology of the red blood cell system. Erythrocytosis. Anemia. 16. Pathophysiology of the white blood cell system. Leukocytosis and leukopenia. 17. Malignancies of hematopoietic cells. Leukemia. 18. Pathophysiology of hemostasis. 19. Myocardial ischemia (coronary insufficiency). Arrhythmia. 20. Heart failure. 21. Disorders of the systemic blood pressure. Hypertension and hypotension. 22. Pathophysiology of respiratory system. 87 66 69 74 79 82 61 8 11 14 18 22 26 34 36 42 45 48 51 52 55 56

23. Pathophysiology of digestion. 24. Pathophysiology of the liver. Jaundice. 25. Pathophysiology of kidneys. 26. Pathophysiology of the pituitary, adrenals and sex glands. 27. Pathophysiology of thyroid gland. 28. Pathophysiology of the nervous system. Disorders of locomotion and sensation. 29. Neuroses. II. CLINICO-LABORATORY PROBLEMS III. EXAMPLES OF CASES DISCUSSION

93 96 98 103 108 114 117 119 139

I. CLINICO-PATHOPHYSIOLOGICAL SITUATIONS AND QUESTIONS TO PREPARE FOR CLASSES: NOSOLOGY Questions to prepare for classes and examination: 1. What is the specific subject matter of pathophysiology? What features distinguish pathophysiology from other medical sciences? 2. Methods of pathophysiology. Modeling of pathological processes and diseases. Means and limitations of experimental pahology. 3. The notion of "nosology". 4. What parts nosology is divided into? 5. The hallmarks of the notions of "well-being" and "disease". 6. The notions of "pathologic process", pathologic state", "pathologic reactions", "typical pathologic processes", "remission", recurrence","complication". 7. Principles used in classification of diseases. 8. Factors of diseases: environmental, genetic, social. 9. The relation between causes and conditions in the development of diseases. 10. The nature and characteristics of pathogenic factors. 11. Reactivity of the body and its role in the development of disease. 12. The notions of "pathogenesis", "vicious circles". 13. Stages of disease. 14. Adaptive reactions and their role in resuscitation and pathology. N1 A mouse is put in a small pressure chamber. Then, during 2-3 min air is pumped out of it to decrease the pressure in the chamber down to 170-180 mm Hg. After 0.5-1 min in the hypobaric conditions the mouse shows signs of anxiety: running over, scratching its face. Two or three minutes later clonic and tonic seizures develop; the mouse

is lying on its side, urinates; terminal breathing ("gasping") develops. Soon afterwards, breathing stops, and the mouse dies. The overall time of the mouse's life in the hypobaric atmosphere is 3-4 min. Questions: 1. What factors affect the mouse in this experiment? 2. Which of these factors could be considered the leading cause of the pathologic process - hypobaric hypoxia? 3. How can you experimentally test your presumptions about the nature of the pathologic process? N2 Air is pumped out of the pressure chamber to bring the barometric pressure to approximately 20-30 mmHg. Then the chamber is filled with pure O2 up to normal atmospheric pressure 750 mm Hg. A mouse is carefully put in the chamber so that the barometric pressure inside remains unchanged. The mouse is then observed as in the previous experiment. At first, seeking behavior is seen. Later the mouse is sitting quietly, showing no signs of pathology. Within approximately 10 minutes after the start of the experiment the mouse is taken out of the chamber, and its normal behavior and condition is established. Questions: 1. Taking into account the results of this test what can you say about the pathologic factors which caused death of the mouse in the previous experiment? N3 A pressure chamber is filled with a mixture of gases composed of 95% N2 and of 5% O2. Barometric pressure inside the chamber is maintained equal to normal atmospheric pressure. A mouse is put in the chamber. A thin jet of the same gas mixture is being passed through the chamber. Partial pressure of O2 in this gas mixture is about 37 mm Hg. After staying in the chamber for 10-15 min the mouse

10

develops seizures and then dies. Considering the results of all 3 experiments, answer the following questions: 1. Which factor caused the development of acute hypobaric hypoxia and death of the mouse? 2. What is the significance of hypobaria itself, that is, decreased pressure of the inhaled air? N4 Three mice are used in the following experiment. Mouse 1 is put in the water basin with water temperature 30oC. Five minutes later mouse 2 is put in the same basin. Ten seconds later both mice are taken out of the water and put in the pressure chamber. The third mouse (intact) is also put in the same chamber. The air is pumped out as in the experiment N 1 to bring the barometric pressure in the chamber down to 170-180 mm Hg. The intact mouse dies after 3-4 min of staying in the hypobaric conditions. Mouse 2 dies after 6-7 min, and the mouse 1 survives 15 min of hypoxia. At this point the experiment is stopped. When mouse 1 is taken out no signs of pathology are observed. Questions: 1. What differences in resistance to hypoxia are seen in this experiment? 2. How can we explain the observed variations in sensitivity to hypobaric hypoxia? 3. Does hypothermia caused by evaporation of water from the surface of the body affect the resistance of mice 1 and 2 to hypobaric hypoxia? How can we name this factor in terms of pathophysiology? N5 Three mice are used in the following experiment. Mouse N 1 is narcotized by a subcutaneous injection of morphine. This mouse is used in the experiment after it loses righting reflex indicating deep sleep. Mouse 2 is administered a central nervous system

11

stimulator phenamine. Mouse 3 serves as control. All three mice are put in the pressure chamber, and the air is pumped out as in the experiment N 1. Mouse 2 dies during the first 1-2 min of its staying in the chamber, mouse 3 - during the 4th min; and mouse 3 survives 15 min in hypobaric conditions. At this point the last mouse is taken out of the chamber and examined after it has waken up. It shows no signs of pathology. Questions: 1. What are possible mechanisms of differences in reactions of the mice to hypobaric hypoxia? 2. How can we modify reactivity of the body toward hypobaric hypoxia? CELL INJURY AND CELL DEATH Questions to prepare for classes and examination: 1. The notion of cell injury. Causes of cell injury. 2. Typical forms of cell injury. a) Dystrophy. b) Dysplasia. c) Derangement of the subcellular structures and components. d) Necrosis, apoptosis 3. General mechanisms of cell injury: a) Derangements of the energy supply and utilization. b) Loss of the integritiy of the cellular membranes. c) Ionic and water dysbalance. d) Changes in the cellular genome or disorders of its realization. e) Disorders of intracellular regulatory mechanisms. 4. Three main etiopathogenic variants of cell injury. I. Ischemic and hypoxic injury. II. Free radical-induced injury. III. Toxic injury. 5. Reversible and irreversible ischemic/hypoxic cell injury. 6. Ischemia/reperfusion injury. 7. Antioxidative system of the cell. 8. Components of apoptosis.

12

a) Signaling pathways that initiate apoptosis. b) Control and integration. c) Execution phase. d) Removal of dead cells by phagocytosis. 9. Adaptive mechanisms operating during cell injury. N6 A technician of the chemical laboratory was not careful working with a toxic volatile chemical. He dropped a flask containing the toxic substance, and smashed it. Before he left the room he had inhaled the noxious vapours of the chemical. Two days later he was admitted to hospital with the following complaints: malaise, somnolence, headache, nausea, back pains, blood in the urine. Blood analysis: erythrocytes 2.7*1012/L, Hb 80 g/L, platelets 120*109/L, leukocytes 3.1*109/L; compensated acidosis (metabolic and renal). The resuls of the special blood biochemistry analysis: increased concentration of free fatty acid, lipid hydroperoxides, and adenosinephosphate; elevated total creatinephosphate kinase (CPK) activity and potassium content. Questions: 1. One of the consequences of the patient's poisoning is a significant depletion of cells in the peripheral blood. What are the possible mechanisms of this effect? 2. Given the results of the special blood tests what mechanisms of the cell injury can you propose? Which of them caused damage to the membrane of the formal particles? Explain your point of view. 3. What are the origin and consequences of acidosis in this case? 4. Why did the signs of intoxication develop in the patient not immediately but 2 days after the incident? N7 In two monozygous sibs (8 months of age) an examination revealed a significant increase in the liver size (hepatomegaly), decreased fasting level of blood glucose (hypoglycemia), and lack of fasting blood glucose changes after the adrenaline administration. Liver

13

biopsy investigation showed an increased content of glycogen and a significantly diminished activity of glycogen phosphorylase in the hepatocytes. Questions: 1. Give the definition of the pathologic process developed in the infants? 2. What are the possible causes of this process? 3. Explain the sequence of events leading to glycogen accumulation in the liver cells in this form of pathology. 4. Explain the association, if any, between the low activity of glycogen phosphorylase in the patients' liver and a) an increased content of glycogen in the hepatocytes; b) hepatomegaly; c) decreased fasting blood glucose levels; d) lack of hyperglycemic effect of adrenaline. N8 Two series of experiments were performed with an aim to determine the effect of a new antibiotic drug on the skin epithelium. In in vivo studies using rats the solution of the tested drug was applied to the skin surface. The employed dose of the antibiotic was much higher than the therapeutic one. The results of this experiment were assessed by in vivo microscopy studies which were performed during the first 24 hours after the end of drug application with 8-hour intervals. In the in vitro experiments the tested drug was added to the suspension of epithelial cells isolated from the same stock of rats. Six hours later the cells were washed out by repetitive centrifugation and suspending in the same media containing no tested drug. The effects of the antiobiotic were studied at the same time points using regular light microscopy and electron microscopy. The results of the experiments: 1. In vivo: 8 hours after the completion of the drug application the morphological analysis of the epithelial cells revealed the signs of dystrophy and focal necrosis; the extent of pathologic alterations increased by the end of the 24 hour observation period.

14

2. In vitro: no signs of the cellular or subsellular damage were seen in the isolated epithelial cells after the drug treatment. The only changes observed included a reversible cell aggregation found in the first test (8 hours after the drug withdrawal), but not in analyses that followed. Questions: 1. How can you explain the difference in results of the in vivo and in vitro experiments? 2. Was damage to the epithelial cells in the in vivo test direct or indirect? Explain your point of view. 3. What are the possible mechanisms of the injurious action of the drug on the epithelium? 4. Taking into account the results of the in vitro experiments, which of these mechanisms are the most likely? 5. What processes or cell functions should be studied in the further in vivo experiments to confirm your presumptions about the mechanisms of the pathogenic effect of the tested drug? HEREDITARY DISEASES Questions to prepare for classes and examination:

1. The concept of hereditary and congenital disorders, phenocopies.


Classification of genetic disorders.

2. Classification of mutations and mutagens. 3. Mendelian disorders: definition and patterns of inheritance. 4. Codominance, pleiotropism, genetic heterogeneity. 5. Definition of penetrance and expressivity. Factors determining
penetrance and expressivity. 7. Features of autosomal dominant disorders. Examples of disorders transmitted by autosomal dominant trait. 8. Features of autosomal recessive disorders. Examples of disorders transmitted by autosomal recessive trait. 9. Features of sex chromosome-linked inheritance. Examples of disorders transmitted by X-linked trait. 10. Biochemical and molecular basis of single-gene (Mendelian) disorders. 11. Features of multifactorial (polygenic)disorders. The most common

15

multifactorial disorders. 12. The concept of chromosomal disorders, their mechanism. Mosaicism. Types of chromosomal aberrations: deletion, balanced and Robertsonian translocation, ring chromosome, inversion. 13. Examples of the most common chromosomal disorders. Monosomies and trisomies.

N9 N., a healthy woman, visited a genetic counseling unit for consultation. She told her doctor that her father suffered from color blindness, but her mother had no vision problems. N. was anxious about the risk of color blindness for her future children. Questions: 1. What are the manifestations of color blindness and the pattern of its inheritance? 2. Can N. carry the gene of color blindness in her genotype? 3. If she can, what is the probability of disease and the probability that her children (separately boys or girls) will be carriers of the abnormal gene? N 10 N., a healthy woman, whose father suffered from hemophilia A and mother was healthy, went to a genetic counseling unit. She was anxious about the risk of hemophilia to her grandsons. Her husband, as well as her son and two daughters were healthy. Questions: 1. What is the pattern of inheritance of hemophilia A? What are the main features of this type of inheritance? 2. What is the risk of the disease descending from her son to her grandson, and from her daughter to the grandchildren? 3. What are the etiology and pathogenesis of hemophilia A? 4. Can this disease have lethal and sublethal forms? N 11

16

Patients S., a pregnant woman, went to a genetic counseling unit for consultation. She told her doctor that her sister suffered from phenylketonuria. In her husband's pedigree there are marriages between close relatives, but none of the children have had phenylketonuria. A thorough examination of the patient and her husband revealed no signs of pathology. Questions: 1. How great is the risk of phenylketonuria to patient S.'s sons? 2. What are the etiology and basic mechanisms of phenylketonuria? Is gender significant for its inheritance? 3. What are the main manifestations of the disease and their pathogenesis? 4. What is the approach to early diagnostics of phenylketinuria in the newborns? 5. Is it possible to prevent phenylpyruvate oligophrenia in children? N 12 Patient M., male, 21 years old, suffers from hereditary form of hypogammaglobulinemia. His father also suffers from this disease; mother is healthy. An examination showed a significant decrease of IgM and IgG levels in the patient's blood. Questions: 1. What are the etiology, basic mechanisms and clinical manifestations of hypogammaglobuliemia? 2. What is the pattern of its inheritance? 3. Characterize the genotype of the patient's mother with respect to this pathology, and also the possible genotypes of the patient's siblings. How much is the risk for the patient's sibs to develop hypogammaglobuliemia? 4. Does the pattern of immune and allergic reactions change in hypogammaglobulinemia? Substantiate your opinion. N 13 In a genetic counseling unit female patient Z. told the doctor that her sister suffered from severe sickle cell disease, but she and her husband were healthy. Z. was concerned about the degree of risk of sickle cell

17

disease to her future children. To answer this question the doctor investigated the types of Hb in the patient's and her husband's blood. The test revealed that in the patient's erythrocytes there were 70% of HbA and 28% of HbS; in her husband's erythrocytes there were only 98% of HbA and no HbS. Questions: 1. What is the pattern of inheritance of sickle cell disease? 2. What are chances for the patient's children to develop sickle cell disease? What is the probability that her children would be healthy carriers of the abnormal HbS gene? 3. Does the development of sickle cell disease depend on the sex? 4. What factors can aggravate the course of the disease? N 14 Population studies of the frequency of various diseases in the city of N. among monozygotic (MZ) and dizygotic (DZ) sibs revealed the following: Frequency in MZ a) schizophrenia b) scarlet fever c) poliomyelitis Questions: 1. What is the value of the Holtzinger factor for these diseases? 2. Estimate the role of genetic and environmental factors in development of these diseases? N 15 A male patient, 3 years old, was admitted to a pediatric department. On examination: signs of growth retardation, flat facial profile, half-open mouth, oblique palpebral fissures and epicanthic folds are evident; transverse skin folds on the palms. Study of the patient's karyotype revealed the following: 46,XY,+t(+14,21). Questions: 1. What disease does the patient suffer from? 87% 95% 44% Frequency in DZ 4% 94% 39%

18

2. Describe and characterize the patient's karyotype? By what features does it differ from the normal one? 3. What are the possible causes and pathogenesis of this disease? 4. Are other variants of karyotype modification possible in this disease? Which of the variants is the most common? DISORDERS OF THE REGIONAL CIRCULATION AND MICROCIRCULATION Questions to prepare for classes and examination:

1. Arterial hyperemia: definition, causes, types (physiologic and pathologic),


mechanisms (neurogenous, neuromyoparalytic and humoral) and consequences. Reactive hyperemia. Manifestations of arterial hyperemia and its mechanisms. 2. Venous hyperemia: definition, causes and consequences. Manifestations of venous hyperemia and their mechanisms. 3. Ischemia: definition, causes and consequences. Manifestations of ischemia and their mechanisms, factors that influence on the consequences of ischemia. 4. Stasis: types and causes. 5. Disturbances of microcirculation: types and causes. 6. Types of transmural disturbances of microcirculation, mechanisms of increased vascular permeability. Mechanisms of extra and intravascular disturbances. 7. Sludge syndrome: types, causes, and mechanisms. N 16 A 68-year-old male patient D. presented with chronic hepatitis and liver cirrhosis. The doctor introduced a needle into the abdominal cavity to perform aspiration of ascitic fluid. By the fifteenth minute, after 5 L of fluid had been removed, the patient felt bad, complaining of weakness, dizziness, and nausea. But the procedure was continued. After 1.5 L more fluid had been evacuated the patient developed syncope and lost consciousness. Several minutes later, after the emergency treatment the patient regained

19

total consciousness but still complained of weakness, dizziness, and nausea. Questions: 1. What was the doctor's mistake during the performance of ascitic fluid aspiration? 2. What are the mechanisms of syncope after the removal of ascitic fluid? 3. What are the possible mechanisms of adaptation of the brain circulation in this case? 4. Why did the adaptive mechanisms turn out to be insufficient in this patient? N 17 A 61-year-old female patient has been suffering from arterial hypertension for many years. During the last 2 years she began to feel cold intolerance in the legs; numbness and pain in the gastrocnemius muscles on walking, and then at rest. These symptoms were more intense at night and caused sleep disturbances. Six months ago she developed a skin lesion on the front surface of her right shin; later it transformed into ulcer. The ulcer was painless but resistant to therapy. During the visit to the doctor the patient complained of dryness in the mouth, constant feeling thirsty, frequent urination. On examination: the skin of the right shin was dry, pale, and cold; no pulsation of the sole artery was detected. Repeated blood tests showed increased levels of cholesterol, fibrinogen; high platelet counts; glucose 190-210 mg/dl. The urine is positive for ketone bodies and glucose. Questions: 1. What forms of pathology can be observed in the patient? 2. Name the likely type of lesion affecting the patient's arteries? Name and characterize the main mechanisms of pathology underlying changes in the patient's arterial wall. 3. What factors caused the development of erosion and ulcer on the patient's shin? 4. Can you assume the presence of the microcirculatory disorders in the vascular system of the patient's right leg? What are the likely causes of these disorders?

20

N 18 A 44-year-old male patient K. suffering from atherosclerosis and coronary heart disease resistant to pharmacologic treatment underwent an excision of the upper cervical sympathetic ganglion and a dissection of the sympathetic nerves innervating the heart. Deep thermography of the myocardium performed during the surgery revealed temperature normalization in the previously "cold" (compared to the adjacent tissue) region of the middle one third of the left ventricle anterior wall. The surgery results in a reduction of frequency and severity of angina pectoris episodes. Questions: 1. What are the possible mechanisms of the ischemic heart attacks in this patient? Substantiate your opinion? 2. How can you explain the fact of severe ischemia in only the middle one third of the left ventricle anterior wall, despite the systemic nature of the atherosclerotic lesions involving coronary arteries? 3. What caused the decrease in temperature in the "cold" (compared to the adjacent regions) zone of the patient's heart before the surgery? 4. Can we consider the "sludge phenomenon" as one of the possible mechanisms of reduced temperature in the ischemic regions of the heart? If you think so, explain its possible causes and mechanisms in this case. N 19 A 56-year-old male patient presented with complaints on fatigability and pains in the gastrocnemius muscles when walking. The symptoms were relieved by rest. This is called the symptom of "intermittent claudication". In addition, he complained of increased sensitivity to cold, numbness, pins-andneedles, tingling sensation in his legs at rest. The patient had a long history of heavy smoking since his teens. His occupation required working outdoors even in cold seasons when he sometimes suffered from cold. The patient's examination showed that skin on both soles was pale and felt cool and dry; the nails crumbled; no pulse was felt on posterior tibial arteries on both legs. The preliminary diagnosis was thrombangitis obliterans. Questions:

21

1. What form of organ circulation disorders is observed in the presented patient? Name its characteristic features. 2. What are the possible causes and mechanisms of this form of the circulation disorders? 3. What is the possible outcome of the circulation disorder in this case? 4. What are the likely mechanisms of development of each symptoms seen in the patient? N 20 An ambulance came 10 min after the vehicle accident, and the doctor started the emergency aid to the accident victim with a blunt trauma of the chest and open fracture of the right leg. A tourniquet was applied to the injured leg to stop severe hemorrhage; then an analgesic (morphine) was injected subcutaneously into the injured leg to prevent shock. In addition, the heart stimulating drugs were injected. However, despite the morphine injection the patient kept groaning of pain and suffered from progressing shortness of breath. The patient's blood pressure was 60/35 mm Hg, pulse rate 126/min. Auscultation of the lungs revealed frequent and depressed breathing on the left side; no sounds were detected on the right side. The consciousness became obtunded. Cyanosis of the skin and mucosal membranes was progressing. The doctor subcutaneously injected a drug stimulating the respiratory center to activate breathing. However, it had no effect, and the patient lost consciousness. Questions: 1. What pathologic processes developed in the victim of the accident? Substantiate your answer by the reported data. 2. Which of the processes is the most pathogenic? Explain its etiology. 3. Describe the main stages of pathogenesis of the mentioned pathologic process. 4. Why did the injections of morphine and the analeptic drug fail to improve the patient's condition?

22

N 21 A 36-year-old female patient who has been suffering from diabetes mellitus for more than 13 years presented with visual losses manifested as blurring of vision; "sand in the eyes" when reading small prints. On examination: a significant decrease in vision acuity, narrowing of lateral visual fields in both eyes; irregular thickening of the retinal microvessels wall; capillary microaneurysms and mural thrombi in the retinal microvessels; focal serous exudates and hemorrhages in the retina, and its neovascularization. The doctor informed the patient that her vision impairment was the result of the diabetic microangiopathy. The latter includes pathologic alteration of the orbital microvessel wall. The patient received appropriate advice and treatment. Questions: 1. What types of microcirculatory disorders can you distinguish in the patient's retina? 2. What specific types of microcirculatory disorders can arise from: - capillary microaneurisms; - irregular microvessels wall thickening; - mural microthrombi. Explain their pathogenesis. 3. Which form (or forms) of microcirculation disorders may give rise to focal retinal edema? 4. Can the microcirculatory changes described produce vision impairment in the patient? Explain the relevant mechanisms. INFLAMMATION Questions to prepare for classes and examination:

1. Definition and significance of inflammation. General features of acute and


chronic inflammation. Causes of inflammation.

2. Primary and secondary alteration in the focus of acute inflammation:


causes, mechanisms of development, manifestation. Physico-chemical changes in the focus of inflammation. 2. Exudation. Definitions of an exudate and transudate.

3. Sequence and mechanisms of vascular changes in acute inflammation.

23

4. Mechanisms of vascular leakage in acute inflammation. 5. Stages of extravasation. Mechanisms of margination, rolling, firm
adhesion and transmigration. Inflammatory mediators that influence on these processes.

6. Chemotaxis: definition, exogenous and endogenous chemoattractants,


regulatory (receptor) mechanisms of chemotaxis.

7. Phagocytosis: stages, major opsonins, opsonic and nonopsonic


phagocytosis, mechanisms of bacterial killing (oxygen free radicals, products of secondary and lysosomal granules). Regurgitated and frustrated phagocytosis.

8. Defects in leukocyte function. Defects in leukocyte adhesion and


intracellular digestion. 9. Local and systemic signs of inflammation.

10. Definition of inflammatory mediators and their classification. 11. Sources, nature and effects of inflammatory mediators: vasoactive
amines, complement, kinin, and clotting systems, arachidonic acid metabolites, platelet-derived factor, cytokines and chemokines, nitric oxide, lysosomal constituents. N 22 A 46-year-old patient B. suffers from gastric ulcer. He has been admitted as an emergency into the surgery department with a preliminary diagnosis of gastric perforation. On admission: acute pain in the epigastrial region, rigid abdomen, tenderness, signs of peritoneal irritation; body temperature 38.5oC; absence of bowel sounds; marked leukocytosis; increased erythrocyte sedimentation rate. Analysis of the aspirated fluid (400 ml of opalescent fluid was evacuated during the needle drainage of the abdomen) showed the presence of a large amount of leukocytes, protein 4%, various types of bacteria including anaerobic. A plain radiograph of the abdomen demonstrated the presence of air under the diaphragm. The patient was transported to the operation room. Questions: 1. Can you confirm or disprove the preliminary diagnosis of gastric perforation? Substantiate your conclusion.

24

2. Define the type of pathologic process in the patient on admission. 3. What are the causes and mechanisms of each of the symptoms presented by the patient? 4. Define the type of fluid evacuated from the patient's abdominal cavity. Describe the characteristic properties of this fluid and explain the mechanisms of its formation. N 23 Two industrial workers visited a primary care physician with heat burns of the shin area. They received them during the boiler breakdown accident. They presented with a similar set of complaints of headache, pain and swelling of skin in the injured area. On examination: patient A. had red, hyperemic shins with marked edema; patient B. had, in addition, blisters filled with a yellowish transparent fluid. Both patients received treatment recommendations, and were accorded a sick-leave. However, both of them ignored the doctor's advice. Three days later the patient A.'s condition improved, and the patient B.'s condition worsened. The latter developed extensive edema and pain in the injured areas, his body temperature increased up to 38.90C; the area of the heat burn was covered with numerous blisters filled with pus. Bacteriologic analysis of the purulent contents revealed the presence of staphylococci. Questions: 1. What pathologic process(es) developed in the given patients? What blood test can confirm the presence of this process in the victims of the industrial accident? 2. How can you explain the different course of development of the same pathologic process in the patients? 3. What mechanisms underlie the clinical symptoms in the patient B.? 4. Why did the noninfectious pathogenic factor (high temperature) cause the emergence of blisters with pus in the patient B.? N 24 A 25-year-old patient A. visited gastroenterologist with complaints of acid belching, discomfort in the epigastrium, sensation of fullness in the stomach

25

after meal, and frequent diarrhea. Gastroduodenoscopy and the complete blood count test with serum protein analysis were performed to specify the diagnosis. The results of the gastroduodenoscopy: gastric mucosa is swelled with enlarged, tortuous, closely adjoining folds; sulci between the folds are deep. Overt signs of arterial and venous hyperemia of mucosa, and focal erosions are observed. The volume of mucus is increased. In the blood: signs of anemia and leukocytosis; dysproteinemia due to increase in globulins fraction. Questions: 1. What pathologic process(es) can be distinguished in this patient? Substantiate your opinion. 2. Which of the pathologic processes is primary, and which of them are secondary? 3. What are the possible causes and mechanisms of symptoms observed in the patient? N 25 With an aim to perform a pathophysiologic analysis of factors determining the unfavorable course of posttraumatic inflammation physicians of the Traumatology Department studied medical histories of their patients. All patients were distributed into 3 groups depending on the type of the observed physiologic dysfunction. The first group included patients who exhibited signs of liver damage; the second group consisted of patients with cardiovascular pathology (atherosclerosis, essential hypertension etc.), and the third group comprised elderly patients with various cerebral disorders (stroke, concussion of the brain, cerebrovascular atherosclerosis). Questions: 1. What pathophysiological grounds has distribution of patients into these three groups? Substantiate your opinion. 2. What other criteria (parameters) could be used to classify patients into groups for studies of various aspects of inflammation? DISORDERS OF THE HOST DEFENCE SYSTEM Questions to prepare for classes and examination:

26

1. Typical forms of disordered immunologic reactivity. 2. Principles of classification of immunodeficiency states. 3. Primary immunodeficiencies: classification and clinical manifestations. 4. Forms of primary immunodeficiency (causes, mechanisms, clinical manifestations): a) B system-dependent: b) T system-dependent: c) A-system-dependent; d) complement-dependent; e) combined. 5. Secondary (acquired) immunodeficiency states. AIDS 6. Physiologic and pathologic tolerance. 7. Graft-versus-host disease. 8. The notion of the "hypersensitivity reaction", "allergy", "atopic allergy". 9. Types of allergens. Exogenous and endogenous allergens. Heterologous, homologous and autologous allergens. 10. Stages of allergy. a) Immunologic stage. b) Pathochemical stage. c) Clinical manifestations. 11. Anaphylactic (I) type of allergy. The nature of allergen and antibody. Mediators of the anaphylactic reaction. 12. Clinical examples of the IgE-medited diseases. Systemic and local anaphylaxis. 13. Type II hypersensitivity. The nature of allergen and antibody. Mechanisms and clinical examples of type II allergies. Complement-dependent reactions and antibody-dependent cell-mediated cytotoxicity. 14. Type III hypersensitivity. The nature of allergen and antibody. Mechanisms and clinical examples of type III allergies. Systemic and local immune complex disease. 15. Type IV hypersensitivity. The nature of allergen and antibody. 16. Autoimmune diseases. 17. Principles of diagnostics and therapy of allergic diseases. Desensitization and nonspecific approach in the treatment of anaphylactic diseases.

27

N 26 A patient who had undergone a surgical intervention for gallstones presented with a slow healing of the postoperative wound, its infection, and fever. The use of antibiotics (after sensitivity tests) had positive effect. Investigation of the patient's blood showed leukopenia due to decreased neutrophils and monocytes, decreased mobility of leukocytes and low activity of leukocytic myeloperoxidase. Similar abnormalities were found in the patient's sister and brother. Questions: 1. Taking into account the observed changes in the leukocyte counts, name the syndrome(s) which the patient suffers from. Substantiate your answer. 2. What are the possible causes, manifestations, and mechanisms of this syndrome? Are there grounds for its hereditary or congenital origin? 3. What are the mechanisms of the decreased neutrophil mobility and low activity of the neutrophil myeloperoxidase? 4. What approaches can be used to increase the antimicrobial resistance of the patient's body? N 27 A 20-year-old patient who suffered from diabetes mellitus presented with slow healing of a surgical wound, purulence, and an increase in the body temperature up to 37.20C after he had undergone the appendectomy. Treatment with antiobiotics for 6 days (after the appropriate sensitivity tests) had no effect. With an aim to find the reason for the treatment failure blood glucose determination and special neutrophil tests were performed. The results of the tests are the following: hyperglycemia (320 mg/dl) is accompanied by a decrease in the number, mobility, and microbicidal activity of neutrophils. Questions: 1. What parameters, in your opinion, should be investigated to elucidate mechanisms of the decreased mobility and microbicidal activity of neutrophils?

28

2. What are the most likely causes and mechanisms of impairment of the neutrophils function in this patient? 3. Is there any association between the abnormal phagocytosis, the infection of wound and its slow healing? If you think there is, name and characterize possible mechanisms of such an association. 4. Why is the body temperature in the patient with a purulent wound increased only moderately? 5. Why did the treatment with antiobiotics in this case have no effect? N 28 During examination of an 11-year-old patient with albinism of skin and irides a physician found multiple skin blisters filled with pus and multiple petechiae. Blood tests revealed the presence of marked anemia, thrombocytopenia, and neutropenia. Special investigation of neutrophils showed a decrease in mobility and microbicidal activity, and also the presence of large granules in the cytoplasm. Hyporesponsiveness of NK cells was also detected. Questions: 1. What type of pathology does the patient suffer from? 2. What are the likely causes of abnormal mobility and microbicidal activity of neutrophils in this patient? 3. What is the cause of this pathology and mechanisms of its main manifestations: anemia, thrombocytopenia, and leukopenia? N 29 A 28-year-old patient K. visited his physician with complaints of recurrent stomatitis, tonsillitis, tracheitis, otitis, and repeated pneumonia even in the summer time. Procedures aimed at enhancing the nonspecific body resistance to infection, such as an increase of cold endurance, had no effect. The results of the laboratory tests are the following: lymphocytes reactivity toward phytohemagglutinine (PHA) and tuberculin is normal; activity of the complement factors and levels of IgM, IgG, and IgA in the blood serum are within the normal range. The complete blood count test with differential showed no changes in erythrocytes count or Hb concentration; leukocytes

29

count is low due to a decrease in monocytes, but the content of granulocytes is normal. The phagocytic activity of macrophages is decreased by 45%. Questions: 1. Define the type of pathology found in the patient. 2. What part of the immune surveillance system is impaired in the given patient: the specific immune response or the nonspecific host defence? Substantiate your answer. 3. If the specific immunity is abnormal what part of it is defective: A, B, or T? Explain the origin of symptoms presented by the patient. 4. If the host-defence system is deficient what subsystem is responsible for the defect? Explain the mechanisms of symptoms seen in the patient. N 30 An 8-year-old patient was admitted to the Pediatrics Department. His parents were concerned about the frequent occurrence of otitis, quinsy, rhinitis, conjunctivitis, bronchitis, pneumonia, and enterocolitis in their child. The current admission was associated with a high risk of bacterial endocarditis and sepsis in the young patient. A test performed in the hospital showed the presence of leukopenia due to a decrease in lymphocytes, especially in T cells, and, to a lesser degree, in B cells, a decrease in the blood levels of IgA and IgE (by 40% and 50% respectively, compared to normal values); concentration of IgG is at the lowest normal value; proliferation of lymphocytes in the presence of phytohemagglutinine is decreased. Questions: 1. Define the type of pathology developed in the patient. Substantiate your answer. 2. What are the possible causes of this pathologic state? 3. Taking into account the results of the laboratory tests explain the mechanism of development and consequences of this type of pathology. 4. How can you explain a decrease in the lymphocytes reaction to phytohemagglutinine, and a considerable reduction in the blood content of IgA and IgE when the levels of IgG are in the normal range.

30

5. What symptoms in this child could be a direct result of the IgA and IgE depletion? N 31 A 32 year-old male patient visited his dermatologist with complaints of weakness, rigor, long-lasting sustained fever, and night sweating. About a month ago he noticed the emergence of pink-violaceous spots and nodules on his right shin with a tendency to expand and coalesce. The use of antihistamine drugs and glucocorticoids had no effect on the skin lesions. On examination: the similar lesions were found in the mouth mucosa and conjunctiva; the subclavian, supraclavicular, submandibular, preauricular, retroauricular, and tonsillar lymph nodes were enlarged. Complete blood count test showed no changes in erythrocyte count, and the presence of leukopenia due to a decrease in lymphocytes and monocytes. Over the 3-year period before the onset of the disease the patient had been working in Africa where he experienced pneumonia three times during the last year of his business trip. Questions: 1. What pathologic process(es) developed in the patient? Substantiate your answer. 2. If you see several pathologic processes which of them is primary, and which are secondary? Explain their interrelationship. 3. What is the cause, and what are the main stages of development of the primary pathologic process? Substantiate your opinion using the given data. 4. What are mechanisms of development of leukopenia, lymphopenia, and monocytopenia? What type of lymphocytes, T or B, contributes more to the of lymphopenia? Substantiate your answer. N 32 After spending an hour and a half in the countryside a 30-year-old man noticed that his eyelids became red and swollen, lacrimation and nasal secretions appeared, the voice became hoarse. He had a feeling of tightness in his chest. When he returned home the intensity of these symptoms decreased, but they were still present.

31

Questions: 1. Define the type of pathologic process developed in the patient. Substantiate your opinion. 2. What methods can be used to determine the cause of this pathologic process? 3. What are the main components of the mechanism of the given process? 4. What treatment approaches and preventive procedures can be used in this case? N 33 A 38-year-old patient K. who had ingested gold-containing drugs according to doctor's prescription, noticed, first, singular, and then multiple petechiae on the skin of his forearms, chest, back, and also in the oral cavity mucosa. Minor contusions were accompanied by extensive subcutaneous hemorrhages. Blood tests showed normal count of erythrocytes, leukocytes, and Hb concentration, but a significant decrease in the platelet count, and an increase in the IgG and IgM content. Therapeutic procedures used by the doctor improved the patient's condition, and he continued to take gold-containing drugs since other forms of medication had poor effect. Questions: 1. Define the type of pathologic reaction developed in the patient. Substantiate your answer. 2. What was the cause and mechanisms of this reaction? Substantiate your opinion by the given data. 3. What procedures could be used to inhibit such pathologic reaction to the gold-containing drugs? N 34 A 30-year-old female patient K. visited her dermatologist several times with complaints on the red, itching spots on the face, neck and hands appearing in the cold environment, such as windy cold weather. The physician prescribed her an ointment which alleviated these symptoms but did not eliminate them. One day, when she was in a hurry to get to her work,

32

she washed her face with cold water. Thirty minutes after she had gone outdoors the sites of the skin exposed to cold water turned red, swelled and itching. The patient had to visit her doctor and later was admitted to hospital Questions: 1. What pathologic process(es) developed in the patient after she had washed her face with cold water? Substantiate your answer. 2. What is the mechanism of this process? 3. What pathologic process should it be distinguished from, and what is the key difference between these two processes? 4. What groups of medicines can be used to prevent or inhibit this pathologic process? N 35 A patient with an open lower extremity trauma was repeatedly injected tetanus antitoxin in combination with antihystamines. On the ninth day after the last antiserum injection he presented with high body temperature (up to 380C), severe weakness, swelling and tenderness of the shoulder and knee joints; intensely pruritic disseminated urticarial eruptions, and the enlarged tender popliteal and inguinal lymph nodes. Questions: 1. What form(s) of pathology may be suspected in this patient? 2. What additional data are necessary for a final conclusion about the form of pathology which the patient suffers from? 3. Taking into account the given data, outline the possible cause and mechanisms of development of this pathology. 4. What approaches can be used to prevent the development of this pathologic process? N 36 On the sixth week of his stay in hospital after extensive myocardial infarction a successfully recovering patient started to suffer from a dull pain in the chest which was aggravated by deep breathing movements, swallow, and changes in the body position. He also presented with fever (body temperature 390C) and a pericardial friction rub. Blood tests showed the presence of

33

eosinophilic leukocytosis and an increased titer of the "antimyocardial" antibodies. A physician made a diagnosis of postmyocardial infarction syndrome (the Dressler's syndrome). Questions: 1. Taking into account the immunogenic nature of the Dressler's syndrome, explain the type and origin of antigens causing this disease. 2. Define the type of a pathologic reaction developed in the patient using the classification system of Gell and Coombs. 3. Describe the mechanism of this pathologic reaction. To what type of immunoglobulin do the "antimyocardial" antibodies belong? 4. Prove (or disprove) the fact that the Dressler's syndrome represents an allergic reaction of the delayed type. N 37 A patient with an extensive full-thickness burn of the thigh got an i.v. infusion of blood plasma. Soon after this, he presented with hyperemia of the face and neck, psychomotor agitation, restlessness, fear of death, an intense throbbing headache, buzzing in the ears, and nausea. A physician suspected the development of allergic reaction and gave the patient an injection of an antihistamine drug. However, the patient's state continued to worsen. He developed choking sensation, acute systemic hypotension (blood pressure 65/45 mm Hg), confusion, pallor, cold sweat, and finally lost his consciousness. Later, seizures and spontaneous urination occurred. Questions: 1. What pathologic process (or reaction) and what type of it developed in the patient after the infusion of blood plasma? Substantiate your answer. 2. Name and characterize the main steps of pathogenesis of this pathologic process. 3. Why did the parenteral injection of the antihistamine drug not improve the patient's condition? What procedures should be performed to prevent this pathologic process (reaction)?

34

4. What factors caused the development of respiratory, hemodynamic and psychoneurologic disorders? Name these factors and characterize mechanisms of their action. 5. What emergency treatment should be employed to help the patient with this pathologic process (reaction)? DISORDERS OF THERMOREGULATION. FEVER. INFECTIOUS PROCESS. Questions to prepare for classes and examination: 1. Definition of fever. Differences between fever and hyperthermia. 2. Types of pyrogens. 3. Pathogenesis of fever. Features of thermoregulation at different stages of fever. 4. The biological significance of fever. 5. Symptoms accompanying fever, and their mechanisms. 6. Types of fever: - based on the extent of temperature rise; - based on the circadian temperature fluctuations. 7. Types of hyperthermia: exertional, nonexertional, drug-induced, neurogenic, endocrinopathic. 8. Principles of treatment of fever. 9. The concept of infectious process. Three stages of infectious process. 10. Entry and colonization of the host. 11. The host defence mechanisms against the entry of infectious agent. 12. Invasion and growth of parasitic organisms in host tissues. 13. The host response. 14. Pathologic components of infectious process. N 38 A 39-year-old patient Z. had been surgically treated 8 days before in the surgical department for perforating duodenal ulcer. In the morning he has presented with progressive weakness, dizziness, nausea; complained of fullness in the epigastrium. Body temperature 39.20C. Ultrasonic studies of the abdomen showed collection of fluid, 4x4x8 cm of size, on the left of the

35

umbilicus. Bacteriologic analysis of the fluid, removed by the needle aspiration, revealed the presence of anaerobic organisms. The patient received the appropriate antibiotic therapy. Three days later the patient's body temperature decreased to 37.40C, and persisted at this level. Questions: 1. What factor caused the increase of the patients body temperature 8 days after the surgery? Substantiate your opinion. 2. What are the specific mechanisms of fever in this case? Use the given data to substantiate your answer. 3. Why was the administration of antibiotics for 3 days ineffective to bring the patient's body temperature to normal values? N 39 A 25-year-old HIV-positive female patient B. was admitted to hospital because of fever (body temperature 38.90C), cough with rales. She complained of pain in her right chest during breathing. Laboratory tests showed leukopenia due to a decrease of lymphocytes and monocytes; bronchial secretions contained large amounts of desquamating epithelium, leukocytes, various strains of bacteria; blood was positive for treponema antigens. Questions: 1. What were the possible sources of pyrogens in this case? Substantiate your answer. 2. How can you explain the development of fever in the patient suffering from leukopenia? 3. Can fever in this case be associated with AIDS? Substantiate your opinion. N 40 A 29-year-old patient M. has been admitted to the hematologic department of hospital two weeks after he had started treatment with cytostatic drugs for chronic myelogenous leukemia. The reason of admission was worsening of condition and increase in the body temperature up to 390C. Examination of the patient revealed a moderate hypochromatic anemia and marked leukopenia. Bacteriologic analysis of

36

his biologic fluids showed the absence of pathogenic strains of microorganisms. Questions: 1. What type of thermoregulation disorder is observed in this patient? Substantiate your answer by arguments for or against the development of fever or hyperthermia (for example, an effect of cytostatic medication)? 2. If you believe that the patient suffers from fever, or if you believe that he has hypothermia, explain its possible causes and mechanisms. 3. What is the main criterion of distinction between fever and hypothermia? N 41 A 18-year-old patient M. felt weakness, dizziness, throbbing headache, chills, and nausea when he returned home from the beach where he had spent 6 hours. Thirty minutes later he developed vomiting, and his body temperature increased up to 390C. He ingested aspirin but it had no much effect. Despite a moderate decrease of body temperature down to 370C his condition continued to worsen, and he called in an ambulance. On the way to hospital he lost his consciousness and was brought to the intensive care unit. Questions: 1. Define the type of a pathologic state developed in the patient. Substantiate your opinion. 2. What are the most likely causes, stages, and mechanisms of this pathologic state? 3. Why did the patient's state continue to worsen despite a decrease in body temperature? 4. What was the cause of loss of consciousness in this patient? HYPOXIA Questions to prepare for classes and examination:

1. Definition of hypoxia and its classification 2. Exogenous hypoxia: causes, types and typical changes of PaO2, PvO2,
Pa-vO2, oxygen blood capacity, PaCO2 and pH in the given type of hypoxia

37

3. Respiratory hypoxia: causes, types and typical changes of PaO2, PvO2,


Pa-vO2, oxygen blood capacity, PaCO2 and pH in the given type of hypoxia

4. Anemic hypoxia: causes, types and typical changes of PaO2, PvO2, PavO2,

oxygen blood capacity, PaCO2 and pH in the given type of hypoxia

5. Circulatory hypoxia: causes, types and typical changes of PaO2, PvO2,


Pa-vO2, oxygen blood capacity, PaCO2 and pH in the given type of hypoxia

6. Histotoxic and substrate types of hypoxia: causes, types and typical


changes of PaO2, PvO2, Pa-vO2, blood oxygencapacity, PaCO2 and pH in the given type of hypoxia

7. Overutilization hypoxia: causes, types and typical changes of PaO2,


PvO2, Pa-vO2, oxygen blood capacity, PaCO2 and pH in the given type of hypoxia

8. Conditions that determine the susceptibility of cells to hypoxia. 9. Signs of acute hypoxia. 10. Adaptive reactions in acute hypoxia. 11. Mechanisms of long-term adaptation to hypoxia.
N 42 In the study of various aspects of pathogenesis of hypoxia and edema a laboratory rat was intravenously injected a high dose of adrenaline. Immediately after the injection paws and ears of the rat turned pale; arterial blood pressure increased from 120/70 mm Hg to 210/175 mm Hg; heart rate and respiration rate increased sharply. The level of PaO2 remained unchanged, but the levels of PvO2 and PaCO2 decreased. Nine minutes after the adrenaline injection despite hyperventilation the rat developed acrocyanosis. The content of gases in the arterial blood remained normal, but a progressive decrease in the PvO2 level was noted. Four minutes later dyspnea and moist rales in the lungs developed; blood pressure decreased sharply; pulse pressure dropped, and heart beats became irregular. At the same time, PaO2 started to decrease, and PaCO2 increased. By the end of the 18th minute the rat developed clonic and

38

tonic seizures, agonal breathing (gasping) and foamy, reddish discharge from the respiratory tract. Soon it died. Questions: 1. Can we state that despite activation of the heart function, increased vascular tone, and high blood pressure the rat suffered from hypoxia immediately after the adrenaline injection? If you think so, substantiate your opinion. If you do not agree, name the time point by which hypoxia has developed. 2. What are the type(s) and pathogenesis of such hypoxia? 3. Did the rat develop edema? If you think so, name the organs where it occurred and the time point after the adrenaline infusion when the edema became evident? 4. If, in your opinion, edema developed in the rat, what forms of pathology in humans may lead to edema with similar localization? N 43 An accident occurred with the aircraft during its flight at the height of 10,000 m. The cabin of the aircraft lost its hermetic state, and in several seconds the barometric pressure inside fell down to atmospheric level of 170 mm Hg at this altitude. The crew failed to make an urgent lowering maneuver, and the aircraft continued the flight at the same height for several minutes. Questions: 1. What pathologic processes developed in the body of passengers during the aircraft accident? Which of them were the most dangerous for their wellbeing and can lead to death? 2. What are the likely causes and mechanisms of these pathologic processes? 3. What types of preexisting pathology could include the highest risk for life in this situation? N 44 A 30-year-old-woman has been brought to hospital by ambulance as emergency case with a preliminary diagnosis of "rupture of a uterine tube

39

with internal bleeding due to ectopic pregnancy". On the way to hospital the patient lost consciousness several times. Examination in hospital showed that the patient was conscious, but confused. She was markedly pale, had a weak, rapid pulse; ausculation revealed muffled heart sounds; blood pressure 80/35 mm Hg. Examination showed the presence of varicous veins in the extremities. The patient told the physician that her condition worsened suddenly when she was eliminating hair on her legs with a depilator. Unexpectedly she felt short of breath, palpitations, and pain in the heart. Later on in hospital, despite inhalation of oxygen, she went on complaining of shortness of breath. Questions: 1. Which of the typical pathologic processes and what type of it could develop in the patient? Substantiate your answer. 2. Name the symptoms caused by this pathologic process. What are the mechanisms, underlying each of the symptoms? 3. Describe the causative factors of this pathologic process. Which of them are most likely to occur? 4. Why was the inhalation of oxygen ineffective to alleviate shortness of breath? N 45 When a physician of the emergency service arrived at the site of a vehicle accident he found that the victim of the accident developed shock associated with an open fracture of the right leg and massive blood loss. The patient also had a closed chest trauma. When bleeding was stopped by application of a tourniquet the physician injected the patient subcutaneously an appropriate dose of morphine (to alleviate pain) and a stimulant of the cardiac function. However, the treatment had no effect, and the symptoms of shock were progressing. Pulse beats were detectable only in the large arteries (60 beats/min), blood pressure decreased to 60/25 mm Hg. On auscultation breathing was depressed over the left chest, and there were no breathing sounds over the right chest. Several minutes later respiration became shallow and ineffective; the patient started to "gasp air". To stimulate

40

breathing the physician injected subcutaneously a stimulant of the respiratory center. However, this treatment also had no effect. Questions: 1. What type (or types) of the emergency condition developed in the patient? 2. What types of hypoxia and in what order developed in the patient? 3. Describe the possible causes and pathogenesis of various types of hypoxia. Explain the typical changes in pH, PaCO2, SaO2, SvO2, blood oxygen capacity, the minute volume of breathing, and the minute volume of circulation during these types of hypoxia. 4. Why was the treatment provided by the physician ineffective? N 46 A 30-year-old patient K. has been treated in the intensive care unit after amputation of uterus which was done under an endotracheal ether narcosis. Suddenly the patient's condition worsened sharply. She developed choking sensation, dyspnea, chills, psychomotor retardation and apathy. Her skin turned pale; acrocyanosis became evident. Physical examination showed an increased respiration rate of 28 per minute, muffled heart sounds, no rales during auscultation of the chest; the pulse was regular 120 beats/min; blood pressure 65/30 mm Hg; concentration of Hb 100 g/L; hematocrit 0.30 (N: 0.36-0.42). The patient was intubated and supplied by oxygen but this measure had no effect. Questions: 1. What pathologic process(es) developed in the patient after the surgery? Substantiate your answer. 2. Can the worsening of the patient's condition be associated with the postsurgery development of hypoxia of: a) respiratory type? b) circulatory type? c) anemic type? d) histotoxic type? What are the possible causes and mechanisms of development of each of the presumed types of hypoxia in this case?

41

3. Are there any signs of the urgent adaptive mechanisms that could compensate hypoxia in this patient? Describe them, if they are present. Why do they appear to be ineffective in this case? N 47 A 50-year-old patient K. has undergone a gastrectomy under narcosis with the use of mechanical ventilation. The surgery was performed urgently due to extensive stomach bleeding which developed suddenly as a result of erosion of the large blood vessel in the decaying tumor. During the treatment of shock and in the course of the surgery the patient received infusions of volume expanders (approximately 1 L) and transfusion of 2.5 L of whole donor blood which had been stored for 2 days. By the 3-d day after the surgery, despite the recovery of the normal blood values of Hb, the patient complained of weakness, headache, dizziness. On examination: the skin of the extremities was cold; severe dyspnea and renal insufficiency were observed; in the conjunctiva a tint of jaundice appeared. Eventually, mechanical ventilation was initiated to treat the patient. Questions: 1. What pathologic process(es) developed in the patient after the surgery? Substantiate your answer. 2. What are the possible causes and mechanisms of development of each of the pathologic processes: a) before the surgery? b) in the course of the surgery and immediately after its completion? c) by the 3-d day after the surgery? 3. What adaptive reactions could be activated in the patient as a result of hypoxia? Why were these adaptive reactions ineffective in this case? N 48 A 60-year-old patient C. had an extensive transmural infarction in the anterior wall of the left ventricle 2 weeks before. During his stay in hospital he woke up at night with choking sensation and called for help. When a nurse came she helped the patient to sit up on the bed with his feet down on the floor. Then she opened the window. The patient felt a little better.

42

However, 10 min later he began complaining of shortness of breath, the need coughing regularly (without secretions), and audible rales in the chest. The nurse gave the patient supplemental oxygen by face mask, and called a physician. Questions: 1. What types of hypoxia did the patient suffer from? Substantiate your answer. 2. In what succession developed these types of hypoxia? 3. How and why did the parameters PaO2, PaCO2, PvO2, PvCO2, SaCO2, SvO2, pH change in the patient's blood? Substantiate your answer. 4. What are the possible mechanisms of immediate adaptation to hypoxia in patient C.? DISORDERS OF CARBOHYDRATE METABOLISM. DIABETES MELLITUS. Questions to prepare for classes and examination: 1. Typical forms of derangement of carbohydrate metabolism. 2. Hypoglycemia: definition, metabolic consequences, adaptive reaction of the endocrine system. 3. Symptoms of hypoglycemia: - derived from autonomic response; - derived from neuroglycopenia. 4. Causes of hypoglycemia. 5. Hypoglycemic encephalopathy and coma. 6. Glycogenoses: classification, manifestations, mechanisms of pathology. 7. Aglycogenoses. 8. Hexosemias. Galactosemia: manifestation and mechanisms of pathology. 9. Pentosemias. Fructose intolerance: manifestations and mechanisms of pathology. 10. Hyperglycemia: causes and mechanisms. 11. Diabetes mellitus: characteristic features, classification, pathogenesis of type I and type II forms.

43

12. Metabolic disorders (carohydrate, protein, lipid) in diabetes mellitus. 13. Complications of diabetes mellitus. Ketoacidotic and hyperosmolar coma.

N 49 A 60-year old patient M. was admitted to hospital in an unconscious state. On examination: the skin appears dry, turgor of the skin and eye ball is reduced; shallow breathing; heart rate 96 beats/min; the tongue is dry; recurrent cramps of the extremities and face muscles. Blood tests show hyperglycemia (600 mg glucose per dl), hyperazotemia, hypernatremia, pH 7.32. The relative who accompanied the patient to hospital told the physician that the patient had been suffering from diabetes mellitus and had ingested small doses of hypoglycemic medicines. During the last month he experienced exacerbation of chronic cholecystitis and colitis, suffered from vomiting and diarrhea which occurred quite often. The patient also felt constantly thirsty and frequently urinated. Questions: 1. Define the pathologic state of the patient on admission. 2. What was the cause of this pathologic state? Describe the main stages of its pathogenesis. 3. Why do patients lose consciousness during the development of this pathologic state and the similar ones? 4. What methods are used to treat patients in this pathologic state? N 50 A 28-year-old male patient visited his physician with complaints of intermittent muscle weakness, dizziness, headache, episodes of poor vision, tremor of hands, irritability and, occasionally, confusion. The paroxysms of this condition occurred more frequently during the last 4 months. The patient attributed his malaise to the psychologic stress that accompanied his professional activity, and also associated it with an acute feeling of hunger. After evaluation of the patient the physician made a diagnosis of neurasthenia and recommended an approprioate treatment. However, the disease went on

44

progressing, and 1.5 months later the patient was brought by ambulance to the emergency room with a diagnosis of coma of unclear etiology. On admission: consciousness is lost; there is evidence of midriasis, muscle cramps; tachycardia, arterial hypotension; irregular breathing; blood glucose level 30 mg/dl. Questions: 1. What form of pathology caused the clinical manifestations presented by the patient during his first visit to the physician? Substantiate your answer. 2. Define the pathologic state of the patient in the emergency room. 3. What are the main steps of pathogenesis of this pathologic state? 4. What other forms of pathology should be kept in mind when you are making a differential diagnosis of this pathologic state? N 51 A 45 year-old patient E. with an excessive body mass suffers from diabetes mellitus. To control his blood glucose he uses hypoglycemic drugs. Two weeks before he was admitted to hospital he had had an episode of excessive alcohol drinking, and soon after that noticed an enhanced feeling of dry mouth; drank a lot of water (up to 8-10 L daily); urinated frequently. He felt general weakness and pain in the legs. On the night before the admission the patient was delirious. When he woke up in the morning he was agitated, restless, and confused. The patient's relatives called in the ambulance. On admission: consciousness is absent; the skin is dry and pale. The results of the blood tests: glucose 1300 mg/dl (N: 80-120 mg/dl), lactic acid 29 mg/dl, pH 7.29. Urine glucose level 4 mg/dl. Questions: 1. What other signs, in addition to the observed in this patient, may be found in a) diabetes mellitus, and b) diabetic coma? Describe these signs and explain their pathogenesis. 2. What are the main stages of the pathogenesis of hyperosmolar coma? 3. What are the main principles of therapy for diabetic hyperosmolar coma?

45

N 52 A female patient, approximately 40-years of age, has been brought to the emergency room in an unconscious state. Witnesses of the incident told the physician that she had suddenly lost her consciousness in the bus on her way back from her summer house in the countryside. A card that was found among her papers indicated that she suffered from diabetes mellitus and took a slow-release form of insulin. On examination: consciousness is absent, corneal and deep tendon reflexes are not observed, the pattern of breathing is unremarkable; blood pressure 80/60 mm Hg; tachycardia is present; the skin appears moist; turgor of the eye balls is increased; general trembling is alternating with episodes of clonic and tonic seizures. The patient was treated with insulin, but no improvement was observed; the patient's condition even worsened: breathing became irregular, blood pressure decreased to 70/50 mm Hg, tachycardia progressed, and the duration of seizures increased. Questions: 1. What pathologic state developed in the patient before and after administration of insulin? Describe causes and mechanisms of this state. 2. What therapeutic procedure would have been appropriate for the patient before and after the administration of insulin? DISORDERS OF LIPID METABOLISM. ATHEROSCLEROSIS. Questions to prepare for classes and examination: 1. Typical forms of disorders of lipid metabolism. 2. The main classes of lipoproteins and their functions. Atherogenic and antiatherogenic lipoproteins. 3. Characteristics of hyperlipidemia. Classification of hyperlipoproteinemias according to Fredrickson. 4. Mechanisms of hyperlipidemia. 5. Characteristics and mechanisms of hypolipidemia. 6. Obesity: definition, diagnostic approaches, pathological significance. Types and pathogenesis of obesity. 7. Features and mechanisms of lypodystrophy.

46

8. Characteristics of lipidoses. 9. Atherosclerosis: description and pathological features. 10. Stages of atherogenesis: - initiation; - formation and evolution of atheroma; - complications. 12. Risk factors of atherosclerosis. N 53 A 15-year-old-boy visited a doctor with complaints of recurrent pain in the region of the heart. The pain was exacerbated at exertion. Examination of the patient showed the presence of small, firm, yellow nodules (xanthomas) over the course of the hand muscles tendons, the presence of corneal lipid arcus. Angiographic examination revealed the coronary artery stenosis. Blood tests results demonstrate: total plasma cholesterol 22 mmol/L (normal values: 3.1-6.4 mmol/L), triglycerides 1.7 mmol/L (normal values: 0.55-1.65 mmol/L), HDL-cholesterol 0.7 mmol/L (normal values: > 0.9 mmol/L), LDL-cholesterol 9 mmol/L (normal values: 3.0-4.5 mmol/L). The patient's parents also have increased plasma levels of cholesterol more than 8 mmol/L. Immunocytochemical analysis of the patient's leukocytes revealed abnormality of the LDL-receptor. Questions: 1. What type of hyperlipidemia is observed in the patient? 2. Is heredity important in the development of this pathology? If it is, what is its type of inheritance. What is the prevalence of this pathology in general population? 3. What mechanisms underlie hyperlipidemia in this patient? 4. Assess the atherogeneicity of the patient's plasma by calculating the index of atherogeneicity. 5. What are the pathologic consequences of hyperlipidemia of this type? What therapeutic approaches may be used to treat this pathology?

47

N 54 During examination of a 5-year-old boy a physician found an enlargement of the liver and spleen (hepatosplenomegaly), swelling of the tonsils which had red-orange color, and an enlargement of inguinal, axillary, and other superficial lymph nodes. Also, clouding of the cornea was observed. Biochemical tests revealed: total cholesterol of the blood plasma 0.1 mmol/L, (normal values: 3.1-6.4 mmol/L), triglycerides 2.3 mmol/L (normal values: 0.55-1.65 mmol/L), HDL fraction almost absent. During electrophoretic separation of the patient's plasma apoproteins A-I and A-II bands were not observed. Questions: 1. Disorders of lipid metabolism of what type is observed in the patient? 2. What mechanisms underlie the development of this disorder? 3. Why is HDL fraction considered antiatherogenic? What factors determine HDL levels in the blood plasma? 4. What role do apoproteins of the A class play in the cholesterol metabolism? N 55 During experimental studies of atherosclerosis one group of rabbits was fed cholesterol added to a standard pelleted food in a dose of 5 g per day. Cholesterol was previously purified to remove contaminating oxides. The second group was treated similarly, but cholesterol was not purified, and the third group received the same diet with cholesterol that was previously subjected to peroxidation by hydrogen peroxide in the presence of ferrous chloride (FeCl2). Six months after the start of the experiment the rabbits were killed by a lethal dose of a narcotic drug; the abdominal portion of the aorta and coronary arteries were isolated, and their internal surface were examined for the presence of lesions. Simultaneously, blood samples were obtained from all rabbits of the experimental groups; plasma was separated from formal particles and used to isolate LDL fraction. In the following in vitro studies samples of the isolated LDL fractions from various groups were added to a media containing cultures of murine peritoneal macrophages, and the extent of intracellular accumulation of cholesterol esters was assessed following incubation for 3 days. LDL fraction of the intact rabbits was used as a control.

48

Questions: 1. Which group of the rabbits is supposed to incur the maximum damage to their arteries due to the excess of cholesterol in the food? 2. Describe the pathways by which LDL can enter peripheral cells, including macrophages. 3. What differences in accumulation of lipids in murine macrophages can be expected with LDL fraction isolated from various experimental animals? Compare LDLs of the control and experimental groups of rabbits. 4. What role do monocytes/macrophages play in atherogenesis? N 56 During his visit to the doctor a 55-year-old male patient complains of anginal pain which recently has increased in frequency and intensity. The patient is the president of a big trading company. During the last months he has experienced great psychologic stress, spent 12-14 hours at his office daily, and slept little; he began smoking more than one pack of cigarettes per day. The patient is a hearty eater, preferring high-calorie fat food. His father died of myocardial infarction. On examination: his height is 173 cm, weight 89 kg, blood pressure 175/100 mm Hg. Blood tests show the following: total plasma cholesterol 6.8 mmol/L (normal values: 3.1-6.4 mmol/L), triglycerides 1.9 mmol/L (normal values 0.55-1.65 mmol/L), HDL cholesterol 0.8 mmol/L (normal values > 0.9 mmol/L), LDL cholesterol 5.3 mmol/L (normal values < 4.9 mmol/L); glucose tolerance is decreased. Questions: 1. What form of pathology is the cause of angina pectoris in this patient? 2. What risk factors of this pathology are observed in the patient? 3. Does the patient have signs of lipid metabolism disorder? If he does, what forms of it? 4. What therapeutic approaches can be used to treat lipid metabolism disorders in this case? WATER DYSBALANCE. EDEMA. Questions to prepare for classes and examination: 1. Typical forms of water dysbalance.

49

2. Hypohydration: causes, types, mechanisms of development, manifestations, consequences, and pronciples of treatment. 3. Overhydration: causes, types, mechanisms of development, manifestations, consequences, and pronciples of treatment. 4. Edema: definition, types, the main factors of development, principles of treatment. 5. Edema during cradiac failure: etiology, pathogenesis, and manifestations. 6. Pulmonary edema: etiology, pathogenesis, manifestations, and consequences. 7. Renal edema: etiology, pathogenesis, manifestations, and consequences. 8. Disorders of sodium, potassium, calcium, manganese, and phosphate metabolism: causes, manifestations, and mechanisms of develpment. N 57 Examination of a 32-year-old patient revealed various signs of pathology, including excessive body mass: his height is 168 cm, and weight 84.5 kg. The patient also has a pasty face, periorbital puffiness, pale skin; he had slow rebound of tissue to its original contour after pressing the feet or shin with the fingertip. The patient told the physician about tightness of a ring and shoes in the evening. An investigation of the cardiovascular system revealed the following: minor arterial hypotension, areas of cardiac dullness are slightly increased; other parameters are unremarkable. The daily urine volume is within the normal range. Questions: 1. What is the possible cause of the patient's excess of body mass? 2. Can we assume that water-ionic balance is deranged in this patient? 3. What type of edema is observed in the patient? 4. What additional data are required to specify the type of edema in this case? N 58 A 42-year-old patient has been admitted to hospital with a diagnosis of uncompensated chronic heart failure due to valvular disease. The patient has normal constitution with paucity of subcutaneous tissue. His height is 165 cm, body weight 81 kg. On examination: the patient needs to sit in bed; he has

50

dyspnea, acrocyanosis, marked lower extremities edema, rales and wheezes during auscultation of the chest. X-ray investigation of the abdominal area shows an accumulation of fluid; the liver is enlarged; stroke volume and cardiac output are decreased; hematocrit 38%; daily urine volume is decreased. Biochemical tests reveal increased plasma activity of renin and increased sodium concentration. Questions: 1. Are there any signs of derangement of water balance in this patient? 2. What type of dyshydria is observed in this case? 3. Is there any association between the accumulation of fluid in the subcutaneous tissue, the abdomen, and the lungs? 4. Explain the pathogenesis of increased blood levels of renin and Na+ in this patient. 5. Explain the pathogenesis of edema in this patient. 6. Explain the role of edema in deterioration of the patient's condition. 7. What therapeutic approaches can be used to treat the edema in this case? N 59 A 22-year-old patient who recovered from severe scarlet fever 2 weeks ago complains of headache, pain in the back, dyspnea, and palpitations. During the last week she has increased her body weight by 11.5 kg. On examination: her face is pale; she has periorbital puffiness, and edema of the shins and feet; the boundaries of the heart dullness are increased; blood pressure is 180/100 mm Hg; daily urine volume is reduced. Urine tests show the presence of erythrocytes and protein. An increased titer of antistreptolysin O antibodies is found in the blood. Questions: 1. Is there evidence of the kidney damage in this patient? What is the possible mechanism of this pathology? 2. What is the cause of hyperhydration in this case: a decrease in water excretion or an increase in water retention? 3. Explain the mechanisms of edema in this patient. N 60

51

A 7-year-old boy developed a progressive swelling of the soft palate with a swallowing difficulty, and then asphyxia after he had drunk mango juice. The mucosal membrane in the swelled area is hyperemic without tenderness; a moderate increase in eosinophils is seen in the blood. The patient's body temperature is normal. His senior sister suffers from attacks of bronchial asthma. Questions: 1. Is edema in this case the result of ordinary inflammation? 2. What is the cause of edema in this patient? 3. Explain the pathogenesis of the given pathology. 4. Does this type of edema lead to life-threatening condition? ACID-BASE DISORDERS Questions to prepare for classes and examination: 1. The concept of acid-base homeostasis. 2. The main buffer systems of the blood. 3. Physiologic mechanisms responsible for the normal acid-base balance: - role of the respiratory system; - role of kidneys.

3. Simple and mixed acid-base disorders.


4. Typical forms of disorders of acid-base balance. 5. Metabolic acidosis: causes and symptoms. 6. The specific examples of metabolic acidosis: - High-anion gap acidosis; - Hyperchloremic metabolic acidosis. 7. Metabolic alkalosis: causes and symptoms. 8. Mechanisms and examples of metabolic alkalosis. 9. Respiratory acidosis: causes and mechanisms. 10. Respiratory alkalosis: causes and mechanisms. 11. Primary and compensatory changes in various types acidosis and alkalosis.

12. Clinical manifestations of alkalosis and acidosis.


13. Laboratory parameters used in diagnostics of acid-base dysbalance.

52

N 61 A group of mountain-climbers in which a physician was included planned to climb a mountain peak of 6200 m. Climbing up to the level of 2900 m was successful, but at this altitude the physician, being an inexperienced climber, felt fatigue, muscle weakness, dizziness, buzzing in the ears (tinnitus). During a one-hour halt the physician collected samples of the capillary blood of his own (B1) and his mate (A1). Then the group continued the ascent. At the height of 5000 m the physician felt acute shortness of breath, severe weakness, heaviness in the body, vision disturbances, and became ataxic. For this reason he decided to stop climbing. The state of the experienced climbers was satisfactory. After the repeated withdrawal of blood samples (A2,B2) all the climbers returned to the camp at the level of 3000 m. The results of laboratory tests of the blood samples are the following: A1 pH pCO2 (mm Hg) pO2 (mm Hg) SB (mmol/L) BE (mmol/L) Questions: 1. Are there any signs of hypoxia and acid-base disorders in the physician or his mate at the given altitudes? If you think there are, define the type of hypoxia and acid-base disorder and explain the cause of its development. Substantiate your answer by the given data. 2. What adaptive mechanisms, if any, were initiated in the body of the physician and the experienced climber when they were ascending? Were there any differencies in the pattern of adaptive mechanisms in the physician and the experienced climber? 3. Why did the adaptive mechanisms fail in the physician? 7.43 32 74 20.5 +2.5 A2 7.35 30 48 18.5 -3.6 B1 7.48 26 70 20.0 +4.5 B2 7.32 42 38 18.5 -5.5

DISORDERS OF THE TISSUE GROWTH. NEOPLASMS. Questions to prepare for classes and examination: 1. Definition of neoplasms. Principles of classification of neoplasms.

53

2. Forms of neoplastic cell abnormality. 3. Characteristic features of benign and malignant tumors. 4. Genetic hypothesis of cancer; classes of genes that incur mutations during neoplastic transformation of a cell. 5. The concept of oncogenes. 6. Mechanisms of transformation of protooncogene to oncogene. 7. Role of antioncogenes in carcinogenesis. 8. Role of genes that regulate DNA repair in carcinogenesis. 9. Chemical carcinogenesis: types of carcinogens; initiation and promotion. 10. Radiation carcinogenesis. 11. Viral carcinogenesis. 12. The concept of tumor progression.

12. Antineoplastic mechanisms. Principles of prevention and treatment of


neoplastic diseases. N 62 A 48-year-old patient N. who has been smoking for 25 years works at the chemical plant where he deals with dyes and organic solvents. He visited his primary care physician with complaints of malaise, weakness, decreased appetite, and persistent cough without secretions. During the previous 3 weeks he noted the presence of blood in the urine, burning sensation in the lower abdomen aggravated by urination. The results of computer tomography scanning, x-ray, and ultrasonography attest against renal or ureter pathology in the patient. Cystoscopy shows proliferation of the bladder's mucosa with erosion of its surface. Biopsy of the lesion in the bladder shows the presence of malignant cells. Questions: 1. What factors could be a potential cause of cancer of the bladder mucosa in this case? Substantiate your answer. 2. What mechanisms of the anticancer defence system could have been deficient in this patient and failed to prevent the development of tumor? 3. Characterize stages of carcinogenesis after the first contact of the bladder mucosa with carcinogen and up to the emergence of the tumor cells.

54

4. Can you exclude the possibility of metastases of the lung cancer in the bladder? Substantiate your opinion. N 63 Seven months after a patient had undergone a surgical removal of the stomach carcinoma and completed a course of chemotherapy he presented with an enlargement of a supraclavial lymph node. Biopsy of the swelled lymph node showed the presence of malignant cells. Some of these cells were similar to the removed tumor cells by their morphology. Questions: 1. Can you attribute the appearance of the tumor cells in the lymph node to the tumor progression phenomenon? Substantiate your answer and characterize the phenomenon of tumor progression (its mechanisms and biological significance). 2. Do you think that the presence of the tumor cells in the lymph node is the result of the primary tumor metastases? - a multicentered tumor growth? - recidivation of the stomach carcinoma? - a new tumor growth? Substantiate your opinion. 3. What factors of the anticancer defence system were ineffective in this case? What are the possible mechanisms of their action? N 64 A 56-year-old patient M., who has been suffering from gastric atrophy and hypoacidity for more than 20 years, complains of fatigability, weakness, pain in the epigastrium, decreased appetite, rapid satiety during meal, nausea, a great loss of body mass during the last 4 months, and a persistent fever. Laboratory tests showed anemia, leukocytosis, hypochlorhydria, and a decreased activity of gastric juice enzymes. Gastroscopy revealed flattening of the mucosal folds in the pyloric area, and the presence of a saucer-like tumor with ulcerative alterations in the center. Questions:

55

1. Why does chronic gastric atrophy promote the development of a gastric tumor? 2. Can we suspect the insufficiency of the antineoplastic mechanisms in this case? If we can, describe these mechanisms. 3. What are the possible causes and mechanisms of fever and anemia in this patient? 4. What are the possible causes of cachexia in this patient? N 65 A 40-year-old patient B. visited his physician with complaints of considerable weakness, dizziness, persistent cough with minor secretions. He had participated in the liquidation of the nuclear power station accident 1.5 years before. B. told the physician that he had been a heavy smoker for 20 years, but 2 years before had quitted smoking. During the last 6 months he repeatedly suffered infectious diseases, such as quinsy, bronchitis, pneumonia. During bronchoscopy a tumor of the main right bronchus was found. Histology of the tumor showed the presence of the squamous epithelium malignant cells. Questions: 1. What factor was the most likely cause of bronchial cancer in this patient? Substantiate your answer. 2. What factors could potentiate the effect of carcinogens in this patient? What are their possible mechanisms of action? 3. What antineoplastic mechanisms should have become activated in this patient: - under the effect of the carcinogens? - in the course of neoplastic transformation of the bronchial epithelial cell? - during the formation of the primary tumor nodule? Why did the antineoplastic mechanisms fail in this case? EMERGENCY STATES Questions to prepare for classes and examination: 1. Shock: characteristics, types, etiology, pathogenesis, manifestations, principles of treatment.

56

2. Coma: characteristics, types, etiology, general pathogenesis, manifestations, principles of treatment. PATHOLOGY OF THE RED BLOOD CELLS SYSTEM. ANEMIA. Questions to prepare for classes and examination: 1. Definition of anemia and its classification. 2. Anemias of blood loss: types, causes, hematological signs.

3. Hemolytic anemias: types, common causes, manifestations,


hematological signs.

4. Hereditary spherocytosis: pathogenesis, manifestations, hematological


signs. 5. Sickle cell disease: pathogenesis, manifestations, hematological signs. 6. Thalassemia syndromes: types, pathogenesis, manifestations, hematological signs.

7. Megaloblastic anemias: types, causes, pathogenesis, manifestations,


hematological signs. 8. Iron deficiency anemia: causes, pathogenesis, manifestations, hematological signs. 9. Aplastic anemia: causes and pathogenesis, hematological signs. 10. Erythrocytosis: causes and mechanisms N 66 A 5-year-old female patient Zulfia was admitted to hospital with a preliminary diagnosis of pneumonia. On admission: the skin and conjunctiva appeared jaundiced, the body temperature was 390C, the spleen and liver were enlarged, the urine was dark; the patient complained of an acute pain in the back, in the right anterior abdomen, in the leg's muscles; X-ray investigation of the chest revealed signs of bronchopneumonia. Blood tests showed the following: Hb 69 g/L, erythrocytes 2.3*1012/L, MCH (mean corpuscular hemoglobin) 30 pg; the presence of poikilocytosis and anisocytosis of erythrocytes. The urine test revealed the presence of Hb. During electrophoretic investigation of the patient's Hb the HbS type was found in addition to the HbA. Questions:

57

1. What forms of pathology are observed in the given patient? Substantiate your answer. 2. Explain the cause-effect and chronologic relationship between the observed forms of pathology. 3. What is the cause of the primary pathology? Which of the given symptoms suggest the presence of this pathology? What are the mechanisms of their development? 4. Explain the mechanisms of the following symptoms: - acute pains in various organs and tissues; - jaundice; - poikilocytosis; - anisocytosis. 5. What measures would you recommend to the girl's parents to prevent the recurrence of the described symptoms? N 67 A 60-year-old female patient K. was admitted to hospital with complaints of weakness, persistent headaches, dizziness, "shakiness", numbness in the hands and legs, minor dyspnea, decreased appetite, tenderness in the oral cavity during salty or acidic meal, a burning sensation in the tongue. The patient is a vegetarian. Two weaks before she visited her physician because of anorexia, pains and heaviness in the epigastium, and diarrhea. Gastric juice investigation showed severe hypochlorhydria. In the blood test there were hyperchromatic anemia, macrocytosis and poikilocytosis of erythrocytes, leukopenia, hypersegmented neutrophil nuclei, and a low platelet count. On examination in hospital: the patient has marked paleness of the skin and mucosal membranes, the conjunctiva appears jaundiced, blood pressure 110/75 mm Hg, the surface of the tongue is bright red, smooth, and shiny ("the polished tongue"); there are signs of stomatitis. The patient has unsteady gait. Concentration of folic acid in the blood serum is normal. Questions: 1. What forms of pathology are observed in the given patient?

58

Substantiate your answer. Explain the likely chronologic and pathogenetic relations between these forms of pathology. 2. Which of the given pathologic processes is primary? Substantiate your answer. 3. Characterize anemia developed in this patient by the following criteria: - type of hemopoiesis; - erythrocyte count in the peripheral blood; - mean corpuscular hemoglobin; - the size and shape of erythrocytes; - the presence of inclusions in the erythrocyte's cytoplasm. 4. Explain the origin of the following symptoms: - shaky gait; - numbness of hands and legs; - stomatitis; - "the polished tongue". 5. Which of the symptoms presented by the patient refer to hypoxia? Prove or disprove the presumption about the presence of hypoxia of the circulatory, respiratory, anemic, or cytotoxic type in this patient? N 68 A 26-year-old pregnant woman visited her physician with complaints of intense muscle weakness, fatigability, swallowing difficulty, choking during meal, and a burning sensation in the tongue. These symptoms developed in the third trimester of pregnancy. On examination: the patient has marked paleness of the skin and mucosal membranes, signs of stomatitis and hypotrophy of the tongue papillae, flattened, spoon-shaped nails (koilonychia). The results of blood tests: Hb 60 g/L, erythrocytes 2.8*1012/L, reticulocytes 0.2%, poikilocytosis and microcytosis of erythrocytes, leukocytes 3.6*109/L, platelets 200*109/L; total serum iron-binding capacity is increased; hypoferritinemia and increased level of free protoporphyrin are found. Questions: 1. What form of pathology developed in the patient? Explain your answer. 2. What is the cause of this pathology? What other factors can cause it?

59

3. What mechanisms underlie each of the symptoms developed in the patient? 4. What approaches may be used to treat this form of pathology? N 69 A patient has been transported to hospital 40 min after he got a gunshot wound. On admission: the patient is conscious, but confused; his skin is pale, breathing is rapid and shallow; he has a running pulse; blood pressure is 65/35 mm Hg. The results of blood tests: Hb 142 g/L, erythrocytes 4.5*1012/L, MCH 31.5 pg. Since the patient had signs of internal bleeding and accumulation of the blood in the peritoneal cavity, he was operated and underwent ligation of the injured mesenteric artery branch. On the third day of treatment in hospital the patient's blood test data were the following: Hb 68 g/L, erythrocytes 2.8*1012/L, reticulocytes 10%. His blood pressure reached 115/70 mm Hg. Questions: 1. What pathologic state was observed in the patient on admission? Substantiate your answer. 2. Evaluate changes in the blood test data on the 1st and the 4th days after the injury, and make a conclusion. Explain the differences in the results of these tests? 3. Was the pathologic state observed in the patient on admission still present on the 4th day of treatment in hospital? If it was, substantiate your opinion. If it was not, define the pathologic state developed on the 4th day after the patient's injury. 4. What approaches would you use to treat the patient on the 4th day after the injury? N 70 A 42-year-old female patient D. visited her general practitioner with complaints of headache, insomnia, weakness, fatigability, episodes of uterine bleeding not associated with menses (dysmenorrhea). The latter occurred about 6 months ago. The results of blood tests: Hb 95 g/L, erythrocytes

60

3.3*1012/L, reticulocytes 8.5%, microcytosis of erythrocytes, leukocytes count 4.9*109/L; platelets 240*109/L. Due to family problems the patient failed further investigation and treatment. Meanwhile, episodes of the uterine bleeding recurred, and her condition continued to deteriorate. Three months later she was admitted to hospital with a diagnosis of uterine myoma. Blood tests made on admission showed: Hb 45 g/L, erythrocytes 2.2*1012/L; erythrocytes are hypochromic with signs of anisocytosis; microcytes are abundant; reticulocytes constitute 0.05%; leukocytes count is 1.9*109/L, and platelets count is 110*109/L. Questions: 1. What forms of blood cells pathology developed in the patient during each of the follow-up periods (outpatient's and inpatient's)? Substantiate your answer. 2. What additional tests are required to specify the pathogenesis of the patient's disease developed by the moment of admission? 3. Explain the reason why the results of the first blood test differ from that of the second one. What is the prognostic value of the observed differences? 4. What approaches may be used to treat this form of pathology? N 71 A 36-year-old-patient underwent an ultrasonic evaluation of the abdominal area. The results of it suggested an enlargement of the right kidney. The later performed computer scanning revealed a mass that had the size of 1.2x1.5x1.5 cm and localized to the superior pole of the right kidney. For this reason the patient was hospitalized. The patient's blood test showed: Hb 180 g/L, erythrocytes count 7.5*1012/L, reticulocytes 10%, leukocytes 4.0*109/L, platelets 250*109/L, hematocrit 0.61; erythropoietin concentration is 20% above the normal values; blood pressure 150/90 mm Hg. A histologic investigation of cells, obtained by needle aspiration of the right kidney's mass, confirmed the presence of hypernephroma. The patient underwent surgical removal of the tumor. Three weeks later his condition improved; the results of his clinical tests were normal. Questions: 1. What form of pathology developed in the patient in connection with hypernephroma? Substantiate your opinion using the given data.

61

2. What is the specific cause of this pathology in the patient? 3. Describe the mechanisms of this pathology and of the symptoms presented by the patient? 4. What other factors may cause the development of this pathology? PATHOLOGY OF THE WHITE BLOOD CELL SYSTEM. LEUKOCYTOSIS, LEUKOPENIA. Questions to prepare for classes and examination: 1. The main stages of leukopoiesis and its regulation by humoral factors. 2. Classification of disorders of white blood cells. 3. Leukopenia: definition and causes. 4. General mechanisms of leukopenia. 5. Causes and mechanisms of decreased leukocytes production. 6. Causes and mechanisms of increased peripheral destruction of leukocytes. 7. Signs and consequenses of neutropenia. 8. Leukocytosis: causes and mechanisms. 9. Causes of increased production in the bone marrow, mobilization, and defective margination of neutrophils. 10. Types of leukocytosis; patterns of shift to the left. 11. The biological significance of leukocytosis. 12. Types and mechanisms of abnormal neutrophil function.

13. Leukemoid reactions: typical features, causes, mechanisms of


development, physiologic significance.

14. Changes in the complete blood test with differential. Absolute and
relative shifts in leukocytes count. N 72 A 38-year-old male industrial worker visited his general practitioner. The patient was a victim of fire at his working place. During the accident he had received epidermal and partial thickness burns of the right side of the body (about 10% of the total body surface area). His condition was fair; body temperature 37.80C. On the fifth day the patient's condition deteriorated due to an intercurrent infection of the burn wound. He developed signs of marked purulent, exudative inflammation of the derma and subcutaneous tissue; his

62

body temperature elevated to 40.20C. Blood tests were taken twice at different time points. The following results were obtained: Analysis "A" Hemoglobin Erythrocytes Reticulocytes Leukocytes: Neutrophils Myelocyte Metamyelocyte Band Segmented Eosinophils Basophils Lymphocytes Monocytes Questions: 1. Evaluate the results of the tests "A" and "B" and make a conclusion to each of them. What is the difference between these two analyses? 2. One of these tests was made on the same day and the other on the fifth day after the trauma. Define the time when each of the tests were made. Substantiate your opinion. 3. What are the causes and mechanisms of the observed changes in the patient's blood? N 73 A 26-year-old female patient N. was treated for obsessive-compulsive disorder and hyperthyroidism in the out-of-town hospital. She was prescribed sedative and antithyroid drugs. Several days after the onset of treatment she visited her physician with complaints of malaise, sore throat, burning sensation in the mouth, and dizziness. On examination: signs of follicular tonsillitis and stomatitis. Blood test showed: Hb 120 g/L, erythrocytes 0 0 6% 68% 2% 0 21% 3% 1% 4% 14% 60% 0 0 16% 5% 125 g/L 4.5 *1012/L 0.5% 10.5 * 109/L Analysis "B" 125 g/L 4.7*1012/L 0.8% 16 * 109/L

63

4.2*1012/L, reticulocytes 0.5%, leukocytes 0.5*109/L, eosinophils and basophils are not found, segmented neutrophils 20%, other neutrophils or precursors - not found; lymphocytes 75%, monocytes 5%. During a bone marrow examination a great number of immature leukocytes without abnormal features were observed. Questions: 1. What forms of pathology developed in the patient? Substantiate your answer. 2. What are the causes of these forms of pathology? 3. What are the causes and mechanisms of changes in the peripheral blood and bone marrow in this case? 4. What mechanisms underlie the symptoms observed in the patient? 5. What approaches can be used to treat the patient? N 74 Complete blood test with differential in a 80-year-old patient who had recurrent episodes of bleeding associated with unresectable gastric carcinoma showed the following: Hb Erythrocytes MCH Reticulocytes Leukocytes (total) promyelocytes myelocytes metamyelocytes band neutrophils segmented neutrophils basophils eosinophils lymphocytes monocytes 85 g/L 3.4*1012/L to be calculated 0.1% 53*109 2.5% 8.0% 11.0% 18.0% 51.0% 0 0.5% 5.0% 4.0%.

64

Neutrophils have toxic granules and increased alkaline phosphatase activity; no Philadelphia chromosome is found in the leukocyte precursors during a bone marrow examination. Questions: 1. Analyze the blood test data and make a conclusion. 2. What are the causes and mechanisms of changes in the red blood and white blood cells? What other forms of blood pathology should the given disorder be distinguished from? Substantiate your answer. 3. Prove or disprove an assumption that changes in the blood test can be the result of metastases of gastric carcinoma in the bone marrow. 4. Is it possible that the patient developed leukemia under the effect of the same carcinogen that had caused gastric carcinoma. N 75 A 29-year-old male patient has signs of moderate radiation injury incurred during accident at the nuclear power station. His blood test taken the next day after the accident shows: Leukocytes Neutrophils: band segmented Eosinophils Basophils Lymphocytes Monocytes Leukocytes Neutrophils: Band Segmented Eosinophils Basophils Lymphocytes Monocytes 2% 53% 0 0 40% 5%. 7% 79% 2% 0 7% 5% 1.0*109/L 11*109/L

The patients blood test made on the 14th day after the accident shows:

65

Questions: 1. Make a conclusion on the results of the two blood tests. 2. What are the mechanisms of changes in white blood cells (for each type of leukocytes) at the beginning and later stages of the radiation disease. 3. Compare the time-course of changes in granulocyte and lymphocyte during the progression of the radiation disease. 4. What type of leukocytes (granulocytes or lymphocytes) is more susceptible to ionizing radiation? N 76 A 48-year-old patient D. has been admitted to hospital with signs of severe tonsillitis and stomatitis. She has a history of chronic headaches and prolonged (many months) intake of amidopyrin at average therapeutic doses. Complete blood count test shows the following: leukocytes 1.8*109/L, neutrophils: segmented 10%, band - not detected; eosinophils - not detected, basophils not detected, lymphocytes 75%, monocytes 15%; red blood cells and plateletes are within the normal range. Questions: 1. Make a conclusion on the results of the blood test. 2. What is the cause-effect association between the clinical symptoms and the blood test findings in this case? Substantiate your answer. 3. What is the most likely cause of changes observed in the peripheral blood of the given patient? Describe the possible mechanism of these changes. 4. What additional tests would be helpful to specify the mechanism of the hematologic changes in this patient? N 77 Blood test has been performed in a patient with unresectable carcinoma of the stomach. It shows the following results: Hb Erythrocytes Leucocytes promyelocytes myelocytes 95 g/L 3.4*1012/L 53*109/L 2.5% 8.0%

66

metamyelocytes band neutrophils segmented neutrophils Basophils Eosinophils Lymphocytes Monocytes Questions:

11.0% 18.0% 51.0% 0 0.5% 5.0% 4.0%

1. Characterize the presented hematologic data. 2. What form of pathology the observed abnormality of the white blood should be distinguished from? What methods can be used for this? 3. Make a conclusion on this blood test. 4. What are the likely causes of the observed changes in the peripheral blood? MALIGNANCIES OF HEMATOPOIETIC CELLS. LEUKEMIA Questions to prepare for classes and examination: 1. Typical features, types, etiology, and general pathogenesis of hematopietic cell tumors. 2. Leukemia: general characteristics, causes, classification and mechanisms of development. 3. Characteristics of the leukemic reaction of the peripheral blood; types of leukemic reaction. 4. Lymphoid tumors: pathological basis, clinical manifestations, and peripheral blood picture in acute lymphoblastic leukemia and chronic lymphocytic leukemia. 5. Myeloid tumors: classification, pathological basis, clinical manifestations, and peripheral blood picture in acute myeloblastic leukemia and chronic myelogenous leukemia. 6. Phenomenon of tumor progression in hematopoietic cell tumors. N 78 A 62-year-old female patient has been hospitalized with the diagnosis of "exacerbation of chronic bronchopneumonia". The patient complains of

67

weakness, dyspnea, fever. On examination: the patient is pale; she has a tint of jaundice in the conjunctiva, and a moderate enlargement of the regional lymph nodes, spleen, and liver. An X-ray examination reveals the signs of right-sided bronchopneumonia, a significant enlargement of the hilar lymph nodes. Complete blood counts with differential shows: Hb Erythrocytes MCH Reticulocytes Platelets Leukocytes (total) myelocytes metamyelocytes band neutrophils segmented neutrophils basophils eosinophils lymphoblasts lymphocytes monocytes 80 g/L 3.0*1012/L to be calculated 0.6% 105*109/L 36*109/L 0 0 0.5% 6.0% 0 0 1.5% 89.5% 2.5%.

In the blood an increased content of unconjugated bilirubin, and the presence of anti-RBC and antiplatelet antibodies are found. Questions: 1. What forms of pathology developed in the patient? 2. Is there association between these forms of pathology? Substantiate your answer. 3. What are the causes and mechanisms of symptoms presented by the patient: - weakness and fatigability? - dyspnea? - bleeding diathesis? - fever? - jaundice in the conjunctiva? - enlargement of the liver, spleen, and regional lymph nodes?

68

- increased blood content of unconjugated bilirubin? N 79 A 52-year-old male patient L. suffers from leukemia, and takes the appropriate treatment. He has visited his physician with complaints of exacerbation of his disease. Recently he has begun to experience progressive weakness, dyspnea, sleep problems, gum bleeding, and fever. He marks the emergence of bruises on the forearms and shins. He also complains of pain in the heart. On examination: paleness of the skin and mucosal membranes, signs of multiple subcutaneous hemorrhages of various age, pustular rash in various locations of the body; the subcutaneous lymph nodes, the liver, and the spleen are enlarged. CBC (complete blood count) with differential has been compared with the test performed two months ago. The results of these tests are the following: A Hb Erythrocytes MCH Reticulocytes Platelets Leukocytes Myeloblasts Promyelocytes Myelocytes Metamyelocytes Band neutrophils Eosinophils Basophils Lymphocytes Monocytes Questions: 86 g/L 3.3*1012/L 0.78 0 100*10 /L 36*109/L 65% 10% 0 6% 3.5% 2% 0 2.5% 0
9

B 125 g/L 4.1*1012/L 0.91 0.4% 260*109/L 12*109/L 3% 6% 10% 15.5% 12.5% 36% 5% 3.5% 6% 2.5%

___________________________________________

Segmented neutrophils 11%

69

1. Make an analysis of the results of the tests A and B. Which of the tests has been done recently and which one some time ago? Substantiate your answer. 2. Define the clinical significance and the pattern of hematologic shift in the patient's blood. 3. What are the causes of the hematologic shift and acute deterioration of the patient's condition? 4. What mechanisms underlie the development of hemorrhages, fever, enlargement of the spleen, the liver, and the lymph nodes; pain in the heart? PATHOPHYSIOLOGY OF HEMOSTASIS Questions to prepare for classes and examination: 1. The system of hemostasis: components and their function. 2. Typical forms of pathology of the hemostatic system. 3. Thrombotic syndrome: its main causes, mechanisms of development, manifestations and consequences. 4. Hemorrhagic syndrome: causes, mechanisms of development, manifestations and consequences. 5. Causes and mechanisms of thrombocytopenia and thrombocytopathia. 6. Hemophylias and von Willebrand disease. 7. Thrombohemoarrhagic conditions. The syndrome of disseminated intravascular coagulation: stages, mechanisms of development, manifestations and consequences. 8. Laboratory parameters used to characterize hemostatic disorders: bleeding time, prothrombin time, partial thromboplastin time. N 80 A 62-year-old patient was admitted to hospital with a diagnosis of "myocardial infarction of the left ventricle and ischemic stroke associated with systemic atherosclerosis". On the next day his state was aggravated by development of thromboembolism of the left popleteal artery and acute renal insufficiency with signs of uremia. On the third day of his stay in the hospital the patient's condition was complicated by the emergence of multiple subcutaneous hemorrhages and gastrointestinal bleeding. Blood tests showed marked thrombocytopenia, a considerable decrease in fibrinogen and

70

prothrombin levels, increased concentration of heparin and fibrin split products, and the enhancement of the blood fibrinolytic activity. Questions: 1. What forms of pathology are obseved in this patient? Substantiate your answer. 2. Which of these forms of pathology caused the development of the myocardial infarction and stroke? - renal insufficiency? - thromboembolism of the popleteal artery? - hemorrhages and bleeding? 3. How can you explain the development of the formaly opposite forms of pathology in this case, such as myocardial infarction and stroke on the one hand, and hypocoagulative disorders petechiae and gastrointestinal bleeding on the other hand? 4. Can we consider these various forms of pathology as parts of a common disorder, or do they occur independantlly in this patient? Substantiate your answer. N 81 Parents of a 3-year-old boy noticed that he often had inflammation of his knees and elbows. They told the doctor that their boy often has extensive nasal bleedings and develops large bruises after he falls down or hurts himself. During teething he had transient gums bleeding. On examination: physical and mental development of the boy is commensurate with his chronological age; knees and elbows have signs of inflammation (swelled, hyperemic, and tender). The results of the complete blood count test are unremarkable; coagulogram shows normal capillary bleeding time and prothrombin time, but considerably increased partial thromboplastin time. Questions: 1. What pathology can be suspected in the patient? Substantiate your answer. 2. What tests should be done to make an accurate diagnosis? 3. What is the likely cause of this pathology? 4. Describe the variants of this pathology and their mechanisms.

71

5. Explain mechanisms underlying each of the symptoms observed in the patient. N 82 A 50-year-old female patient K. with decompensated insufficiency of the aortic valve underwent an implantation of the prosthetic valve. During the surgery extracorporeal circulation was employed. Three weeks later the patient's condition deteriorated. She developed marked dyspnea, high fever; complained of a pain in the heart. For this reason one more operation was performed to replace the current prosthesis and implant a new one. Examination of the removed valve prosthesis showed thrombotic vegetations with microbial colonies on its surface. A day later the patient died. At autopsy: multiple focal hemorrhages in the brain and other organs, signs of systemic vasculitis, mural thrombi; intravascular conglomerates of formal particles and fibrin sheath were observed. Questions: 1. What disturbances of the blood clotting system are present in this case? What syndrome may be suspected on the ground of cinical signs? 2. What pathologic processes or medical interventions could have caused the given disturbances of the clotting system? 3. What are the possible mechanisms of the clotting system disorders in this patient, and mechanisms of the observed symptoms? N 83 A 38-year-old patient A., an alcohol addict, was admitted to hospital with gastrointestinal bleeding and hematuria. On examination: the patient had a tint of jaundice in the skin and mucosal membranes, the hard and nodular liver which extended 2 cm below the right costal margin. The results of complete blood count test were unremarkable. Blood biochemistry tests revealed an increased concentration of conjugated and unconjugated bilirubine, decreased levels of clotting factors II, VII, IX, X, increased plasma aspartate transaminase (AST) activity, increased prothrombin time and partial thromboplastin time. Suspecting a vitamin K-dependent hypocoagulative

72

state, the physician prescribed the patient a vitamin K containing medicine, but this treatment had no positive effect. Questions: 1. What forms of pathology developed in the patient? Substantiate your answer. 2. Which of them are primary ("triggering"), and which are secondary? What is the cause-effect relationship between these forms of pathology? 3. What are the causes and mechanisms of coagulopathy in this patient? What role does chronic alcohol intoxication play in it? Substantiate your answer. 4. Why was the vitamin K treatment ineffective in this case? What treatment would be appropriate to this patient? N 84 A 65-year-old patient N. with unresectable gastric carcinoma was admitted to hospital with signs of significant worsening of his condition. He complained of severe dyspnea after a minor physical exertion, pain on the right lateral side of the chest, non-productive cough, and fever. Six hours after the admission he presented with signs of angina pectoris that were resistant to nitroglycerine, and also, cerebral vascular insufficiency manifested by rightsided hemiparesis. The patient was delivered to the intensive care room. When he was being transferred from the gurney to the bed he developed emesis with blood. Considering the risk of myocardial infarction and aggravation of the cerebral vascular problems, the physician planned to treat the patient with anticoagulative and fibrinolytic drugs. Before the start of the antithrombotic therapy immediate blood tests were performed, and the following results were obtained: Hb 105 g/L, erythrocytes 3.5*1012/L, leukocytes 12*109/L, platelets 40*109/L; decreased concentration of fibrinogen; an increased prothrombin time and partial thromboplastin time; a 50% decrease in antithrombin III blood level. Taking into account the blood test data the physician changed his mind about the treatment schedule. Questions:

73

1. What forms of pathology are observed in this patient? What is the causeeffect relationship between them? Substantiate your answer. 2. The results of the blood tests suggest the presence of coagulopathy. Define the type of this coagulopathy and explain its cause, mechanism, and clinical course. Substantiate your opinion by the clinical and laboratory data. 3. Why did the physician quit the antithrombotic therapy? What can you recommend as an alternative treatment in this case? N 85 Patient N. has been admitted to hospital with complaints of intense retrosternal pain. The use of nitroglycerin orally did not relieve it. Moreover the patient's condition worsened, and he was transported to the intensive care unit. Taking into account the risk of myocardial infarction the physician decided to administer anticoagulant and fibrinolytic drugs to the patient. Before the start of the anticoagulant therapy an immediate blood test was performed. The blood test data: Hb 105 g/L, erythrocytes 3.5*1012/L, leukocytes 12*109/L, platelets 80*109/L, hypofibrinogenemia, an increase in prothrombin time and partial thromboplastin time, antithrombin III concentration is decreased by 50% below normal values. Having considered these data the physician withdrew from administration of anticoagulants and fibrinolytics. Questions: 1. What type of hemostasis disorder developed in the patinet? 2. Is this hemostasis disorder associated with cellular mechanisms? or blood plasma factors? or both? 3. Why did the physician change his plan with respect to anticoagulant treatment? N 86 A 20-year-old patient A. and a 25-year-old patient B. have come to see their physician with similar complaints of easy formation of bruises, and prolonged bleeding after laceration. Patient A. noted often gums bleeding during cleaning the teeth, and both of them mentioned that after tooth extraction the wound was oozing more than a day. Patient B. also complained

74

of swelling and inflammation of his knee joints, and pain in the knees during walking. Patient A. informed the physician that his mother has the same symptoms as he does, while the patient B.'s parents have no signs of hemostatic disorders. On examination: A. shows increased capillary bleeding time; in B. this parameter is within the normal range. Patient B. has a significantly increased partial thromboplastin time (PTT), but in A. this parameter is normal. In both patients prothrombin time, platelet count, prothrombin and fibrinogen content in the blood are within normal limits. Questions: 1. What components of the hemostatic system are abnormal in these patients? Substantiate your answer. 2. What disorders can be suspected in patients A. and B.? 3. What additional laboratory data are required to confirm the preliminary diagnoses, and what are their likely results? 4. What role does heredity play in manifestation of these diseases? 5. Describe the mechanisms of symptoms presented by these patients. CORONARY INSUFFICIENCY. MYOCARDIAL INFARCTION. ARRHYTHMIAS. Questions to prepare for classes and examination: 1. Definition of coronary insufficiency. The main cuases of coronary insufficiency. 2. Mechanisms of injury to the myocardial cells during ischemia and reperfusion: role of Ca2+, oxygen free radicals, and leukocytes. 3. Adaptive mechanisms developing during acute and chronic coronary insufficiency. 4. Biochemical, mechanical and electrical changes elicited in the myocardium during ischemia. Signs of reversible and irreversible ischemic injury. 5. The syndromes of coronary insufficiency: angina pectoris, myocardial infarction, chronic ischemic heart disease, sudden coronary death. 6. Methods of diagnostics of myocardial infarction. 7. Cardiac arrhythmias: types, mechanisms of development, hemodynamic consequences.

75

8. Reentry arrhythmias: conditions required for reentry, types and mechanisms of reentry, examples of arrhythmias based on reentry. N 87 A 46-year-old patient was admitted to the intensive care unit with complaints of intense retrosternal pain and squeezing in the chest which lasted 1.5 hours. During the last week the patient worked hard, slept little, and smoked more than usually; he also drank too much coffee and tea. The patient reported that before the current episode he had no health problems and went in for sports. On examination: the patient's condition is guarded; he is pale, and has evidence of acrocyanosis. Auscultation shows an unremarkable pattern of breathing at the rate of 28 per minute, no evidence of rales; the heart sounds are muffled and arrhythmic; blood pressure is 100/70 mm Hg. The ECG reveals recurrent atrial fibrillation at the rate of 360 per minute, right bundle branch block; S-T segment elevation in leads I, Vl, V1-V4. Blood test shows: leukocytes 9.2*109/L, other parameters are unremarkable. Questions: 1. What forms of pathology developed in the patient? Substantiate your answer. 2. What is the cause-effect association between these forms of pathology? 3. What is the most likely cause of the pathologic state accompanied by the retrosternal pain? 4. What metabolic and electrophysiologic changes in the myocardium promoted the development of atrial fibrillation? 5. What was the immediate cause of the patient's pathologic state? N 88 A 56-year-old male patient B. was brought by the ambulance to the intensive care unit. He complained of intense retrosternal pain and a pressure sensation which lasted approximately 3 hours, and was not relieved by repeated doses of nitroglycerine. On examination: the patient's condition is fair; he has a hypersthenic constitution, a hyperemic face; there are signs of acrocyanosis. Auscultation reveals the presence of rales; breathing rate is 20 per minute; the heart sounds are muffled; heart rhythm is regular, 80 beats

76

per minute; blood pressure is 90/65 mm Hg. ECG shows sinus rhythm, elevation of the S-T segment in leads II, III, Vf. Cardiac index measured 1 month before was 3.3 L/min/m2 (normal values: 2.7-3.0 L/min/m2). Blood test shows the following: Hb 196 g/L, erythrocytes 6.5*1012/L, leukocytes 12.0*109/L, platelets 450*109/L, ESR 15 mm/h; blood serum activity of the creatine kinase MB fraction 70 IU/L (normal values: 0-12 IU/L); prothrombin index 120% (normal values: 95-105%). Bone marrow examination suggests the presence of polycythemia vera. Questions: 1. What forms of pathology developed in this patient? 2. What are the likely causes of each of these forms of pathology? Explain the possible association between these forms. 3. What mechanisms contribute to heart symptoms in this patient? 4. Is there association between the heart pathology and polycythemia vera in this case? N 89 A 58-year-old man, who suffered from ischemic heart disease, took several tablets of nitroglycerine to alleviate prolonged (more than 30 minutes) retrosternal pain which radiated to the epigastrial area. However, the pain persisted and became burning. Soon after this, the patient started to feel weakness, dizziness, and nausea. The ambulance physician, called in by the patient's relatives, suspected a myocardial infarction and gave the patient an injection of a combination of drugs including an anesthetic, an antiarrhythmic, and an anticoagulant. Two hours after the onset of the heart attack, despite vigorous treatment, the patient developed arterial hypotension (blood pressure 70/55 mm Hg), low rate of urinary excretion (25% of the normal value per hour), acrocyanosis, dyspnea, rales during auscultation. Blood test showed the following: erythrocyte count 6.0*1012/L, leukocytes 11.9*109/L. Questions: 1. Define the pathologic state developed in the patient: - before the arrival of the ambulance? - 2 hours after the onset of the heart attack? Substantiate your answer.

77

2. Which factors caused the heart pathology in each of these pathologic states? 3. Both of these pathologic states can cause cardiac insufficiency. What are the main mechanisms of its development in each of the cases? 4. What actions could be undertaken to prevent cardiac insufficiency and its consequences in these pathologic states? N 90 A 52-year-old patient S. has been admitted to the hospital's Cardiology Department. He had a history of syncope preceded by a prolonged period of recurrent episodes of palpitations. They were usually accompanied by weakness, dizziness, and choking sensation. The day before the admission the patient incurred a severe psychologic stress related to death of his close relative, who was heavy smoker, and suffered from ischemic heart disease. On examination: blood tests are unremarkable. Holter ECG monitoring shows 11 episodes of arrhythmia lasting from 20 to 60 sec. During these episodes P wave is poorly exhibited and occasionally superimposes on QRS complex. The frequency of P wave is usually about 70 per minute; QRS complexes are regular, and occur at rates of 190 per min; they are often distorted and look similar to ventricular extrasystoles. Episodes of arrhythmia are accompanied by a decrease in systemic blood pressure. Questions: 1. Define the form of heart pathology developed in the patient. Substantiate your answer. 2. What are the possible causes of this pathology? 3. What electrophysiologic mechanisms underlie the ECG changes observed in the patient? What metabolic disorders in the myocardium could promote the observed ECG changes? 4. Is there a risk of sudden death during the described episode of heart dysfunction? If there is, what could be the immediate cause of death? N 91 A 62-year-old patient K. experienced myocardial infarction in the posterosuperior area of the left ventricle and interventricular septum 5 days

78

ago. Suddenly, he felt weakness, dizziness, nausea, turned pale, and lost consciousness. ECG shows regular atrial waves at rates of 109 per min, and regular ventricular rhythm with a frequency of 42 per min; there is no association between P wave and QRS complexes; systemic blood pressure is 65/50 mm Hg. Questions: 1. Define the form of cardiac pathology developed in the patient? Substantiate your answer using the clinical data and ECG. 2. What electrophysiologic mechanism underlies this form of pathology? 3. What metabolic changes and in what area of the heart can produce the electrophysiologic disturbancies developed in the patient? Substantiate your answer. 4. Describe the principles of adequate treatment in this case. N 92 A 58-year-old patient M. was hospitalized 3 days ago with a diagnosis of myocardial infarction in the middle one third of the anterior wall of the left ventricle. Suddenly, he felt weakness, dizziness, discomfort in the heart. ECG registered during the attack revealed regular atrial "sawtooth" waves at rates of 350 per min; each two atrial waves were accompanied by a QRS complex of a normal shape. Systemic blood pressure was 90/50 mm Hg. The physician gave the patient an intravenous infusion of beta-adrenoblockers and calcium antagonists. Twenty minutes later the patient's condition improved. His blood pressure increased up to 110/75 mm Hg, but changes in the ECG were still present, though the rate of atrial waves and QRS complexes decreased. The next day after meal a similar episode of cardiac disorder occurred. In this case pharmacologic treatment had no effect. The patient was urgently transported to the intensive care unit. Questions: 1. Define the type of cardiac disorder developed in the patient. Substantiate your answer using the clinical data and ECG. 2. What electrophysiologic disturbances and in what area of the heart underlie changes in ECG in this case? Substantiate your answer.

79

3. What metabolic disorders in the myocardium could result in the indicated electrophysiologic changes? 4. Is the described episode of cardiac disorder life-threatening? Explain why, if it is. HEART FAILURE Questions to prepare for classes and examination: 1. Definition, general causes and classification of heart failure. 2. Myocardial and nonmyocardial cardiac insufficiency. 3. Adaptive reactions during acute and chronic heart failure: Frank-Starling mechanism, myocardial hypertrophy, redistribution of cardiac output, salt and water retention, activation of the sympathetic system. 4. Pathogenesis of the myocardium decompensation during hypertrophy. 5. Structural and hemodynamic abnormalities in heart failure. 6. Clinical features of heart failure: the syndromes of cardiac edema and cachexia. N 93* A 40-year-old patient Z. suffers from tuberculosis. He has come to see his physician with complaints of dyspnea, pains in the right upper quadrant, and subfebrile fever. These symptoms emerged 4 weeks ago and were progressing. On examination: the face is pale and rounded, orthopnea is evident (the patient is sitting upright with his hands resting upon chair; this posture is necessary to ease breathing); the area of cardiac dullness is increased to the left and to the right by 2 cm, heart rate is 100 per min, blood pressure is 90/60 mm Hg. On auscultation: muffled heart sounds, crackles at the base of the lungs. Breathing rate is 26 per min. The neck veins are distended; the liver is tender on palpation, and extends 3 cm below the costal margin; there is trace pretibial and feet edema. X-ray examination of the chest shows the globular shadow of the heart. Questions: 1. Does the patient have signs of cardiac insufficiency? Substantiate your answer.

80

2. What additional tests are required to specify the form of the heart pathology in this case? 3. Make a conclusion about the form of cardiac insufficiency in this patient. N 94 A 48-year-old patient N. suffered from polycythemia vera for 7 years. Two years ago he was examined by the cardiologist, who found a hyperkinetic type of his blood circulation: stroke volume was increased by 35%, and systemic vascular resistance decreased by 30% compared with the corresponding normal values; systemic venous return to the heart was significantly elevated. Two days ago the patient was admitted to hospital with complaints of weakness, dizziness, dyspnea in the supine position (orthopnea), nocturia, palpitations, lower extremities edema in the evening, poor appetite. On examination: the patient is pale; he has evidence of acrocyanosis; the heart size is increased. Ausculation reveals displacement of the point of maximal impulse down and to the left, and the presence of rales. The results of blood tests are the following: Hb 205 g/L, erythrocyte count 7.0*1012/L, platelets 520*109/L, leukocyte count 14.5*109/L; few normoblasts are present. Cardiac index is 2.5 L/min/m2 (normal values: 2.7-3.0 L/min/m2); systemic vascular resistance is reduced by 17% compared to the normal. An X-ray investigation of the chest demonstrates an increase in the size of the heart cavities and signs of congestion in the lungs. Questions: 1. What forms of pathology developed in the patient? Substantiate your answer. 2. What specific form of pathology was the immediate reason for the patient's hospitalization? Explain your opinion. 3. What are causes and mechanisms of this pathology in the given patient? 4. What mechanisms underlie each of the symptoms in the given patient? N 95 A 26-year-old female patient, who suffered from rheumatic fever since childhood, was admitted to hospital with complaints of recurrent pain in the heart, palpitations, particularly in the supine position, dyspnea, syncope.

81

These symptoms appeared at the end of pregnancy and increased after childbirth; they were provoked by strenuous exercise. Blood tests show the following: Hb 129 g/L, erythrocytes 4.1*1012/L, leukocytes 4.2*109/L; platelets 250*109/L; blood serum cholesterol is within the normal range. Blood pressure is 165/50 mm Hg; cardiac dullness is increased to the left; apical impulse is enhanced and displaced down and to the left. On auscultation early diastolic murmer is heard in the right second intercostal space. ECG reveals the signs of the left axis deviation; during physical exercise on bicycle ergometer the ST segment is depressed. An X-ray examination shows an increase of the heart size, particularly to the left, and enlargement of the proximal portion of the aorta. Questions: 1. What forms of pathology developed in the patient? Substantiate your answer. 2. What is the cause-effect association between these forms of pathology? Substantiate your answer. 3. What mechanisms underlie each of the symptoms in this patient? 4. Are there any signs of the adaptive mechanisms compensating for the heart insufficiency in this case? If there are, describe them, explain how do they operate, and what is their significance. N 96 A 42-year-old patient with acquired mitral valve insufficiency, associated with rheumatic heart disease, presents with the following symptoms: an increased area of the cardiac dullness, enlargement of the liver, accumulation of fluid in the abdominal cavity, decreased daily urinary volume, a significant edema of the lower extremeties, and an increased level of aldosterone in the blood. Questions: 1. What type of cardiac insufficiency developed in this patient? Substantiate your answer. 2. What are the main mechanisms of decompensation of the myocardial contractile function in this case?

82

3. What mechanisms underlie the development of aldosteronism in cardiac insufficiency? 4. Taking into account the described symptoms, explain the pathogenesis of edema in this patient. N 97 Patient M. suffers from arterial hypertension. He is visiting his physician with complaints of recurrent dyspnea with difficult inspiration, especially during physical exertion. Several days before he had a night attack of severe inspiratory dyspnea accompanied by shortness of breath and fear of death. He called in an ambulance. When it came the ambulance physician gave him an injection of a medicine that quelled the attack. The physician made a preliminary diagnosis of asthma based on the patient's history. During the examination in hospital the patient's blood pressure is 155/120 mm Hg, X-ray investigation shows an increase in the size of the left ventricle of the heart. Question: 1. What type of asthma (bronchial or cardiac) developed in the patient? Substantiate your view. 2. What is the immediate cause of it? Is this cause related to overexertion of the heart? If it is, which of the ventricles is affected, and by what factor: preload or postload? 3. What is the pathological basis (on the cellular and subcellular levels) of impairment of the cardiac contractility? 4. What are the main principles of treatment of heart failure? DISORDERS OF SYSTEMIC BLOOD PRESSURE. ARTERIAL HYPOTENSION AND HYPERTENSION Questions to prepare for classes and examination: 1. Principles of physiological control of systemic blood pressure. 2. Typical forms of disorder of systemic blood pressure. 3. Arterial hypotension: essential and symptomatic. 4. Types and mechanisms of development of symptomatic arterial hypotension. 5. Definition and principles of classification of arterial hypertension.

83

6. Renal arterial hypertension: pathogenesis of renovascular, renoprival, renal parenchymal hypertension. 7. Pathogenesis of endocrinogenic and neurogenic arterial hypertension. 8. Essential hypertension: etiology and pathogenesis. 9. Clinical variants, stages and principles of treatment of essential hypertension. N 98 A 39-year-old patient visited his physician with complaints of regular headaches which he usually treated by aspirin. According to the patient's history the headaches first appeared 4 months ago, and before that the patient had no health problems, though occasionally he had elevations of the blood pressure. On examination: the patient's skin and visible mucosal membranes are hyperemic; blood pressure is 185/125 mm Hg; the liver is enlarged: the shins and soles are swelled. The patient was addmitted to hospital. Blood test performed in the hospital showed erythrocytosis (r.b.c. count 7.2*1012/L), leukocytosis (w.b.c. count 11.2*109/L), thrombocytosis (platelets count 650*109/L). Questions. 1. What forms of pathology developed in the patient? Substantiate your answer. 2. Define the type of arterial hypertension which the patient suffers from, describe its pathogenesis. 3. How can the observed changes in the blood contribute to the pathogenesis of arterial hypertension in this patient? Explain the mechanism. N 99 A 36-year-old patient M. visited his physician with complaints of recurrent episodes of severe headache, blurred vision, face flushing, sweating, dizziness, palpitations, and anxiety. These symptoms usually occur during physical exertion or in situations of intense psychologic stress. At rest the patient's blood pressure is 136/85 mm Hg, heart rate 80 beats/min. Blood and urine tests are unremarkable. After a loading exercise the patient's blood pressure rose up to 230/165 mm Hg, heart rate 188 beats/min, glucose blood

84

level 190 mg/dl. In the urine sample, collected immediately after the exercise, there is an increased concentration of catecholamines and their metabolites. An X-ray investigation of the abdominal region shows enlargement of the right adrenal gland. Questions: 1. Name and describe forms of pathology developed in the patient. 2. What is the mechanisms of the increased blood pressure in this patient? 3. What mechanisms contribute to the development of each of the presented symptoms? N 100 A 46-year-old male patient was admitted to hospital with complaints of palpitations, headache, sleep disturbances, nervousness, irritability, and fatigability. On examination in the hospital: blood pressure is 160/100 mm Hg, heart rate 92 beats/min; stroke volume is increased; vessels of the eye fundus are constricted, their walls are thickened; angiography of the kidneys and adrenal glands is unremarkable; the level of catecholamine products and corticosteroids in the urine is elevated. Questions: 1. What forms of pathology developed in this patient? Substantiate your answer. 2. Which of them appears to be the primary factor of pathology in this patient? What is the likely cause and mechanism of development of this form of pathology? Substantiate your answer using the given clinical data. 3. What treatment approaches are relevant in this case? N 101 A 25-year-old male sportsman visited his physician with complaints of sudden muscle weakness, headache, dizziness during physical exercise, increased frequency and volume of urination. These symptoms appeared 3 weeks ago. On examination: blood cell count is within the normal range, there is evidence of hypernatremia and hypokalemia; blood pressure is 165/110 mm Hg; daily urine volume is 3500 ml, hyposthenuria; an X-ray examination of the chest shows increased size of the heart.

85

Questions: 1. What forms of pathology may be suspected in this patient? Substantiate your answer. 2. What additional data are required to make an unequivocal conclusion about the patient's disease? Explain clinical significance of these data. 3. What mechanisms underlie the symptoms presented by this patient? N 102 A 58-year-old female patient K. suffers from arterial hypertension. During the last 1.5 months she noticed her gaining body weight, emergence of cold intolerance, numbness and pain in the gastroknemius muscle on walking, and then at rest. The latter usually occurred at night causing sleep disturbances. Five months ago an erosion appeared in the lower one third of the right shin; later it transformed into an ulcer which was painless and resistant to treatment. She had a persistent subfebrile fever (37.2-37.4oC body temperature). During her visit to the doctor the patient also complained of a dry mouth, thirst, drinking much water (up to 4-5 L daily), frequent and excessive urination. On examination: the skin of the shins is dry, pale, and cold; palpation of the sole and the area behind the knee fails to reveal pulsation of arteries. Blood tests show increased levels of cholesterol and fibrinogen; elevated platelet count; fasting glucose blood level is 180 mg/dl. Questions: 1. What form of pathology, in addition to arterial hypertension, can be suspected in this patient? Substantiate your answer by the given clinical and laboratory data. 2. What can be the possible cause of this form of pathology? 3. What mechanisms underlie the development of this pathology and the presented symptoms? 4. Is there any cause-effect association between the observed pathology and the shin ulcer in this patient? If there is, characterize this association. If there is not, explain the development of ulcer in this case. N 103

86

A 48-year-old patient K. visited his physician with complaints of recurrent episodes of intense headache in the occipital area, blurred vision, accompanied by rigor, nausea, and, usually, vomiting. These symptoms were provoked by prolonged periods of psychologic stress. During the last episode, which had ocurred 2 weeks ago, the physician of the ambulance had found the increased blood pressure, 195/110 mm Hg, in the patient's right hand, and 200/115 mm Hg in the left hand; signs of coronary insufficiency and paroxysmal ventricular tachycardia. On examination: blood pressure was 195/115 mm Hg; the patient experiences intense weakness, dizziness, nausea; he has problems with coordination due to weakness of the right hand and leg; his speech is slurred, responses are slow and often off the mark. Questions: 1. What forms of pathology developed in this patient? Substantiate your answer. 2. Which of these forms of pathology is the most significant in determining the patient's state? What are their likely causes and mechanisms of development? 3. Does the cause-effect association exist between the mentioned forms of pathology? If it does, explain the point of this association. N 104 A 57-year-old patient A., the chief of a big trading company, was hospitalized after a profilactic examination, which disclosed the following: blood pressure 170/100 mm Hg, the heart rhythm is regular, 89 beats per minute, the circulating blood volume is 20% higher than normal, the heart area is increased to the left, the apical impulse is accentuated. ECG reveals the signs of the left ventricular hypertrophy. During funduscopy attenuated retinal arterioles and increased tortuosity of the microvascular network are observed. The breathing rate is 21 per min. Blood tests show the following: erythrocytes 6.0*1012/L, Hb 158 g/L, leukocytes 4.0*109/L, platelets 330*109/L; hypernatremia, the aldosterone levels are within the normal range, hypercholesterolemia. The patient is emotional, nervous, denies smoking. Questions:

87

1. What form of pathology developed in the patient? Substantiate your answer. 2. What is the likely cause and the main stages of pathogenesis of this pathology? 3. How can you explain the presence of hypernatremia and an increased blood volume in the setting of the normal aldosterone levels? 4. What pathologic state can be suspected on the basis of tachycardia, hurried breathing and erythrocytosis? What are the causes and significance of these symptoms? N 105 A 40-year-old patient has been admitted to hospital with arterial hypertension. Blood pressure 175/115 mm Hg. The patient complains of muscle weakness and headaches. Laboratory testing shows polyuria, severe hypokalemia, increased content of 17-hydroxycorticosteroids in the urine. Ultrasound investigation revealed a mass in the right adrenal/ Questions: 1. What is the likely cause of arterial hypertension in this patient? 2. What are the presumable mechanisms of the sustained increase in blood pressure in this case? 3. What additional tests are needed to specify the diagnosis? Describe their likely results which could confirm your preliminary diagnosis. PATHOPHYSIOLOGY OF RESPIRATORY SYSTEM Questions to prepare for classes and examination: 1. Typical forms of disorder of the respiratory system: disorders of alveolar ventilation, disorders of pulmonary perfusion, disorder of ventilation-perfusion matching, disorder of diffusive capacity of the alveolar-capillary barrier. ventilation. 3. Alveolar hyper- and hypoventilation: causes, mechanisms of development, consequences, and typical changes in breathing volumes.

2. Static and dynamic parameters of breathing used to characterize

88

4. Disorders of the neurogenic control of breathing. Pathological forms of breathing. 5. Types and consequences of perfusion dicorders: pulmonary arterial hyperand hypotension, venous hypertension. 6. Types and mechanisms of ventilation-perfusion mismatch. 7. Diseases characterized by abnormal diffusive capacity of the alveolarcapillary barrier. Pneumoconioses. 8. Combined disorders respiratory function. Adults respiratory distress syndrome. N 106 A 38-year-old patient K. has a history of heavy smoking more than 1.5 packs of cigarettes per day for the last 12 years. He complains of recurrent bronchitis and tracheitis, persistent cough with secretions, and dyspnea during physical exertion. On examination: the patient has asthenic body habitus, the barrel chest; he uses accessory muscles during breathing. X-ray of the chest shows diminished lungs vascular markings, diaphragm depression and the bronchioles' walls thickening. Arterial blood gases: PaO2 PaCO2 SaO2 Total lung capacity Vital capacity Inspiratory reserve volume Expiratory reserve volume Residual volume Tiffeneau ratio(FEV1/FVC) 83 mm Hg 49 mm Hg 96.1% 119% 80% 86% 88% 114% 85%

Oxygen carrying capacity 19.0 vol% The results of spirometry (% to normal values):

Functional residual volume 112%

The patient has signs of diminished diffusive capacity of the alveolocapillary membrane; his breathing rate is 20 per min. After injection of a bronchodilator drug aminophylline (euphylline) the value of Tiffeneau ratio increases by 7%.

89

Questions: 1. Does the patient have signs of impaired alveolar ventilation? If he does, what type of hypoventilation is observed: obstructive or restrictive? What is the cause of this disorder? 2. Are there signs of the lung perfusion disorder in this case? If there are, explain its possible causes and mechanisms of development. 3. Are there signs of increase in the volume of small airway closure? If there are, explain its cause. If there are no, substantiate your point of view. 4. Make a general conclusion about the condition of the respiratory function in this patient? N 107* A 36-year-old patient K., a mine worker, has been admitted to hospital with a preliminary diagnosis of silicosis. He complains of dyspnea, more severe on walking and physical exercise; sustained cough (occasionally with secretions), chest pain. ___________________________________________________________ Arterial blood gases: PaO2 PaCO2 Oxygen carrying capacity SaO2 Spirometry: Forced vital capacity (FVC) FVC in % to normal Forced expiratory volume in 1 s (FEV1) Tiffeneau ratio (FEV1/FVC) Additional data: Breathing rate 19/min ___________________________________________________________ The voluntary hyperventilation test yields the PaO2 value of 92 mm Hg. Questions: 4.2 L 92% 2.6 L ? 90 mm Hg 40 mm Hg 19.2 vol% 94.3%

Pulmonary minute volume (% of normal) 124%

90

1. Does the patient have disturbances of respiratory function? Describe the signs of it, if he does. 2. Does the patient have disturbances of alveolar ventilation? If he does, define its type (obstructive or restrictive). 3. Taking into account the preliminary diagnosis of pneumoconiosis how to test the diffusive capacity of the lungs in this patient? 4. Make a final conclusion about the type of respiratory system disorder in patient K. N 108 A 56-year-old patient K. experienced myocardial infarction 4 weeks before. He complains of progressive dyspnea, accompanied by a cough with scanty blood-stained sputum. ECG demonstrates signs of the former myocardial infarction. Evaluation of the respiratory function shows the following: Breathing rate Forced vital capacity (% of the normal value) Total lung capacity (% of the normal value) Tiffeneau ratio Questions: 1. Does the patient have signs of impaired alveolar ventilation of the obstructive type or restrictive type? Substantiate your answer. 2. Does the patient have signs of the pulmonary perfusion disorder? If he does, describe its causes and mechanisms of development. 3. Are there any signs of diffusive capacity disorder? What tests can be used to assess the diffusion of gases across the alveolocapillary membrane? 4. Make a final conclusion about the state of the respiratory system in this patient. N 109 A 65-year-old patient A. visited his physician with complaints of choking sensation that emerges when he falls to sleep. Choking attacks are accompanied by fear of death and severe palpitations, which are sometimes followed by retrosternal pain radiating to the left scapula. The use of 26/min 76 70 76%

Pulmonary minute volume (% of the normal value) 140

91

nitroglycerin during choking attacks and pain in the heart had no effect. However, if the patient wakes up and makes several deep voluntary inspirations the unpleasant sensations subside. Blood test data and ECG are unremarkable. The patient has a history of viral infection complicated by signs of polyneuropathy; in his childhood he had polymyelytis. Questions: 1. What form of pathology of the respiratory system does the patient suffer from? Substantiate your answer. 2. What is the possible cause of this pathology? Can you define the level at which the control of the ventilatory function is lost: respiratory center, afferent pathways, efferent pathways? Substantiate your point of view. 3. Why is pulmonary disorder in this patient accompanied by episodes of angina-like pain in the heart? 4. Why do choking episodes never occur when he is awake? N 110 A 36-year-old patient K. has been admitted to hospital with complaints of dyspnea, exacerbating on walking and physical exercise,and persistent cough. Examination shows cyanosis of the mucous membranes, rhonchi during auscultation of the lungs, hyperresonance on percussion of the chest. Testing of pulmonary function and arterial blood gases reveals the following: Arterial blood gases: PaO2 PaCO2 Oxygen-carrying capacity SaO2 Spirometry: Total lung capacity Forced vital capacity Inspiratory reserve volume Expiratory reserve volume Residual volume Increased Decreased Decreased Decreased Increased 84 mm Hg 50 mm Hg 19.8 vol% 94.4%

___________________________________________________________

Functional residual capacity Increased

92

Tiffeneau ratio (FEV1/FVC) Decreased The voluntary hyperventilation test yields the PaO2 value of 86 mm Hg. Administration of bronchodilator drug aminophylline (euphylline) results in a significant increase in Tiffeneau ratio in this patient. Questions: 1. Does the patient have signs of reduced airways patency? If he does, describe them, and characterize their diagnostic value. 2. Does the patient have signs of impaired elastic recoil of the lung tissue? Describe them, if he does. 3. Can we suspect the disturbance of diffusive capacity in this patient? Substantiate your answer. 4. Make a final conclusion about the state of respiratory function in this patient. N 111 A 56-year-old patient M. has been repeatedly admitted to the therapeutic department of hospital with complaints of dyspnea during moderate physical exertion, a nonproductive cough, episodes of choking, associated with cough and wheezing which is more marked on expiration. Testing of respiratory function and arterial blood gases reveals the following: PaO2 PaCO2 Forced vital capacity (FVC) FVC (% of the normal value) Forced expiratory volume in 1 s (FEV1) Tiffeneau ratio Residual volume/Total lung capacity (% of the normal value) Maximal expiratory flow rate (% of the normal value) 98% After administration of bronchodilator drug aminophylline (euphylline) Tiffeneau ratio increases by 15%. The cough at the end of a wheezing episode produces thick, stringy mucus taking the form of casts of the distal 110% 90 mm Hg 30 mm Hg 3.6 L 86% 2.1 L ?

Pulmonary minute volume (% of the normal value) 119%

93

airways (Curschmann's spirals). Blood test reveals the following: Hb 136 g/L, erythrocyte count 5.5*1012/L, leukocyte count 9*109/L, eosinophilia. The chest roentgenogram shows increased retrosternal translucency. Questions: 1. Are there some data in favor of changes in compliance (increased or decreased) of the lung tissue? Substantiate your answer. 2. Can we suspect a disorder of alveolar ventilation of the obstructive type in this case? If we can, what are its causes and mechanisms? 3. Does the patient have signs of increased volume of the small airway closure? If he does, explain the mechanism of this phenomenon. 4. Make a final conclusion about the state of respiratory function in this patient. PATHOPHYSIOLOGY OF DIGESTION Questions to prepare for classes and examination: 1. Typical forms of functional pathology of the digestive tract: general etiology and mechanisms. 2. Disorders of appetite, taste, and initial treatment of food in the oral cavity. 3. Disorders of swallowing: dysphagia, achalasia, diffuse esophageal spasm. 4. Disorders of digestion in the stomach: disturbance of secretory and motor function. Dumping syndrome. 5. Disorders of digestion in the intestine: disturbance of secretion, motor function, and absorption. 6. The syndrome of malabsorption: etiopathogenesis, manifestations and consequences. 7. Pathogenesis of peptic ulcer of the stomach and duodenum. 8. Disorders of excretory function of the pancreas. N 112 A 43-year-old patient N. suffers from severe chronic gastritis and enteritis. He has come to see his physician with complaints of recurrent diarrhea, progressive malaise, weakness, wasting, and generalized edema. The examination reveals the following: hypochromatic anemia, signs of multiple vitamin deficiencies, adrenal insufficiency, and immunodeficiency state. Stool

94

test findings: the presence of undigested muscle fibers (creatorrhea), steatorrhea, and amylorrhea. The results of suction biopsy of the small intestine are as follows: reduced activity of the intralumenal and brush border fractions of digestive enzymes, atrophy of the enterocytes' microvilli. Questions: 1. Define the syndrome developed in the patient. Substantiate your answer. 2. What are the possible causes and pathogenesis of this syndrome? 3. What disturbancies of digestion can be found in this patient? 4. Describe pathogenesis of each of the symptoms found in patient N. N 113 A 35-year-old patient D. has been admitted to hospital with complaints of severe episodes of "gnawing" pain (burning sensation) in the epigastrium occurring 2-3 hours after a meal. Recently the pains have aggravated, and became accompanied by nausea and, occasionally, vomiting. Vomiting makes him feel better. The pains in the epigastrium also occur at night and wake him up; they are relieved by food and alkalies. The patient reports about increased nervousness and irritability; he acknowledges heavy smoking and excessive alcohol consumption. Eight months ago he divorced and moved to a hostel near his place of work. During the last months he lost 8 kg of his body weight. On the basis of the patient's complaints and laboratory data the physician made a diagnosis and prescribed a treatment. Questions: 1. What form of pathology does the patient suffer from? Substantiate your opinion. 2. What laboratory tests are required for accurate diagnosis, and what could be their possible results? 3. What is the most likely cause of pathology in this case, and what are its most significant risk factors in the given patient? 4. Explain the mechanisms of pain (burning) in the epigastrium, and severe weight loss in this patient. N 114

95

A 46-year-old patient B. has come to see his physician with complaints of progressive pain in the periumbilical region of the abdomen. These episodes usually occur after meal, and the pain radiates to the back. He also complains of anorexia, recurrent diarrhea, nausea, frequent vomiting, weakness, sensation of fullness or the presence of a foreign body in the epigastrium. Last year the patient lost 12 kg of body weight. During the last 7 months he began to feel persistent thirst (drank about 5 L of fluid daily), and frequently urinated. The patient reports alcohol abuse during the last 15 years. Ten years ago after an episode of excessive drinking he experienced an acute attack of pancreatitis, and was treated in hospital. Later he had three episodes of acute abdominal pain with dyspepsia, but did not see his physician or take any treatment. He continued excessive drinking and did not keep to any specific diet. Laboratory testing shows the following: blood glucose level 15 mmol/L, decreased glucose tolerance, urine glucose 4%, 24-h urine volume 3-4 L; significant steatorrhea, 5-fold decrease in the maximum trypsin activity and low bicarbonate concentration in the pancreatic juice. The results of the X-ray, ultrasonic investigation, computer tomograpgy of the pancreas: diffuse infiltration of the gland, foci of the increased density (calcareous deposits?). Questions: 1. What forms of pathology can be distinguished in this patient? 2. What are the causes of each of these forms? Substantiate your answer. 3. Is there any pathogenic association between these forms of pathology. If there is, what is their most likely sequence of development? 4. Explain mechanisms of the following symptoms in this patient: severe wasting, anorexia, weakness, bloating, nausea. PATHOPHYSIOLOGY OF THE LIVER. JAUNDICE. Questions to prepare for classes and examination: 1. General causes and mechanisms of liver dysfunction. 2. The syndrome of hepatic insufficiency. Typical disorders of carbohydrate, aminoacid, protein, and lipid metabolism in hepatic insufficiency; failure of detoxification function. 3. Causes, types and pathogenesis of hepatic coma.

96

4. Hemolytic jaundice: causes, mechanisms, clinical and laboratory manifestations. 5. Obstructive jaundice: types, causes, mechanisms, clinical and laboratory manifestations, consequences. 6. Hepatocellular jaundice: causes, mechanisms, stages, clinical and laboratory manifestations. 7. Primary enzymopathic jaundices: etiopathogenesis and manifestations of the Gilbert, Crigler-Najar, and Dubbin-Johnson syndromes. N 115 A 46-year-old patient B. has visited his physician with complaints of right upper quadrant tenderness, nausea, vomiting, weakness, fatigability. In the patient's history there is an episode of jaundice at the age of seven. On examination: the skin and conjunctiva appear jaundiced; blood pressure 110/75 mm Hg, heart rate 86 per min; the liver is enlarged and nontender on palpation; the spleen is significantly enlarged; the stool is normally coloured; the urine is dark. Blood test data: Hb 72 g/L, erythrocyte count 3.0*1012/L, reticulocytes 11%; leukocyte count 3.7*109/L, white blood cell count differential is unremarkable; ESR 9 mm/h. Microscopy of the blood smear demonstrates anisocytosis, poikilocytosis, and spherocytosis of erythrocytes. Osmotic fragility test shows increased hemolysis. Blood biochemistry data: total protein 60 g/L (N:70-80 g/L), total bilirubin 5.8 mg/dl (N<1.3 mg/dl), conjugated bilirubin 0.15 mg/dl (N<0.30 mg/dl), urobilinogen is detected; serum AST activity 52 U/L (N< 38 U/L); glucose level 75 mg/dl. Urine test data: 24-h urine volume 1450 ml, specific gravity 1028, protein and glucose are not detectable; positive reaction for Hb. Questions: 1. What forms of pathology does the patient suffer from? 2. Are there signs of hepatic insufficiency in this patient? 3. What is your conclusion about the state of the bile pigment metabolism in this patient? What is the pathological basis of changes in the bile pigment metabolism in this case?

97

4. What changes in the bile pigment metabolism can be expected in acholia and cholemia? N 116 A 28-year-old patient C. has been admitted to hospital with complaints of right upper quadrant tenderness, dyspepsia, weakness, fatigability, depressed mood. The patient noticed that recently his stool has become clay-colored. Two months ago the patient was bit by a dog and received injections of rabies antiserum. On examination: the skin and conjunctiva appear jaundiced; the liver is enlarged, soft, tender on palpation; the spleen is enlarged; signs of moderate edema of the face, upper and lower extremities. Blood test data: Hb 105 g/L, erythrocyte count 3.5*1012/L, leukocyte count 6.7*109/L, ESR 26 mm/h. Blood biochemistry results: hypo- and dysproteinemia; total bilirubin 5.8 mg/dl (N<1.3 mg/dl), conjugated bilirubin 4.1 mg/dl (N<0.3 mg/dl), urobilinogen is present, glucose 75 mg/dl, serum ALT 510 U/L (N< 38 U/L), AST 495 U/L (N< 38). Questions: 1. What forms of pathology can be distinguished in this patient? Substantiate your answer. 2. Is there cause-effect association between these forms of pathology. If there is, which of them is primary? Explain its possible cause, mechanisms of development, and outcome. 3. Explain causes and pathogenesis of symptoms observed in this patient? N 117 A 52-year-old patient B., an employee of the chemico-pharmaceutical company, has been brought to hospital by an ambulance. On examination in the emergency room: the patient appears drowsy and apathetic, his speech is slow and incoherent; the tongue is covered with a yellow fur; body temperature 36.9oC; the skin and conjunctiva appear jaundiced. There are spider angiomata on the skin of his arms and upper torso, and erythema of the palms. The abdomen is tensely distended due to the accumulation of ascitic fluid making palpation of the liver difficult; there is evidence of the face and extremities edema.

98

Results of the immediate blood test: Hb 102 g/L, erythrocyte count 3.6*1012/L, leukocyte count 4.8*109/L, ESR 35 mm/h. Blood biochemistry data: hypo- and dysproteinemia, total bilirubin 70.6 umol/L (normal values: 8.6-20.5 umol/L), conjugated bilirubin 13.7 mol/L (normal values < 5.14 mol/L), cholic acids are present, plasma glucose 5.3 mmol/L (normal values: 4.4-6.7 mmol/L), increased level of ketone bodies, decreased concentration of blood urea nitrogen (BUN), prothrombin index decreased, choline esterase activity decreased, total creatine kinase activity normal, ALT activity increased; HBsAg is not detectable. Questions: 1. What forms of pathology does the patient suffer from? Substantiate your answer. 2. What is the primary pathology in this case and what are its possible causes and mechanisms of development? 3. Is there any association between the observed forms of pathology? Substantiate your answer. 4. What are the possible outcomes of the patient's condition? PATHOPHYSIOLOGY OF KIDNEYS Questions to prepare for classes and examination: 1. General causes and mechanisms of kidneys dysfunction. Disorders of filtration, tubular reabsorption and excretion. 2. Renal insufficiency: definition, general causes, mechanisms and manifestations. 3. Uremic syndrome: definition, typical manifestations and consequences. Uremic coma. 4. Syndromes of acute and chronic renal insufficiency: typical features, causes and manifestations. 5. Nephrotic and nefritic syndromes: typical features, causes and manifestations. 6. Pyelonephritis: characteristic features, causes and pathogenesis. 7. Nephrolitiasis: causes, mechanisms of development, consequences. 8. Characteristic changes in urinalysis in syndromes of renal insufficiency.

99

N 118 A 48-year-old patient K. has been suffering from chronic diffuse glomerulonephritis for 5 years. During the last several weeks he notes the emergence of the following symptoms: dull aching in the region of the heart, palpitations, generalized edema, especially prominent in the lower extremities on walking. Urine test data: 24-h urine volume 1100 ml, specific gravity 1042, protein content 3.3%. The urinary sediment microscopic examination reveals the presence of a large amount of granular and waxy casts. Blood pressure 170/95 mm Hg. Blood biochemistry data: blood urea nitrogen 6.9 mmol/L (normal values: 3.3-6.6 mmol/L), total protein content 48 g/L (N: 70-80 g/L), albumine 15 g/L (N:40-50 g/L), globulins 28 g/L (N: 20-30 g/L), hyperlipidemia, hypernatremia. Acid-base status: pH 7.34, PaCO2 33 mm Hg, SB= 17.0 meq/L, BB= 36 meq/L, BE= -7 meq/L, titratable acidity 10 mmol/d, NH4+ 18 mmol/d. Questions: 1. What forms of pathology are observed in this patient? What is the causeeffect and chronologic relationship between these forms of pathology? Substantiate your answer. 2. What specific physiologic processes in the kidneys are deranged to produce excretory disorder in this patient? 3. What syndrome is characterized by the following combination of symptoms: generalized edema, proteinuria, hypoalbuminemia, hyperlipidemia. What are the mechanisms of each of these symptoms? 4. Are there signs of uremia in this patient? Substantiate your answer. N 119 A 35-year-old patient M. complains of generalized edema, more evident after sleep, back pains, fever, and body weight gain of 6 kg. These symptoms developed 2 weeks after he had had an acute sinusitis. His urine test results are as follows: 24-h urine volume 650 ml, specific gravity 1028, protein content 0.1%, glucose and acetone - not detected. Microscopy of the urinary sediment reveals 40 erythrocytes per high-power microscopic field, a great

100

amount of leukocytes, hyaline and erythrocyte casts in moderate quantities. Blood pressure 150/110 mm Hg. Blood test data: BUN 10.4 mmol/L (normal values: 3.3-6.6 mmol/L), total protein content 73 g/L (N:70-80 g/L), hypernatremia, hypokalemia. Endogenous creatinine clearance 50 ml/min (N: 120 ml/min). Acid-base status: pH 7.3, PaCO2 32 mm Hg, SB= 16.5 meq/L, BE= -8 meq/L, titratable acidity of the daily urine 8 mmol/d, NH4+ 15 mmol/d. Questions: 1. What forms of pathology can be distinguished in this patient? What is their cause-effect and chronologic association? 2. What factor caused injury to the kidneys? 3. Describe stages of nephropathy in this case. Define this form of nephropathy. 4. What additional data are required to specify the diagnosis? 5. What are the mechanisms of each of the observed symptoms? 6. Does this patient have renal insufficiency? Substantiate your view. N 120* A victim of the vehicle accident was transported to hospital 5 hours after the accident. He was examined by a physician of the emergency service who found multiple ribs fractures, contusions of soft tissues of the pelvis and lower extremities with extensive hemorrhages. On admission the patient is confused, markedly pale; he has a thready pulse, blood pressure 60/20 mm Hg, periodic pattern of breathing. After a day of intense treatment by volume expanders (he received 3 L of polyglukin) and infusion of 0.5 L of blood his blood pressure increased to 110/60 mm Hg. On the next day after infusion therapy diuresis was still absent, and during the following three days the patient's condition remained guarded. He complained of severe headache, dizziness, cyclic vomiting, inhibited state. The patient had short-termed episodes of seizures, edema of the skin; bradycardia and extrasystole on ECG records; his diuresis was at the level of 150-250 ml/day, but blood pressure increased to 160/90 mm Hg. Blood test shows: BUN (blood urea nitrogen) 90 mg/dl (N: 20-40 mg/dl), hyperkalemia, hypermagnesimea, hyponatremia, hypochloremia, pH 7.30. Urine test: specific gravity 1.040, mild

101

proteinuria, myoglobinuria; sediment shows the presence of casts and few leukocytes in the h.p.f. On the 5-7th day in the hospital the patient developed great elevation of diuresis (up to 2500 ml/day), and his condition markedly improved. Vomiting, seizures and headache ceased, and the extent of edema reduced. Repeated urinalysis shows: specific gravity 1.010-1.012, slight proteinuria, large quantity of granular casts in the urine sediment. Questions: 1. Define the type of renal syndrome in this patient. What are its causes? 2. What are the causes of anuria during the shock before normalization of blood pressure in this patient? 3. Why was diuresis not restored despite extensive transfusion theraspy? 4. What are the mechanisms of symptoms observed on the 2-4th day after the trauma? N 121 A 22-year-old female patient K. developed pain in the back, dyspnea, palpitations, and headache 2 weeks after she had had severe tonsillitis. She gained 9 kg of body weight in 4 days. On examination: paleness of the face, eyelids are edematous, the palpebral fissures are narrowed; trace pretibial and feet edema; the area of cardiac dullness is increased. Blood pressure 140/95 mm Hg; the 24-h urine volume significantly decreased; there is marked proteinuria. Microscopic examination of the urine sediment reveals the presence of a great amount of erythrocytes, leukocytes, granular casts. In the blood there are high titers of the antistreptolysin-O and antihyaluronidase antibodies. Questions: 1. What forms of pathology developed in the patient? 2. What is the cause-effect relationship between these forms of pathology? Substabtiate your answer. 3. What are the cause and mechanisms of derangement of the renal function? 4. What are the mechanisms of each of the symptoms developed in this patient? Which of the following mechanisms caused hyperhydration in this

102

patient: a decreased rate of renal excretion or extrarenal factors affecting fluid retention in the body? N 122 A 33-year-old patient N. had acute diffuse glomerulonephritis 4 years ago. He is visiting his physician with complaints of recurrent headaches, malaise, dizziness, and moderate generalized edema. Urine test results: 24-h urine volume 3600 ml, specific gravity 1006, protein content 0.6%, glucose and acetone not detected. Microscopic examination of the urine sediment shows the presence of occasional dysmorphic erythrocytes per high-power microscopic field, a small quantity of hyaline and waxy casts. Blood pressure 160/95 mm Hg. Blood test data: BUN 11.5 mmol/L (normal values 3.3-6.6 mmol/L), total protein 56 g/L (N: 70-80 g/L). Endogenous creatinine clearance 50 ml/min (N: 120 ml/min). Acid-base status: pH=7.3, PaCO2= 36 mm Hg, SB= 18 meq/L, BB= 36 meq/L, BE= -5 meq/L, titratable acidity of the daily urine 12 mmol/d, NH4+= 10 mmol/d. Questions: 1. What specific physiologic processes in the kidneys are deranged to produce impairment of the excretory function in this patient? 2. What are the possible causes and pathogenesis of each of these processes? 3. Can we assume the presence of renal insufficiency in this patient? 4. What are the mechanisms of polyuria, hyposthenuria, arterial hypertension, hyperazotemia, hypo- and dysproteinemia? N 123 A 52-year-old patient M. who has suffered from chronic glomerulonephritis for 12 years visits his physician with complaints of recently appeared symptoms of drowsiness in the day time and difficulty falling asleep at night, fatigability, apathy, sensation of fullness in the retrosternal and epigastric regions, nausea, diarrhea, and itching of the skin. On examination: blood pressure is 165/95 mm Hg, the area of the cardiac dullness is increased to the left, pericardial friction rub over the whole area of the heart during auscultation. Blood test data: anemia, leukopenia, significant azotemia, hypo-

103

and dysproteinemia. Endogenous creatinine clearance is 45 ml/min (N: 120 ml/min). Urine test data: 24-h urine volume 450 ml, specific gravity 1029, proteinuria. Microscopic examination of the urine sediment shows the presence of 10 erythrocytes per high-power microscopic field, a great amount of leukocytes, occasional granular and waxy casts. Acid-base status: pH= 7.32, PaCO2= 32 mm Hg, SB= 16 meq/L, titratable acidity of the daily urine 11 mmol/d, NH4+ 10 mmol/d. Questions: 1. Define the condition developed in this patient. 2. What is the most likely cause of this condition? Substantiate your answer. 3. What are mechanisms of the psychoneurologic, cardiologic, and hematologic symptoms, as well as of changes in the urine and acid-base balance in this patient? 4. What condition could develop if the observed symptoms continue to aggravate? PATHOPHYSIOLOGY OF THE PITUITARY, ADRENALS AND SEX GLANDS Questions to prepare for classes and examination: 1. General etiology and pathogenesis of endocrine disorders. 2. Typical forms of functional pathology of adenopituitary. Hypopituitarism: types, causes, mechanisms of development and manifestations. Sheehans syndrome, dwarfism, hypogonadotropic hypogonadism. 3. Hyperpituitarism: types, causes, mechanisms of development and manifestations. Acromegaly and gigantism; hyperprolactinemia. 4. Typical forms of disorders of neurohypophysis. Diabetes insipidus and syndrome of inappropriate arginine-vasopressin secretion. 5. Typical forms of adrenals dysfunstion. Hypercortisolism: types, pathogenesis and manifestations. Cushings disease and syndrome, hyperaldosteronism. 6. Hypocortisolism: types, pathogenesis and manifestations. Acute adrenal failure and Addisons disease, hypoaldosteronism, syndromes of adrenal androgen excess.

104

7. Disorders of adrenals medulla. Manifestations and consequences of phyochromocytoma. N 124 A 56-year-old man T. has met his relative after 6 years living apart. The relative was amazed by dramatic changes in T.'s appearance: thickened and ridged skin on the forehead, deepened frontal and nasolabial folds, the nose and lips enlarged, the mandible and malar bones were prominent with protrusion of orbital ridge, and the teeth became widely separated. T. has visited his physician who suspected acromegaly, and sent the patient's blood to laboratory to test the level of growth hormone. The results of the test showed that growth hormone level in the T.'s blood is within the normal range. Further examination revealed a decreased glucose tolerance, an increased level of glucose in the blood (8.3 mmol/L); the presence of hyperphosphatemia and glucosuria; pH of the blood 7.34, plasma concentration of ketone bodies 2.8 mg/dl (N:0.5-2.5 mg/dl), and that of lactic acid 2.2 mmol/L (N:0.99-1.75 mmol/L). Questions: 1. Do you agree with the diagnosis of acromegaly in this patient? Substantiate your answer. 2. Can we suspect the presence of diabetes mellitus in this patient? What clinical data may support this diagnosis? What mechanisms may determine the development of diabetes mellitus in this case? 3. What additional data are required for a final conclusion about the nature of pathology in this patient? 4. What mechanisms underlie the symptoms presented by the patient? N 125 A primary care physician has sent a 18-year-old male patient to hospital for a thorough examination. On admission: the patient is 185 cm tall and weighs 69 kg; he has narrow shoulders, abnormally long legs and increased thigh girth, high-pithched feminine voice, signs of gynecomastia, and diminished hair growth on the trunk, limbs, and beard area. The patient has underdeveloped musculature and decreased power (according to

105

dynamometric measuring), small penis and testes. Special tests revealed the presence of azoospermia, low IQ, and a karyotype of 47,XXY. Questions: 1. Name the form of pathology (or syndrome) presented by this patient, and explain its likely cause. 2. Define the form of endocrine pathology in this patient, and explain its cause. 3. What are the mechanisms of each of the symptoms observed in this patient? N 126 A 38-year-old patient M., a geologist, had tick-borne encephalitis 3 months before during his last expedition. When he returned home from his expedition he visited his doctor with complaints of malaise, frequent headaches, persistent thirst (drinks daily about 6-8 L of fluid), and frequent excessive urination. Blood test shows signs of normochromatic anemia, leukopenia, thrombocytopenia; hematokrit is 0.28. Urine test data: 24-h urine volume is 7.5 L, specific gravity is 1002, protein is not found, glucose and ketone bodies - not detected. Acid-base status of the capillary blood: pH 7.37, PaCO2 40 mm Hg, SB=22 meq/L, ketone bodies 1.1 mg/dl (N: 0.5-2.5 mg/dl), lactic acid 1.32 mmol/L (N:0.99-1.75 mmol/L). Questions: 1. What form of pathology does the patient suffer from? Substantiate your answer. 2. What are its most likely cause and mechanisms of development? 3. What condition with similar manifestations should the current form be differentiated from? 4. What are the mechanisms of the presented symptoms and changes in the laboratory parameters? N 127 A 40-year-old patient has been admitted to hospital with arterial hypertension of unclear etiology. His blood pressure is 175/115 mm Hg. The

106

patient complains of muscle weakness and headaches. Laboratory testing shows polyuria, severe hypokalemia, increased content of 17-hydroxycorticosteroids in the urine. Questions: 1. What is the likely cause of arterial hypertension in this patient? 2. What are the presumable mechanisms of the sustained increase in blood pressure in this case? 3. What additional tests are needed for acurate diagnosis? Describe their likely results which could confirm your preliminary diagnosis. N 128 A 36-year-old patient X. has visited his physician with complaints of severe headache, dizziness, episodes of palpitation and anginal pain, sweating, trembling and fear of death after intense physical exertion. On examination at rest: blood pressure 140/90 mm Hg, heart rate 76 beats per min; complete blood test and urinalysis are unremarkable. After controlled exercise the blood pressure increases up to 230/165 mm Hg, heart rate 188 beats per min, blood glucose 7.8 mmol/L. There is a significant increase in the urine content of catecholamines and their metabolites. An X-ray examination of the low back area reveals elevation of the left adrenal; size and contours of the kidneys are unremarkable. Questions: 1. What forms of pathology can be distinguished in this patient? 2. Is there any pathogenic association between these forms of pathology? 3. What factors cause the significant increase in systolic and diastolic blood pressure in this case? 4. What are the mechanisms of the presented symptoms? N 129 A 32-year-old male patient, a former professional boxer in heavy weight category, visited his physician with complaints of progressive gaining of weight (increased his body mass by 7 kg for 6 months), muscle weakness, easy bruising, dizziness, headaches (more often in occipital area), recurring

107

blurring of vision (specks dancing before the eyes), persistent thirst (drinks daily 5-6 L of fluid), and frequent excessive urination. On examination: the patient has a hypersthenic body habitus with excess fat deposition on the face (moon face), neck (buffalo hump), and over the clavicles; there are purplish striae of the skin on the abdomen and excessive hair growth on the chest and back; a large number of bruises of various colour is observed on the upper and lower extremities. Blood pressure is 185/110 mm Hg. Blood test data: Hb 130 g/L, erythrocyte count 5.1*1012/L, leukocytes count 10*109/L, neutrophilia, relative lymphopenia, and lack of eosinophils; ESR 5 mm/h, glucose level 8.7 mmol/L, hypernatremia. Urine test results: 24h urine volume 6000 ml, specific gravity 1025, glucosuria, protein and ketone bodies are not found; content of unbound cortisol is increased. Questions: 1. What forms of endocrine pathology does the patient suffer from? Substantiate your answer. 2. What is the cause-effect relationship between these forms of pathology? Which of them is primary and secondary? 3. What are the causes and mechanisms of development of these pathologic conditions? 4. What are the mechanisms of each of the symptoms presented by the patient? N 130 A 40-year-old patient B. has been transported to the emergency room by an ambulance. On examination: the patient appears lethargic, his speech is slowed; he complains of headache, severe weakness, pain in the abdominal area, nausea, vomiting; body temperature 39.5oC, blood pressue 65/40 mm Hg. He has a marked hyperpigmentation of the face, hands, lips, and skin folds. According to the patient's report his condition became worse after he had received news about his mother's death. Examination in the hospital reveals sustained arterial hypotension (blood pressure 70/50 mm Hg), hyperkalemia, hyponatremia, hypovolemia; blood glucose 2.9 mmol/L. ECG shows the presence of extrasystoles, episodes of atrial paroxysmal tachycardia and atrial flutter, atrio-ventricular conduction

108

disturbancies. Urine test demonstrates a decreased content of unbound cortisol and 17-hydroxycorticosteroids. Test with intravenous injection of corticotropin yields only a slight increase in the blood level of cortisol. A special immunologic test shows the presence of antiadrenal antibodies. Two years ago the patient had a gunshot wound in the back area. Questions: 1. Define the state which the patient had at the moment of admission to the hospital? What factor caused the development of this condition? Substantiate your answer. 2. Define the form of pathology in this patient taking into account the results of his examination in the hospital? What is the relationship between this form of pathology and the state in which the patient was transported to hospital? 3. What is the likely cause and pathogenesis of this form of pathology? 4. What are the mechanisms of each of the symptoms presented by the patient? PATHOPHYSIOLOGY OF THYROID GLAND Questions to prepare for classes and examination: 1. Synthesis and control of secretion of thyroid hormones. Physiological effects of thyroid hormones. 2. Hypothyroidism: types, causes, mechanisms of development and clinical manifestations. Common hypothyroid states: myxedema, cretinism, endemic goiter. 3. Hyperthyroidism: types, causes, mechanisms of development and clinical manifestations. 4. Pathogenesis and manifestations of Graves disease, Hashimotos thyroiditis, iod-basedow and Wolf-Chaikoffs phenomena. 5. Thyrotoxic and myxedema coma. 6. Typical disorders of parathyroid gland. Causes and manifestations of hypo- and hyperparathyroid states. N 131 A 30-year-old female patient L. complains of fatigability, muscle weakness, difficulty in falling asleep, palpitations, excessive sweating and heat

109

intolerance, frequent stool, and significant weight loss. Three years ago she experienced a severe viral infection, and since that time suffered from episodes of unexplained pain in the anterior neck in the region of the thyroid gland. She ignored these symptoms and did not visit her doctor. On examination: the thyroid gland is slightly increased, indurated, nontender on palpation; heart rate 98 beats per min; blood pressure 150/80 mm Hg; signs of the left heart ventricle hypertrophy; the skin feels warm and moist; exophthalmos is evident. When the patient is instructed to look at the finger moving downwards there is a moment before the upper lid catches up with the globe (lid lag). Concentration of thyrotropin in the patient's blood serum is decreased, while the content of IgG is increased. Questions: 1. What form of pathology does the patient suffer from? Substantiate your answer. 2. What is its cause and mechanisms of development? Does the increase in serum IgG have pathogenic significance? 3. What mechanisms underlie the symptoms developed in the patient? 4. What additional laboratory tests can be performed to confirm the diagnosis? N 132* An endocrinologist observes two patients during their follow-up visits: a 50year-old woman M. and her 26-year-old daughter D. Both patients have a significant enlargement of their thyroid gland and clinical signs of thyrotoxicosis. The results of their clinical and laboratory investigations suggest the diagnosis of the Graves disease. The patient D.s condition improved after a treatment course, while the patient M.s condition kept worsening after an 8-months course of medication by thyrostatic drugs. During reexamination M. presented with new complaints of sluggishness, motor retardation, drowsiness during the day time and difficulty going to sleep at night, memory problems, decreased working ability, edema of the face and extremities and cold intolerance. These symptoms emerged after M. had recovered from viral infection. The physician suspected the presence of Hashimotos thyroiditis in M., and changed the treatment. Questions:

110

1. Can we consider the emergence of new symptoms in M. as complication of the thyrostatic therapy? 2. 3. 4. 5.
6.

2. What laboratory tests can be made to confirm the diagnosis in M.? 3. What other clinical forms of thyroid gland pathology should be considered to make the final diagnosis? 4. Are there common mechanisms in the pathogenesis of Graves disease and Hashimotos thyroiditis?

7.

N 133 A 46-year-old patient A. has presented with a diffuse enlargement of the thyroid gland, an increase in basal metabolic rate, and subfebrile fever. His heart rate is 140 beats per min; breathing rate 22 per min; he has evidence of a fine tremor of the fingers and tongue; excessive sweating; marked exophthalmos. Uptake of I131 by the thyroid gland is increased by 20% compared with normal values. Blood levels of T3 and T4 are elevated; thyroidstimulating immunoglobulins are present in blood plasma. The patient complains of nervousness, restlessness, irritability, difficulty in concentrating. A 30-year-old patient B. also presents with diffuse enlargement of the thyroid gland, which developed 2 years after he had resigned from the military service on the atomic submarine. He has a decreased basal metabolic rate and body temperature, diminished heart rate, and lowered levels of T3 and T4 in the blood; thyroid-stimulating immunoglobulins are not found. Uptake of I131 by the thyroid gland is decreased by 5% below the normal value. The patient looks sluggish, apathetic, lethargic; his speech is slow and slurred; the voice is hoarse and muffled. The physician initiated an iodine therapy in both of these patients. Questions: 1. What forms of the thyroid gland pathology developed in the patients A. and B.? Substantiate your answer. 2. Explain the pathogenesis of the thyroid gland transformation in these patients. Explain differences in the mechanisms of enlargement of the thyroid gland in both cases?

111

3. Explain why the levels of T3 and T4 are increased in the patient A., and decreased in the patient B., despite the augmented uptake of I131 by the thyroid gland in both of them. 4. What are the mechanisms underlying each of the symptoms presented by these patients? 5. Was the prescription of iodine preparation to these patients appropriate? Can we expect the therapeutic effect iodine in both cases? N 134 A 36-year-old female patient X. has been admitted to hospital with complaints of recurrent attacks of retrosternal pain, palpitations, irregular heartbeats, fatigability, weakness, excessive sweating, frequent diarrhea, significant weight loss for the last 4 months, and subfebrile fever. According to the patient's history, the symptoms emerged 7 months ago in connection with emotional stress caused by family problems. On examination: the skin feels moist and warm; there is evidence of fine tremor of the fingers and tongue; signs of exophthalmos. ECG shows tachycardia, paroxysms of fibrillation, elevation of the S-T segment; the thyroid gland is slightly increased; blood pressure is 145/75 mm Hg. The physician made a preliminary diagnosis of "neurocirculatory asthenia", and prescribed tranquilizers and cardiotonic medicines. However, this therapy failed to improve the patient's condition. After additional evaluations the treatment regimen was changed to administration of propylthiouracil (a drug that blocks synthesis of thyroid hormones). This therapy was successful. Questions: 1. What additional data are needed for the final conclusion about the form of pathology presented by the patient? 2. What form of pathology does the patient suffer from given the presented symptoms and the assumed results of the laboratory tests? 3. What are possible variants of pathogenesis of this form of pathology? 4. Why did tranquilizers and cardiotonic drugs fail, and propylthiouracil succeed in improving the patient's condition? 5. What form of the cardiac pathology developed in the patient? Explain its pathogenesis.

112

N 135 A 42-year-old female patient N. has visited her physician with complaints of muscle weakness, persistent drowsiness, fatigability, constipation, cold intolerance, excessive weight gain (she gained 7 kg in the previous 2 months), and loss of hearing ability. She noted that her voice became hoarse over the last 3 months. Two years ago the patient had infectious parotitis and acute thyroiditis. On examination: the patient has excessive body weight, sluggish in her movements, answers slowly to questions; the skin feels dry and rough and has a yellowish tint; the face is puffy, with coarse features; the upper and lower extremities are pasty, but rebound fast after pressing with the finger tip. The patients body temperature is 35.60C; blood pressure is 100/60 mm Hg; heart rate 65 beats per min; breathing rate 13 per min. The thyroid gland is slightly enlarged, thick and tender on palpation; Achilles and other tendon reflexes are retarded. Special tests show a decrease in I131 uptake by the thyroid gland, an increase in blood level of thyrotropin and a high titer of antithyroglobulin antibodies. Questions: 1. What form of endocrine pathology developed in the patient? Substantiate your answer. 2. What is the likely cause of this pathology and its pathogenesis? 3. What mechanisms underlie each of the symptoms presented by the patient? N 136 A 66-year-old female patient T. has suffered from chronic thyroiditis complicated by hypothyroid state for 6 years. Two weeks ago she experienced a severe Salmonella food poisoning. Recently she went to the marketplace to do the shopping. It was a cold day (-160C), and after 10 min standing in a queue she felt bad. The bystanders called in an ambulance, and she was taken to hospital. On examination in the emergency room: the patient's condition is guarded; her mental status is depressed; the face is pasty; the hands, forearms, shins and feet appear puffy, but the tissue rebounds fast after pressing with the finger tip. Blood pressure is 60/40 mm

113

Hg; heart rate 55 beats per min; breathing rate 13 per min; body temperature 35.30C; deep tendon reflexes are slowed; abdominal distention is remarkable. Questions: 1. What pathologic state developed in the patient? Substantiate your answer. 2. What is the cause of this state? What are the main steps of its pathogenesis? 3. What mechanisms underlie each of the symptoms presented by the patient? 4. What approaches can be used to treat the patient in this pathologic condition? N 137 A 7-year-old boy has been admitted to hospital. On examination: the patient has signs of physical retardation - his height is below the physiological norm for the chronologic age group; he has a short neck and extremities, a big head, a pot belly, a flat nose bridge; the face is pasty and has a flat expression; he has thick lips and a large protruding tongue; the teeth are irregularly placed, and many are affected by caries. The patient's speech is slurred; he pronounces words incorrectly, responds slowly, and has difficulties in answering the simple questions; he looks capricious and nervous. The patient's blood pressure is 90/55 mm Hg; heart rate 65 per min. The thyroid gland has normal size, nontender on palpation. Blood test shows a significant decrease in T3 and T4 levels. The patient's mother during her pregnancy spent some time in the area contaminated by radioactive fission products. Questions: 1. What form of pathology does the patient suffer from? Substantiate your answer. 2. What are the likely cause of this pathology and its pathogenesis? 3. What mechanisms underlie each of the symptoms presented by the patient? PATHOPHYSIOLOGY OF THE NERVOUS SYSTEM. DISORDERS OF LOCOMOTION AND SENSATION Questions to prepare for classes and examination:

114

1. General etiology, pathogenesis, and typical forms of pathology of the nervous system. 2. Disorders of locomotion: manifestations of the upper (central) and lower (peripheral) motor neuron damage. 3. Disorders of the extrapyramidal system: muscle dystonia and dyskinesia. 4. Hypokinetic and hyperkinetic movement disorders: types, etiology and pathogenesis. 5. Disorders of sensation: general etiology and classification. 6. Positive and negative abnormal sensory phenomena. 7. Hypo- and hypersthesia, dysesthesia: types and mechanisms of disorders. 8. Levels of sensation disorders. 8. Localization of sensory abnormalities. 9. Types and mechanisms of ataxia. 10. Disorders of trophism. Denervation syndrome. N 138 A 59-year-old patient D., a physician, the day before his admission to hospital woke up in the morning, got up from bed, and noticed that he had difficulty in keeping the upright posture, permanently tilting to the left. After he was assisted in lying down on the bed he felt severe vertigo and nausea. When he again called for help he noticed the emergence of aphonia (a). About an hour later he noted emergence and then aggravation of paresthesia (b) on the right side of the body. When he was eating liquid meal (he could not eat solid meals because of nausea) regurgitaion (c) occurred. Neurologic evaluation shows the following: the leftside paresis (d) of the soft palate; a horizontal nistagmus (e) on lateral gaze, especially manifest when the patient is looking to the left; the left side facial and truncal hemihypesthesia (f); muscle hypotonia (g) and hyporeflexia (h) in the left extremities; finger-to-nose and heel-to-shin tests reveal ataxia (i) and coarse tremor (j) on the left side. The patient's blood pressure is 195/106 mm Hg; the area of cardiac dullness is increased to the left by 1.5 cm; heart rate is 90 beats per min.

115

Questions: 1. What types of the nervous system pathology developed in the patient? Substantiate your answer. 2. What is the likely cause of these types of pathology? Localize the foci of injury in the structures of the nervous system. Substantiate your answer. 3. Explain the meaning of the terms marked by letters. 4. What is the likely cause of the each of the observed symptoms? N 139 A 30-year-old patient K. experienced trauma of the left thigh with severe bleeding and supposed injury of the sciatic nerve. Two weeks later he noted the emergence of prickling and tingling sensation (a) in the shin and the sole of the injured leg. Then he began to suffer from attacks of spontaneous, diffuse, hardly tolerable, burning pain (b) that aggravated when the patient attempted to warm the affected leg. Thus, to soothe the pain he used to immerse his foot in cold water, but this measure gave only a temporary relief. Examination of the injured leg 2 months after the trauma shows: the skin of the foot is pale, dry, and slightly sloughing (c); light touch evokes pain (c); circumference of the thigh in its middle third is decreased by 4 cm (d) compared to the intact leg. Questions: 1. Make a conclusion about the form of the patient's pathology taking into account the observed symptoms. Substantiate your answer. 2. Name the symptoms marked by letters using the appropriate medical terms. 3. What are the possible mechanisms of the pain syndrome in this case? What factors are responsible for the pain in the second week and the second month after the trauma? N 140 A 42-year-old patient B. has been admitted to hospital with complaints of progressive weakness and loss of pain sensation in the right hand after injury; superficial skin abrasions in this hand usually give rise to persistent suppuration. These symptoms first appeared 5 years ago. During the last 6

116

months the patient noticed difficulty swallowing solid meals; his voice acquired nasal intonation. Examination by a neuropathologist revealed narrowing of the right palpebral fissure due to ptosis, a decrease of pain sense in the right face, drooping of the soft palate, absence of gag and cough reflexes in the pharynx, signs of vocal cords paresis on the right side, atrophy of the interosseous muscles and absence of periosteal reflexes in the right hand; segmental dissociated loss of pain and temperature sensations on the right side; tendon reflexes in the lower extremities are brisk. Questions: 1. Make a conclusion about the form of pathology in this patient using the described symptoms. 2. What are the likely mechanisms of development of the observed symptoms? 3. At what level of the structural organization of the nervous system could the observed injury produce: 1) the disorders of sensation, and 2) the locomotor abnormalities in this patient? 4. Taking into account the course of the disease and the pattern of the functional deficit, make a presumption about the likely cause of the nervous system pathology in this patient. N 141 A 38-year-old female patient K. about a year ago noted the emergence of shooting pain in the right neck after she had recovered from a viral infection. Later she noticed the development of numbness (a) in the great toe of the left foot. This loss of sensation gradually spreaded upward in the course of several weeks, and reached the level of the clavicle. In addition the patient began to suffer from progressive weakness of the left leg, and then of the left hand. Her right hand lost sensitivity to cold and warmth (b). Neurologic evaluation shows: tenderness of the neck paravertebral zones on the left side (c); atrophy of the left neck muscles; loss of deep sensation in the fingers of the left extremities; voluntary movements of the fingers and toes of the left extremities are limited (d); brisk tendon knee-jerk reflex; the presence of Babinski's reflex on the left side; column sensory loss (e) of pain and temperature sensation below C5 level. Cerebrospinal fluid examination

117

reveals: protein content 500 mg/L (N: 150-350 mg/L), total cell count 6 per ml (N: 3-4 per ml), hydrostatic pressure is 150 mm H2O (N: 100-180 mm H2O). Questions: 1. Make a conclusion about the form of pathology presented by the patient. Substantiate your answer. 2. Name the symptoms marked by letters using the appropriate medical terms. 3. What are the likely causes of the locomotion and sensation disorders in this patient? What pathologic process(es) could cause these disorders? 4. Positive Babinski's reflex characterizes what type of the nervous system disorder? What other neurologic symptoms are typical for this disorder? NEUROSES Questions to prepare for classes and examination: 1. Basics of conditioning: conditioned and uncoditioned responses. 2. Failure of the cortex functions as a pathological basis of experimental neurosis. 3. Types of experimental neurosis: neurosis with domination excitation, neurosis with domination of inhibition, neurosis due to overexertion of the lability function. 4. Neurosis in humans: definition of the concept, characteristic features, relation to experimental neurosis. 5. The main causes, prevalence, and predisposing factors of neurosis in humans. 6. Pathogenetic models of neuroses: biological, psychosocial, behavioral. 7. The main types of neuroses: neurasthenia, hysteric neurosis, compulsory neurosis; their characteristic features and pathological basis.

N 142 A 29-year-old female patient C. has been admitted to hospital with complaints of poor appetite, depressive mood, episodes of the vision, hearing, and taste sense losses; occasionally, "loss of voice"; frequent headaches.

118

According to her history in her childhood she was nervous, sensitive, capricious, cry-baby; lacked self-confidence. The symptoms presented on admission developed during the last year when she encountered serious life problems related to divorce and the need to find a new accomodation, faced conflicts at her work. During her stay in hospital the patient was unduly exacting, required persistent care of the personnel, blamed the hospital and her doctor for poor attention to her personality. She occasionally developed vomiting during her meal or immediately after it, especially when she was being observed by the medical personnel. When she was taking her physiotherapy procedures (massage, shower) she lost her consciousness three times. This was accompanied by a temporary paresis of hands. Somatic status is unremarkable. Questions: 1. What form of the nervous system pathology developed in this patient? Substantiate your answer. 2. What are causes of this pathology and conditions that promoted its development? 3. What other types of this pathology do you know? Characterize their main features. 4. Describe mechanisms which may underlie the development of this pathology. N 143 A 42-year-old patient N. was brought up by parents who thought that attaining high social status is the necessary prerequisite of happy life. Because of his parents' pressure the patient tried hard to excel his classmates at academic results. He spent a lot of time preparing for lessons though he had mediocre capabilities and studies came hard to him. After finishing high school he entered a technical college, since his parents insisted that he should continue education. His studies at the college required more efforts, and he spent much time preparing for his classes, sometimes working at night. When he graduated from the college he found a position of foreman at the factory. Soon after that a position of the shop superintendent became

119

vacant, and he took it though his professional basis and experience was not sufficient to get along with his new duties. He also lacked experience as a manager. Soon he met with serious problems. The shop ceased to fulfil the output programs, and he incurred criticism and reproaches of his bosses and the staff. At that time he started to suffer from headaches, painful sensations in the area of the heart, difficulty in falling asleep, irritability, fatigability, and a decreased capacity for work; he became over-anxious about his health. The current complaints are weakness, fatigability, difficulty in concentrating and falling asleep, dizziness, sweating, painful sensations in the heart area and skeletal muscles, rapid exhaustion after minimal mental activity, depressive mood, worry about decreasing mental and bodily well-being, anhedonia. On examination: blood pressure is 120/65 mm Hg, heart rate 72 beats/min; focal neurologic symptoms are not found. Questions: 1. What form of pathology developed in this patient? 2. What is the cause of this pathology in this case? 3. What factors and what personal traits of the patient could facilitate the development of this disorder? II. CLINICO-LABORATORY PROBLEMS Normal parameters of the aterial blood gases: Patm. Patm.O2 Palv.O2 PaO2 PvO2 PaCO2 SaO2 SvO2 750-765 mm Hg (atmospheric pressure) 158-160 mm Hg (partial pressure of O2 in the atmosphere) 105-110 mm Hg (partial pressure of O2 in the alveolar air) 95-100 mm Hg (partial tension of O2 in the arterial blood) 45-55 mm Hg (partial tension of O2 in the venous blood) 38-42 mm Hg (partial tension of CO2 in the arterial blood) 97-98% (saturation of the arterial blood by O2) 60-75% (saturation of the venous blood by O2) 19.5-21.0 vol% 6-8 L/min 5-6 L/min

Oxygen-carrying capacity Pulmonary minute volume Circulation minute volume

120

Define the type of hypoxia; explain its possible cause(s) and mechanisms of development: N 144 Patm.O2 Palv.O2 PaO2 PvO2 PaCO2 SaO2 SvO2 volume volume pH 160 mm Hg 105 mm Hg 96 mm Hg 60 mm Hg 30 mm Hg 98% 91% 7.3 L/min 6.9 L/min 7.31 N 145 Patm.O2 Palv.O2 PaO2 PvO2 PaCO2 MetHb SaO2 SvO2 volume volume pH Lactic acid N 146 Patm.O2 Palv.O2 PaO2 PvO2 PaCO2 SaO2 SvO2 volume volume pH Lactic acid 150 mm Hg 94 mm Hg 76 mm Hg 21 mm Hg 48 mm Hg 90% 32% 4.6 L/min 6.4 L/min 7.31 25 mg/dl N 147 Patm.O2 Palv.O2 PaO2 PvO2 PaCO2 Hb SaO2 SvO2 volume volume pH 158 mm Hg 105 mm Hg 96 mm Hg 18 mm Hg 28 mm Hg 140 g/L 95% 27% 8.8 L/min 2.9 L/min 7.31 158 mm Hg 105 mm Hg 95 mm Hg 38 mm Hg 40 mm Hg 40% 70% 20% 8.8 L/min 7.0 L/min 7.30 20.5 mg/dl

Pulmonary min. Circulation min.

Pulmonary min. Circulation min.

Lactic acid 26.5 mg/dl

Pulmonary min. Circulation min.

Pulmonary min. Circulation min.

121

Lactic acid N 148 Patm.O2 Palv.O2 PaO2 PvO2 PaCO2 SaO2 SvO2 volume volume pH 105 mm Hg 55 mm Hg 40 mm Hg 12 mm Hg 58 mm Hg 67% 11% 4.5 L/min 3.4 L/min 7.28 N 149* Patm.O2 Palv.O2 PaO2 PvO2 PaCO2 SaO2 SvO2 volume volume pH daily urine Hb

26.5 mg/dl

158 mm Hg 88 mm Hg 61 mm Hg 16 mm Hg 59 mm Hg 88% 25% 2.85 L/min 8.50 L/min 7.25 60 mmol/d 140 g/L

Pulmonary min. Circulation min.

Pulmonary min. Circulation min.

Titratable acidity of the

Normal parameters of the acid-base homeostasis of the capillary blood: Parameter pH PCO2 SB BB BE Ketone bodies Lactic acid Titratable acidity (of the daily urine) NH4+ (of the daily urine) 30-50 mmol/d Value 7.35-7.45 36-45 mm Hg 21-25 mmol/L 45-52 mmol/L +/- 2.5 mmol/L 0.5-2.5 mg/dl 6-16 mg/dl 20-40 mmol/d

122

Define the type of acid-base disorder and explain its possible mechanism of development: N 150 pH PCO2 SB BE 7.18 75 mm Hg 27 mmol/L +2.5 mmol/L N 151 pH SB BB BE 7.25 22 mmol/L 45 mmol/L +2 mmol/L PCO2 52 mm Hg

________________________ The patient is undergoing surgical operation with cardiopulmonary bypass N 152* pH SB BB 7.20 14 mmol/L 29 mmol/L PCO2 37 mm Hg

__________________________ The patient has a bronchial asthma attack N 153 pH SB BB BE Ketone bodies 7.36 19,5 mmol/L 39 mmol/L -5 mmol/L 7.5 mg/dl 37 mmol/d pCO2 36 mm Hg

BE -12 mmol/L ____________________ The patient is undergoing surgical operation with cardiopulmonary bypass

Titratable acidity

_________________________ The patient suffers from diabetes mellitus

N 154 pH pCO2 SB BB BE 7.30 38 mm Hg 18 mmol/L 36 mmol/L -6 mmol/L 45 mmol/d

N 155 pH pCO2 SB BB BE Titratable acidity 7.30 35 mm Hg 16.5 mmol/L 35 mmol/L -9 mmol/L 8 mmol/d

Lactic acid 26 mg/dl Titratable acidity ________________________ The patient is admitted to

NH4+ 17 mmol/d __________________________ The patient suffers from

123

hospital with a preliminary diagnosis of "acute myocardial infarction" N 156 pH pCO2 SB BB BE 7.22 36 mm Hg 14 mmol/L 24 mmol/L -8 mmol/L

"acute glomerulonephritis"

N 157 pH pCO2 SB BB BE 7.22 47 mm Hg 18.5 mmol/L 40.5 mmol/L -7 mmol/L

____________________ The patient has an intestinal fistula; he has been losing intestinal juice for a long period of time

Lactic acid 28 mg/dL Titratable acidity 8 mmol/d NH4+ 15 mmol/d ________________________ The patient suffers from severe toxic injury of liver and kidneys with oligouria

N 158 pH pCO2 SB BE BB 7.11 50 mm Hg 15.5 mmol/L -13 mol/L 38 mmol/L 58 mg/dL

N 159 pH pCO2 SB BB BE 7.17 51 mm Hg 18 mmol/L 45 mmol/L -8 mmol/L

Ketone bodies

_____________________ The patient suffers from acute left ventricular insufficiency with lungs edema

Titratable acidity 70 mmol/d ___________________________ The patient has coma

N 160 pH SB BE 7.52 20 mmol/L -2 mmol/L PCO2 25 mm Hg

N 161 pH SB BB 7.51 28 mmol/L 57 mmol/L PCO2 36 mm Hg

124

BB

40 mmol/L

BE

+5,5 mmol/L

___________________________ The patient is undergoing surgery with mechanical ventilation N 162 pH SB BE BB 7.59 28 mmol/L +5 mmol/L 50 mmol/L PCO2 30 mm Hg

_________________________ The patient suffers from uncontrolable vomiting, tetanus

N 163 pH SB BB BE 7.27 11 mmol/L 27 mmol/L -17 mmol/L PCO2 25 mm Hg

_______________________ The patient with brain contusion has cyclic vomiting and dyspnea N 164 pH PCO2 SB BE BB Lactic acid 7.22 49 mm Hg 19.5 mmol/L -3 mmol/L 44 mmol/L 23 mg/dL 15 mmol/d

Hematokrit 0.36 __________________________ The patient had an acute blood loss N 165 pH PCO2 SB BB BE Titratable acidity 7.33 35 mm Hg 18 mmol/L 42 mmol/L -5,5 mmol/L 12 mmol/L

Titratable acidity

Hematokrit 0.3 __________________________ The patient has been admitted to hospital with a diagnosis of hemolytic anemia of unclear etiology N 167

_________________________ The patient has been transported to the intensive care unit after surgical removal of thrombus in the abdominal aorta N 166 pH SB 7.57 25 mmol/L pCO2 28 mm Hg

pH SB

7.45 20 mmol/L

pCO2 30 mm Hg

125

BE BB

+4 mmol/L 48 mmol/L 12 mg/dL 20 mmol/d

BB BE

50.5 mmol/L -0.5 mmol/L

Lactic acid

Hematokrit 0.4 ________________________ The patient suffers from brain contusion and bouts of vomiting

Titratable acidity

________________________ The patient has had the hysteria hyperventilation episode just before the analysis N 168 pH pCO2 SB BE BB Lactic acid 7.09 51 mm Hg 15 mmol/L -12,5 mmol/L 38 mmol/L 25 mg/dL

N 169 pH SB BB BE 7.36 26,5 mmol/L 52,5 mmol/L +6 mmol/L pCO2 49 mm Hg

Titratable acidity 18 mmol/d ________________________ The patient was admitted to hospital 9 days before with a diagnosis of acute myocardial infarction

Titratable acidity 18 mmol/d _______________________ The patient suffers from purulent peritonitis

Normal results of the complete blood test with differential: Hemoglobin (Hb) Erythrocytes Mean corpuscular Hb (MCH) Reticulocytes Platelets Leukocytes Neutrophils: myelocytes metamyelocytes 0 0 130-160 g/L (men) 120-140 g/L (women) 4.0-5.0*1012/L (men) 3.9-4.7*1012/L (women) 25-33 pg 0.3-1.2% 180-320*1012/L 4.0-9.0*109/L

126

band segmented Eosinophils Basophils Lymphocytes/plasmocytes Monocytes

1-6% 47-72% 0.5-5.0% 0-1% 19-37%/0 3-11% 2-15 mm/h (women)

Erythrocytes sedimentation rate (ESR) 2-10 mm/h (men) Hematokrit (Ht) 0.40-0.48 (men) 0.36-0.42 (women) Characterize changes in complete blood tests with differential, and indicate their possible causes, mechanisms of development and consequences. Make a general conclusion on the results of tests. Symbol - indicates that the referred parameter has not been measured. Symbol ? indicates that the referred parameter should be calculated. N 170 Hb Erythrocytes MCH Reticulocytes Platelets Leukocytes Neutrophils: myelocytes metamyelocytes band segmented Eosinophils Basophils Lymphocytes Monocytes 0 4% 16% 60% 1% 0 15% 4% 0 0 0 3% 0 0 89% 8% Few lymphoblasts are present 150 g/L 4.7*1012/L ? 0.5% 220*109/L 17.0*109/L N 171 112 g/L 4.1*1012/L ? 0.2% 130*109/L 21.5*109/L

127

N 172 Hb Erythrocytes MCH Reticulocytes Platelets Leukocytes Neutrophils: myelocytes metamyelocytes band segmented Eosinophils Basophils Lymphocytes Lymphoblasts Monocytes ESR 0 0 2% 10% 0% 0 78% 7% 3% 68 mm/h * * Anisocytosis and microcytosis of erythrocytes. 34 g/L 1.0*10 /L ? 0% few 11.3*109/L
12

N 173 80 g/L 3.0*1012/L ? 0% 85*109/L 0.9*109/L 23%

10% 0 55% 12% **

**High blood levels of IgG, IgM, and factors of complement; increased blood concentration of unconjugated bilirubin. N 174 Hb Erythrocytes MCH Reticulocytes Platelets Leukocytes Promyelocytes Neutrophils: myelocytes 0 3% 18 g/L 0.8*10 /L ? 0.3% 180*109/L 88.0*109/L 0
12

N 175 162 g/L 4.7*1012/L ? 1.0% 250*109/L 39.0*109/L 3%

128

metamyelocytes band segmented Eosinophils Basophils Lymphocytes Lymphoblasts Monocytes N 176 Hb Erythrocytes MCH Reticulocytes Platelets Leukocytes Myeloblasts Promyelocytes Neutrophils: myelocytes metamyelocytes band segmented Eosinophils Basophils Lymphocytes Monocytes N 178 Hb Erythrocytes MCH Reticulocytes Platelets

0 1% 2% 1% 0 60% 35% 1%

10.5% 19% 25.5% 0 0 27% 0 12% N 177

100 g/L 3.6*1012/L ? 0.6% 200*10 /L 36.0*109/L 0 0 0 6% 9% 11% 60% 0 12% 2%


9

95 g/L 3.7*1012/L ? 3% 110*109/L 2.9*109/L 33% 1% 0 0 2% 10% 20% 0 32% 2% N 179

190 g/L 7.4*1012/L ? 1.6% 550*109/L

170 g/L 5.3*1012/L ? 0.5% 480*109/L

129

Leukocytes Myeloblasts Promyelocytes Neutrophils: myelocytes metamyelocytes band segmented Eosinophils Basophils Lymphocytes Monocytes ESR

11.0*109/L 0 0 0 0.5% 16% 56% 11% 1.5% 12% 3% 10 mm/h *

16.5*109/L 33% 1% 0 3% 16% 70% 1% 0 3% 7% 1 mm/h **

* Significant polychromatophylia of erythrocytes, few normoblasts. **Anisocytosis and macrocytosis of erythrocytes. N 180* Hb Erythrocytes MCH Reticulocytes Platelets Leukocytes Neutrophils Eosinophils Basophils Lymphocytes Lymphoblasts (or myeloblasts?) Monocytes 12% * * Anisocytosis and microcytosis of erythrocytes. **Positive reaction for myeloperoxidase. 5% ** 82 g/L 3.02*1012/L ? 0 0.15*109/L 0.5*109/L 0 10% 0 78% 0 N 181 64 g/L 2.0*1012/L ? 0 120*109/L 84.0*109/L 2% 0 0 2.5% 90.5%

130

N 182 Hb Erythrocytes MCH Reticulocytes Platelets Leukocytes Promyelocytes Neutrophils: myelocytes metamyelocytes band segmented Eosinophils Basophils Lymphocytes Lymphoblasts Monocytes ESR N 184 Hb Erythrocytes MCH Reticulocytes Platelets Leukocytes Myeloblasts Promyelocytes Neutrophils: myelocytes metamyelocytes band 0 0 0 76 g/L 2.8*1012/L ? 0.1% 0.8*10 /L 128*109/L 97% 0.5%
9

N 183 168 g/L 4.8*10 /L ? 240*109/L 41.0*109/L 2% 5.5% 28% 20.5% 26.5% 0 0 6% 0 11.5% 12

34 g/L 1.08*1012/L ? 0.1% few 1.3*109/L 0 0 0 2% 10% 20% 0 50% 16% 2% 80 mm/h N 185 99 g/L 3.8*1012/L ? 0 5.0*109/L 2.8*109/L 31% 1% 0 0 4%

131

segmented Eosinophils Basophils Lymphocytes Monocytes N 186 Hb Erythrocytes MCH Leukocytes Myeloblasts Promyelocytes Neutrophils: myelocytes metamyelocytes band segmented Eosinophils Basophils Lymphocytes Monocytes N 188 Hb Erythrocytes MCH Platelets Leukocytes Promyelocytes Neutrophils: myelocytes metamyelocytes band

2.5% 0 0 0 0

10% 22% 0 30% 2% N 187

58 g/L 3.1*10 /L ? 182*109/L 4% 12% 4.5% 10% 8% 37.5% 5% 9% 0 0


12

62 g/L 3.9*1012/L ? 67.5*109/L 0 10% 4.5% 6% 39% 22% 0.5% 0 14.5% 3.5% N 189

98 g/L 3.9*1012/L ? 200*109/L 38.0*109/L 0 0 3% 14%

108 g/L 4.0*1012/L ? 490*109/L 23.9*109/L 2.5% 4% 6% 14%

132

segmented Eosinophils Basophils Lymphocytes Monocytes N 190 Hb Erythrocytes MCH Reticulocytes Leukocytes Promyelocytes Neutrophils: myelocytes metamyelocytes band segmented Eosinophils Basophils Lymphocytes Monocytes N 192 Hb Erythrocytes Ht MCH Reticulocytes Leukocytes Promyelocytes Neutrophils: myelocytes metamyelocytes

62% 8% 0 12% 1%

43% 4.5% 3% 18% 5% N 191

115 g/L 3.5*10 /L ? 0.4% 14.8*109/L 0 0 0 3.5% 42% 0.5% 0 43% 11%
12

80 g/L 2.9*1012/L ? 0.1% 38*109/L 2% 6% 10% 17% 51% 0.5% 0 12% 1.5% N 193

128 g/L 4.1*1012/L ? 0.4% 42*109/L 2% 3% 9%

118 g/L 3.6*1012/L 0.38 ? 0.3% 3.8*109/L 0 0 0

133

band segmented Eosinophils Basophils Lymphocytes Monocytes

14% 44% 4.5% 1.5% 20% 2%

4% 56% 4.5% 1% 30% 4.5%

Normal parameters of bilirubin pigments metabolism Peripheral blood: Total bilirubin Unconjugated bilirubin Conjugated bilirubin Urobilinogen Stercobilinogen < 1.3 mg/dl < 1.0 mg/dl < 0.3 mg/dl not detectable detectable

Define changes in bile pigments level and composition in biological fluids and explain possible causes and mechanisms of their development. Make a general conclusion on the given data. N 194* blood: Total bilirubine Conjugated bilirubine Urobilinogen Stercobilinogen Cholic acids 2.6 mg/dl 0.15 mg/dl + increased not found N 195 blood: Total bilirubine Conjugated bilirubine Urobilinogen Stercobilinogen Cholic acids 2.4 mg/dl 1.4 mg/dl trace amounts trace amounts + urine: + trace amounts trace amounts + stool: very low urine: not found + increased not found stool: increased -

134

Alanine aminotransferase Aspartate aminotransferase

increased increased N 196 blood

urine: trace amounts + norm not found

stool: -

Total bilirubine Conjugated bilirubine Urobilinogen Stercobilinogen Cholic acids

1.5 mg/dl 0.5 mg/dl + norm not found N 197 blood:

urine: trace amounts not found decreased + -

stool: decreased -

Total bilirubine Conjugated bilirubine Urobilinogen Stercobilinogen Cholic acids Alanine aminotansferase Aspartate aminotransferase

2.43 mg/dl 0.35 mg/dl not found decreased + very high very high N 198 blood:

urine: + not found not found + -

stool: not found -

Total bilirubine Conjugated bilirubine Urobilinogen Stercobilinogen Cholic acids Alanine aminotransferase Aspartate aminotransferase

2.4 mg/dl 1.4 mg/dl not found not found + norm norm N 199 blood:

urine: -

stool: -

Total bilirubine

1.8 mg/dl

135

Conjugated bilirubine Urobilinogen Stercobilinogen Cholic acids

0.2 mg/dl not found norm not found N 200 blood:

+ not found norm not found

norm -

urine: + not found + -

stool: -

Total bilirubine Conjugated bilirubine Urobilinogen Stercobilinogen Cholic acids Alanine aminotransferase

3.5 mg/dl 2.1mg/dl not found trace amounts + increased N 201 blood:

trace amounts decreased

urine: trace amounts + increased not found -

stool: norm -

Total bilirubine Conjugated bilirubine Urobilinogen Stercobilinogen Cholic acids Alanine aminotransferase

4.4 mg/dl 0.35 mg/dl + increased not found increased N 202 blood

urine: not found not found increased not found -

stool: increased -

Total bilirubine Conjugated bilirubine Urobilinogen Stercobilinogen Cholic acids Alanine aminotransferase

2.1 mg/dl 0.2 mg/dl not found increased not found norm N 203 blood:

urine:

stool:

136

Total bilirubine Conjugated bilirubine Urobilinogen Stercobilinogen Cholic acids Alanine aminotransferase

3.6 mg/dl 2.5 mg/dl not found not found not found norm

+ not found not found -

not found trace amount

Normal parameters of renal function: Diuresis Specific gravity Protein Glucose Ketone bodies Microscopy of the sediment Erythrocytes: few per high power field (h.p.f.) Leukocytes: few per h.p.f. Hyaline casts: few in the sample. Additional data Glucose of blood plasma 65-110 mg/dl N 204* Diuresis Specific gravity Protein Glucose Ketone bodies red blood cell casts. Additional data: blood pressure 165/105 mm Hg; blood urea nitrogen 50 mg/dl (N:20-40 mg/dl). High titers of antistreptolysin O in blood plasma. N 205 Diuresis 2800 ml 800 ml 1029 1 g/l not found not found 800-1200 ml/d 1.018-1.025 absent absent absent

Microscopy: 40-50 erythrocytes per high power field; few hyaline and

137

Specific gravity Protein Glucose Ketone bodies hyaline casts.

1009 2 g/l not found not found

Microscopy: few erythrocytes per high power field; Additional data: blood pressure 185/100 mm Hg; blood urea nitrogen 80 mg/dl (N:20-40 mg/dl). N 206 Diuresis Specific gravity Protein Glucose Ketone bodies hyaline, waxy, granular casts. Additional data: blood pressure 175/95 mm Hg; blood urea nitrogen 190 mg/dl (N:20-40 mg/dl). N 207 Diuresis Specific gravity Protein Glucose Ketone bodies 1000 ml 1037 33 g/l not found not found 420 ml 1011 2 g/l not found not found

Microscopy: few erythrocytes per high power field;

Microscopy: large amount of granular and waxy casts. Additional data: blood pressure 120/65 mm Hg; blood urea nitrogen 35 mg/dl (N:20-40 mg/dl). N 208 Diuresis Specific gravity Protein 1600 ml 1025 not found

138

Glucose Ketone bodies Blood glucose N 209 Diuresis Specific gravity Protein Glucose Ketone bodies Blood glucose N 210 Diuresis Specific gravity Protein Glucose Ketone bodies

2,5% not found 80 mg/dl

5500 ml 1040 not found 4% positive reaction 300 mg/dl (N:80-120 mg/dl)

340 ml 1035 1,5 g/l not found not found

Microscopy: 30-35 erythrocytes per high power field; granular and erythrocyte casts 2-3 per h.p.f. Additional data: blood pressure 145/100 mm Hg; blood urea nitrogen 210 mg/dl (N:20-40 mg/dl) N 211 Diuresis Specific gravity Protein Glucose Ketone bodies 165 ml 1015 3,5 g/l not found not found

Microscopy: few erythrocytes; hyaline casts. Additional data: blood pressure 145/90 mm Hg; blood urea nitrogen 48 mg/dl (N:20-40 mg/dl). N 212

139

Diuresis Specific gravity Protein Glucose Ketone bodies Microscopy: few hyaline casts per h.p.f.

3800 ml 1014 not found not found not found

Additional data: blood pressure 115/80 mm Hg; blood urea nitrogen 35 mg/dl (N:20-40 mg/dl). N 213 Diuresis Specific gravity Protein Glucose Ketone bodies Titratable acidity 1600 ml 1020 not found 1,5% not found 5 mmol/L

Microscopy: large amounts of salt cristals. Additional data: positive reaction for cystein and arginine in the urine, high concentration of phosphates. III. EXAMPLES OF CASES DISCUSSION N 93 A 40-year-old patient Z. suffers from tuberculosis. He has come to see his physician with complaints of dyspnea, pains in the right upper quadrant, and subfebrile fever. These symptoms emerged 4 weeks ago and were progressing. On examination: the face is pale and rounded, orthopnea is evident (the patient is sitting upright with his hands resting upon chair; this posture is necessary to ease breathing); the area of cardiac dullness is increased to the left and to the right by 2 cm, heart rate is 100 per min, blood pressure is 90/60 mm Hg. On auscultation: muffled heart sounds, crackles at the base of the lungs. Breathing rate is 26 per min. The neck veins are distended; the liver is tender on palpation, and extends 3 cm below the costal margin; there is trace pretibial and feet edema. X-ray examination of the chest shows the globular shadow of the heart.

140

Questions: 1. Does the patient have signs of cardiac insufficiency? Substantiate your answer. 2. What additional tests are required to specify the form of the heart pathology in this case? 3. Make a conclusion about the form of cardiac insufficiency in this patient. Answers: 1. The patient has cardiac insufficiency manifested by orthopnea, tachycardia, rales in the lungs, distention of the neck veins, enlargement of the liver, edema of the lower extremities. 2. Ultrasound investigation may show the presence of fluid in the pericardial cavity. 3. Total diastolic heart insufficiency caused by tuberculous pericarditis which complicates tuberculosis in this patient. Fluid in the pericardial cavity compresses the heart and disturbs filling of ventricles. N 107 A 36-year-old patient K., a mine worker, has been admitted to hospital with a preliminary diagnosis of silicosis. He complains of dyspnea, more severe on walking and physical exercise; sustained cough (occasionally with secretions), chest pain. _____________________________________________________ Arterial blood gases: PaO2 PaCO2 Oxygen carrying capacity SaO2 Spirometry: Forced vital capacity (FVC) FVC in % to normal Forced expiratory volume in 1 s (FEV1) Tiffeneau ratio (FEV1/FVC) Pulmonary minute volume (% of normal) Additional data: 4.2 L 92% 2.6 L ? 124% 90 mm Hg 40 mm Hg 19.2 vol% 94.3%

141

Breathing rate

19/min

_________________________________________ The voluntary hyperventilation test yields the PaO2 value of 92 mm Hg. Questions: 1. Does the patient have disturbances of respiratory function? Describe the signs of it, if he does. 2. Does the patient have disturbances of alveolar ventilation? If he does, define its type (obstructive or restrictive). 3. Taking into account the preliminary diagnosis of pneumoconiosis how to test the diffusive capacity of the lungs in this patient? 4. Make a final conclusion about the type of respiratory system disorder in patient K. Answers: 1. Symptoms of respiratory disorder in this patient include dyspnea, increased breathing rate (BR) and pulmonary minute volume (PMV), hypoxemia, decreased Tiffeneau ratio, lack of significant changes in PaO2 after voluntary hyperventilation. 2. The patient has signs of disturbance of alveolar ventilation mainly of obstructive type. The former is confirmed by a decrease in the Tiffeneau ratio (< 62%) and increased BR and PMV values. 3. The diffusive capacity of the lungs may be evaluated by the voluntary hyperventilation test. In this case PaO2 (92 mm Hg) shows little changes during the test as compared with the initial values. Hence, the diffusive capacity of the lungs for O2 is decreased. 4. General conclusion: the patient has a disorder of respiratory function due to alveolar hypoventilation of the obstructive type (presumably, as a result of obstruction of the airways by mucous secretions), and due to a decrease of the diffusive capacity of the aerohematic barrier N 120 A victim of the vehicle accident was transported to hospital 5 hours after the accident. He was examined by a physician of the emergency service who found multiple ribs fractures, contusions of soft tissues of the pelvis and lower extremities with extensive hemorrhages. On admission the patient is .

142

confused, markedly pale; he has a thready pulse, blood pressure 60/20 mm Hg, periodic pattern of breathing. After a day of intense treatment by volume expanders (he received 3 L of polyglukin) and infusion of 0.5 L of blood his blood pressure increased to 110/60 mm Hg. On the next day after infusion therapy diuresis was still absent, and during the following three days the patient's condition remained guarded. He complained of severe headache, dizziness, cyclic vomiting, inhibited state. The patient had short-termed episodes of seizures, edema of the skin; bradycardia and extrasystole on ECG records; his diuresis was at the level of 150-250 ml/day, but blood pressure increased to 160/90 mm Hg. Blood test shows: BUN (blood urea nitrogen) 90 mg/dl (N: 20-40 mg/dl), hyperkalemia, hypermagnesimea, hyponatremia, hypochloremia, pH 7.30. Urine test: specific gravity 1.040, mild proteinuria, myoglobinuria; sediment shows the presence of casts and few leukocytes in the h.p.f. On the 5-7th day in the hospital the patient developed great elevation of diuresis (up to 2500 ml/day), and his condition markedly improved. Vomiting, seizures and headache ceased, and the extent of edema reduced. Repeated urinalysis shows: specific gravity 1.010-1.012, slight proteinuria, large quantity of granular casts in the urine sediment. Questions: 1. Define the type of renal syndrome in this patient. What are its causes? 2. What are the causes of anuria during the shock before normalization of blood pressure in this patient? 3. Why was diuresis not restored despite extensive transfusion theraspy? 4. What are the mechanisms of symptoms observed on the 2-4th day after the trauma? Answers. 1. Acute renal failure results from traumatic shock and occlusion of renal tubules by myoglobin (myoglobin-induced nephrosis). 2. Low blood pressure in the renal glomeruli. At blood pressure levels below 70 mm Hg renal blood flow decreases by 90% or more, and filtration pressure in the glomerular capillaries becomes negligible. 3. Renal ischemia caused by shock and obturation of renal tubules by myoglobin leads to detachment of tubular epithelia and blocking of the

143

tubules' lumen. The latter increases the interstitial pressure in the medulla and may further compress the tubules. High pressure of the primary filtrate reduces the efficiency of glomerular filtration. Combination of these factors precludes the normalization of diuresis. 4. Severe headache, dizziness, frequent ( intractable) vomiting, psychomotor retardation suggest the development of brain edema caused by retention of water in the body. Brain edema is also promoted by increased arterial pressure. Increased levels of blood urea nitrogen, hyperkalemia and hypermagnesimea result from a decreased rate of glomerular filtration and impairment of tubular reabsorption. Decreased blood levels of sodium and chloride are caused by intractable vomiting that could have led to development of hypochloremic alkalosis. However, disorders of the tubular function result in impairment of acidogenesis and ammoniogenesis leading to renal excretory acidosis. The following increase in daily urine output and low specific gravity of the urine suggest a disturbance of the kidney concentrating ability. N 132 An endocrinologist observes two patients during their follow-up visits: a 50year-old woman M. and her 26-year-old daughter D. Both patients have a significant enlargement of their thyroid gland and clinical signs of thyrotoxicosis. The results of their clinical and laboratory investigations suggest the diagnosis of the Graves disease. The patient D.s condition improved after a treatment course, while the patient M.s condition kept worsening after a 8-months course of medication by thyrostatic drugs. During reexamination M. presented with new complaints of sluggishness, motor retardation, drowsiness during the day time and difficulty going to sleep at night, memory problems, decreased working ability, edema of the face and extremities and cold intolerance. These symptoms emerged after M. had recovered from viral infection. The physician suspected the presence of Hashimotos thyroiditis in M., and changed the treatment. Questions: 1. Can we consider the emergence of new symptoms in M. as complication of the thyrostatic therapy?

144

2. What laboratory tests can be made to confirm the diagnosis in M.? 3. What other clinical forms of thyroid gland pathology should be considered to make the final diagnosis? 4. Are there common mechanisms in the pathogenesis of Graves disease Answers. 1. Thyrotoxicosis is characterized by: - cardiovascular disorders, - disorders of the central nervous and autonomous system, - activation of catabolic processes that manifest in wasting, subfebrile body temperature, myopathy, and osteochondrosis, - gastrointestinal disorders, - disorders of endocrine glands, such as adrenal insufficiency, ovarian dysfunction, fibrocystic breast disease. Inappropriate use of thyrostatic drugs may induce hypothyroidism. 2. A more accurate diagnosis may be obtained by determination of T3,T4 and TSH levels in blood, immunological tests (antithyroglobulin and antithyroid peroxidase antibodies, antibodies against the TSH receptor), needle aspiration of the thyroid gland (it may reveal the histologic signs of the autoimmune thyroiditis). 3. Differential diagnosis may include acute and subacute thyroiditis. 4. The common step is the involvement of immunologic mechanisms. Antithyroglobulin and antimicrosomal antibodies as well as thyroid-stimulating immunoglobulins have been found in plasma of patients with Hashimotos thyroiditis. These antibodies form immune complexes with the relevant antigens and cause destruction of thyrocytes. At the initial stage injury to the thyroid gland may release high quantities of thyroid hormones in the blood producing thyrotoxicosis. Later, with progressive destruction and lymphocytic infiltration of follicles the hyperfunctional state transfers into the functional insufficiency with a characteristic clinical picture. N 149 Define the type of hypoxia and explain its possible cause(s) and and Hashimotos thyroiditis?

145

mechanisms of development: Patm.O2 Palv.O2 PaO2 PvO2 PaCO2 SaO2 SvO2 Pulmonary minute volume 2.85 L/min (N: 6-8 L) Circulation minute volume 8.50 L/min (N: 5-6 L) pH Lactic acid the daily urine Hb Conclusion. Complex respiratory and circulatory type of hypoxia. The respiratory type is due to hypoventilation because PaO2 is decreased and PaCO2 is increased l while pulmonary minute volume is low. The circulatory type is confirmed by high arterial-venous difference in O2 : SaO2 - SvO2. A decrease in pH is caused by accumulation of lactic acid and H2CO3 in blood. Renal function, judging by the ability to excrete H+ (titratable acidity), is normal. N 152 Define the type of acid-base disorder and explain its possible mechanisms. pH pCO2 SB BB BE 7.20 37 mm Hg 14 mm/L 29 mm/L -12 mm/L 7.25 20.0 mg/dl (N: 6-16 mg/dl) 60 mmol/d 140 g/L 158 mm Hg 88 mm Hg 61 mm Hg 16 mm Hg 59 mm Hg 88% 25%

Titratable acidity of

The patient is undergoing surgical operation with cardiopulmonary bypass

146

Conclusion The patient has an uncompensated acidosis (pH level is below 7.35). This acidosis is not respiratory since there is no increase in pCO2 and SB. Decrease in SB suggests the accumulation of nonvolatile acids in blood plasma. Taking into account the described circumstances we may assume the presence of metabolic acidosis caused by circulatory insufficiency. N 180 Characterize changes in blood test and explain their possible causes, mechanisms of development and consequences. Make a general conclusion on the given data. Hb Erythrocytes MCH Reticulocytes Platelets Leukocytes Neutrophils Eosinophils Basophils Lymphocytes Monocytes Conclusion. There are signs of hypochromatic aregeneratory anemia of the normoblastic type; thrombocytopenia, leukopenia; relative eosinophylia, but the absolute content of eosinophils is within the normal range; relative lymphocytosis and absolute lymphopenia; relative monocytosis and absolute monocytopenia. Aplastic anemia may be preliminary diagnosed since there is evidence of impairment of all the hematopietic cell lines. N 194 82 g/L 3.02*1012/L to be calculated 0 0.15*109/L 0.5*109/L 0 10% 0 78% 12%

* Anisocytosis and microcytosis of erythrocytes.

147

Define changes in bile pigments content and composition in biological fluids and explain their possible causes and mechanisms of development. Make a general conclusion on the given data. blood: Total bilirubine Conjugated bilirubine Urobilinogen Stercobilinogen Cholic acids Conclusion. The patient has increased levels of total blood bilirubine owing to unconjugated fraction (2.60 - 0.15 = 2.45 mg/dl). The increased content of stercobilinogen in feces and urine suggests the increased release of conjugated bilirubine in the small intestine. These data suggest that elevation of unconjugated bilirubine in blood is caused by its increased production due to hemolysis of erythrocytes. Elevation of urobilinogen levels in blood and urine results from its enhanced absorption in the intestine and failure of hepatocytes to completely remove it from the blood due to saturation of the membrane transporter. General conclusion: the patient has hemolytic jaundice. N 204 Define changes in urinalysis and explain their possible causes and mechanisms of development. Make a general conclusion on the given data. Diuresis Specific gravity Protein Glucose Ketone bodies 800 ml 1.029 1 g/L not found not found 2.6 mg/dl 0.15 mg/dl + increased not found urine: not found + increased not found stool: increased -

Microscopy of the urine sediment: Erythrocytes 40-50 per high power field, few hyaline and red blood cell casts. Additional data: Blood pressure 165/105 mm Hg

148

BUN Conclusion: 1. Oligouria. 2. Hypersthenuria. 3. Proteinuria. 4. Hematuria.

50 mg/dl

High titers of antistreptolysin O in blood plasma.

5. Significant excess of hyaline and red blood cell casts. 6. Arterial hypertension. 7. Azotemia. 8. The presence of antistreptolysin O in blood. The patient has signs of nephritic syndrome. Hypersthenuria suggests that tubular function is not affected. Oligouria, hematuria, arterial hypertension and azotemia point to the presence of injury to the renal glomeruli. This pathologic process is supposed to be acute, since the chronic process is characterized by injury to all parts of the nephron. General conclusion: acute diffuse glomerulonephritis caused by streptococcal infection; renal arterial hypertension.

Anda mungkin juga menyukai